You are on page 1of 180

A Unique Math E-Book for All

Competitive Exam

Solution, Edited & Completed

by

Yousuf Ali
Section-A: Most Wanted Bank Math
Question :এক এক ঑ ১০% ক
৬৩০ ক , ক ?
[Sonali Bank officer -2014]
[Same As SEBL-2015]
Solution-1:
Let,
Cost Price=X Tk
10% profit on cost price Then market price=X + X of 10%
=1.1 X Tk
Again 10% profits on market price
Then Selling price
=1.1x + 1.1x of 10%
=1.21x Tk
Now profits=Selling - Cost Price
=(1.21x - x) Tk
=0.21x Tk
When,
Profit .21x Tk then Cost Price=X Tk
--------630 Tk--------------------------
=[(630*x)/.21x]
=3000 Tk
Solution-2:
Let,
Cost Price of the article=100 Tk
10% profit on cost price then
Market price=(100 + 100 of 10%)
=110 Tk
Again,
10% profits on market price then selling price=(110+110 of 10%) Tk
=121 Tk
Now,
Profit=(121-100)=21 Tk
When
Profit 21 Tk then Cost Price=100 tk
--------630 Tk -----------------------------------=[(100*630)/21]
=3000 Tk
Answer:3000 Tk

Solution-3:====
Short Cut/MCQ Way
Let , Cost Price=100 Tk
10% profits on both way
Profit
=10+10+[(10*100)/100]
=21 Tk
Profit 21 then CP =100 TK
-------630 TK --------=[100*630/21]
=3000 TK
Answer:3000 Tk

*****Part -4********
Question :If 5 is added to the sum of two digits a number consisting of two digits,the sum
will be three times the digits of the tenth place.Moreover if the place of the digits are
interchanged,the number thus found will be 9 less the original number.What is the number?
[Dhaka Bank MTO=2016]
Same As:
**Rupali Bank-2013
**Union Bank-2014
**HBFC SO-2015
**Janata Bank-2015
SO ON+++++
MCQ:
**Jamuna Bank MTO-2012
**ATEO-2005
**DBBL PO-2012
**BB AD-2012
**ICB -2008
SO ON-------

Solution: Let,
Unit digit = x
Tenth Digit = y
Original Number =10y+x
According to the First Condition,
x+y+5= 3y
Or x= 2y – 5 -----------[1]
Again
According to the question,
10x+y = 10y+ x – 9
Or, 9x – 9y= -9
Or, 9(2y–5) – 9y= -9
[Putting x value ]
Or, 18y – 45 – 9y = -9
Or, 9y = 36
Or, y = 4
Then, x = 2y – 5 = 2×4– 5 = 3
So the number =[10*4+3]=43
Answer :43

Bonus Math
SIBL[TSO]- 2016
Question :A man running speed is 3 times of walking speed. The man run a distance and
came back walking. Total time taken by him was 2 hrs. What is the distance in miles if he
runs 9 miles per hour?
Solution:Let,
Distance =x
Running speed = 9m/hr
So, walking speed = 9/3 = 3 m/hr According to the question,
x/9 + x/3 = 2
or, (x+ 3x)/9 = 2
or, 4x= 18
or, x= 4.5
Answer :4.5 miles

Question :The length of a rectangular field is 30 feet more than its width and the perimeter is
380 feet.What is the area of the field in square feet?
Solution:Let,
Width = x feet
Length = (x + 30) feet
According to question,
2{x + (x + 30)} = 380
or, 2x + 30 = 190
or, x = 160/2
or,x= 80
Now
Width = 80 feet
Length = (80 + 30) feet = 110 feet
So
Area of rectangle be
=110*80 = 8800 sq. feet
Answer :8800 sq.feet

♦ Similar But aktu difference


Question :The length of a rectangular field is 30 feet more than its width and the perimeter is
380 feet. How much would it cost to cover the field with grass at the rate of 20 Tk per square
feet?
Solution:Let,
Width = x feet
Length = (x + 30) feet
According to question,
2{x + (x + 30)} = 380
or, 2x + 30 = 190
or, x = 160/2
or,x= 80
Now
Width = 80 feet
Length = (80 + 30) feet = 110 feet
So
Area of rectangle be
=110*80 = 8800 sq. feet
Cost to cover by grass
=8800*20
=1,76,000 Tk
Answer :176000 Tk

Question :A borrower pays 6.5% interest per year on the first Tk 600, he borrowed and
7.25% per year on next part of the loan in excess of Tk 600.How much interest will the
borrower pay on a loan of tk. 6500 for 1 year?
Solution:
Here,
Interest for the frist Tk 600
=[6.5*600]/100 =39Tk
Rest of the amount
=[6500 – 600]=5900 Tk
Interest for rest amount
=[7.25*5900]/100 = 427.75 Tk
Total interest = [427.75 + 39]
= 466.75 Tk
Answer :466.75

*****Part-3********
Question:ক এক ক ৩০ ,খ ১৫ এ গ ১০ ক ২ খএ
৩ গ,ক ক ক ঐক ক ?[BKB SO-2015]
[A can do a piece of work in 30 days,B can do it in 15 days and C can do it in 10 days.Every
second days B and every third days C help A .How many days will finish the whole work?]
[Bangladesh Krishi Bank SO-2015]
More:Same As
**34th BCS WRITTEN
**33th BCS WRITTEN
**21th BCS WRITTEN
**PSC Non cader
Precaution:=========
Most of the Guide Book this question answer is wrong
Such as 12/10 etc
Correct Answer:11 Days
Solution:1
Let,
Total work=1 portion
LCM of 2 & 3=6
Per 6 days A work= 6 days
Per 6 days B work=(6/2)=3 days
Per 6 days C work=(6/3)=2 days
So,
6 days (A+B+C)'s work
=(6/30+3/15+2/10) portion
=3/5 portion
Remaining work
=(1-3/5) portion
=2/3 portion
After 2 days(A+B)'s work
=(2/30+1/15)portion
=2/15 portion
[Every second days B help A]
Remaining work
=(2/5-2/15) portion
=4/15 portion
Total time=(6+2)=8 days
9th day(A+C) work
=(1/30+1/10) portion
=2/15 portion
Remaining work
=(4/15-2/15)portion
=2/15 portion
Another 2 days(A+B)'s work
=(2/30+1/15) portion
=2/15 portion
Remaining work
=(2/15-2/15)
=0
So total time taken to finish the work=(9+2)=11 days
Answer:11 days
Solution:2==================
এখ ,
২঑৩এ =৬
৬ ক ক ক,খ,গ এক এ ৬ ক ক
=(৬/৩০)+(৩/১৫)+২/১০)
=৩/৫
ক ক =(১-৩/৫)
= ২/৫
২/৩<৩/৫
ক,খ,গ এক এ ৩ ক ক =(৩/৩০+১/১৫+১/১০)
=৪/১৫
ক ক =(২/৫-৪/১৫)
=২/১৫
২/১৫<৪/১৫
ক এক ১/৩০ ক ক
ক ক =(২/১৫-১/৩০)
=১/১০
এখ ক, খ এক এ ক ১/১০ ক
ক, খ এক এ ১ ক ক =(১/৩০+১/১৫)=১/১০ ক
ক ক :(১/১০-১/১০)=0

=৬+৩+১+১=১১
Answer :11 days

Part-2=======================
Question: In a flight of 600 km ,an aircraft was slowed down due to bad weather.Its average
speed for the trip was reduced by 200 km/hr and the time of flight increased by 30
minutes.The duration of the flight is.[Indian Bix]
[Rupali Bank Written-2013]
Answer Type
A.1 hour B.2 hrs
C.3. hrs D.4 hrs
Answer:1 hour
**Most of JoB Guide the answer is
(60+30) minutes=1.5 hrs which is incorrect
-
*Duration of the flight always fixed
* গ ক খ fixed ক খ
* ক কঘ ড় ৎ গ

Solution:1==================
Let,
The duration of the flight be=x hrs
Original distance be=600 km
According to the question,
Original speed-Reduced speed=200
600/x-600/(x+1/2)=200
Or,600/x-1200/(2x+1)=200
Or,2/x-{6/(2x+1)}=1
Or,(6x+3-6x)/{x(2x+1)}=1
Or,2x^2+3x-2x-3=0
Or,x(2x+3)-1(2x-3)=0
Or,(2x+3)(x-1)=0
Now,
X-1=0
Or,X=1
And
2x+3=0
Or,X=-(3/2)
[neglecting the negative value]
Answer:1 hour
============================
Solution:2
Let,
Original time be= t hrs
Original speed be=S km/hrs
Here,Distance (D)=600 km
According to the first condition,
D=st
Or,s=D/t
Or,s=600/t=======(1)
According to the 2nd condition,
600=(s-200)*(1/2+t)
{(Putting value s=600/t) 2nd condition}
======================
======================
======================
Or,(2t+3)(t-1)=0
Now,
2t+3=0
Or,t=-(3/2)
[neglecting Negative value]
And
t-1=0
Or,t=1
So Duration of the flight 1 hour
Answer:1 hour

**********Part-1**************

Question :A man sells an article at a profit of 25%. If he had bought it at 20% less and sold it
for Tk.10.50 less, he would have gained 30%. Find the cost price of the article.
[Bangladesh Bank AD-2017] Mark-15
#More Written Exam Same As*
*SBC AM -2016
*BKB -2012
*NBL-2014
*BCS Written
*Non cader Written
*INDIAN BIX
*BAPEX
*Islamic Bank-2013
*BRC-2008
*So on
#Preliminary Exam:
*16th BCS
*MBA-2007-08;1999;1997
*NCC BANK-2002
*PBL-2000
So on=====
Solution-1:
Let,
Cost price of the article=100 TK
*25% profit first selling price
=(100+100 of 25%)=125 TK
*20% less cost price
=(100-100 of 20%)=80 TK
*30% profit second selling price
=(100+100 of 30%)=130 TK
Now,
Cost price TK 100 then Sp=130 TK
»»»»»»»»»»TK1»»»»=(130/100)TK
»»»»»»»»»» TK80»»»=(130*80/130)
=104 TK
Differences of two selling price
=(125-104)=21 Tk
When
TK 21 less then cost price=100 TK
TK10.50»»»»»»»»»»»»»»»»
=(100*10.50)/21=50 TK
Answer:50 TK
(Alternative solution)
Solution-2:
Let,
Cost price be= x TK
First Selling price=(x+x of 25%)
=5x/4 TK
20% less cost price
=(100-20) or 80% of x=4x/5 TK
Second SP=(4x/5+4x/5 of 30%)
=26x/25 TK
According to the question,
(5x/4)-(26x/25)=10.50
=================
=================
Or, X=50
The cost price of the article=50 TK
Answer:50 TK
(Alternative solution)
Solution-3:
Let,
The cost price of the article is Tk100x
So, the selling price of the article is = Tk 100x + 25x = Tk 125x.
If he had bought it for(100x - 20x )= 80x, and sold it for (125x-10.50), the gain percentage
would have been 30%.
So,
or,80x × 0.30 = (125x-10.50) - 80x
Or,24x = 45x - 10.50
Or,21x = 10.50
Or,x = 10.50/21 = 0.50
So, the cost price of the article is = 100x = 100×0.5 = 50 tk
Answer:50 TK
##Preliminary Base Solution##
Sells-1=(100+25)=125 TK
Cost 20% less,
New cost=(100-20)=80 TK
Sells-2=80+80 of 30%=104 TK
Now,
Cost price=
(100*More or less)/(Sells1-Sells-2)
=(100*10.50)/(125-104)
=50 TK
Answer:50 TK
Question :a, b, c, d, e are 5 consecutive numbers in increasing order, deleting one of them
from the set decreased the sum of the remaining numbers by 20% of the sum of 5. Which one
of the number is deleted from the set?
[Bangladesh Bank AD-2014]
[Bangladesh Bank AD-2012]
[PBL SO-2013]
Solution-1:====
Since a,b,C,d,e are increasing order consecutive number
b=a+1
c=a+2
d=a+3
e=a+4
The sum of five numbers
=a+a+1+a+2+a+3+a+4
=5a +10
Now we are given that the sum decreased by 20% when one number was deleted
Hence,
The new sum should be
=(5a+10)-20% of( 5a+10)
=4a+8
Now,
New sum
=old sum- Dropped number
4a+8=5a+10-Dropped number
Dropped number=a+2=C
Answer:C

Solutions-2:====
Let, the consecutive numbers are,
a =1
b=1+1=2
c=1+2=3
d=1+3=4
e=1+4=5
So, Total = 1 + 2 + 3 + 4 + 5 = 15
Deleting one of the five numbers from the set then decreased 20% of the sum.
20% of the sum
=(15 x 20)/100
=3
So, the deleted number is the 3rd as c from the set
Answer :C

Solution-3:=====
Let,
C=x. b=x-1. a=x-2
d=x+1
e=x+2
Sum=x-2+x-1+x+x+1+x+2
=5x
Suppose,
The deleted number was=p
According to the question,
P=20% of 5x
P=x
Answer:C

Question :In a two digit number, the digit in the units place is more than twice the digit in
ten's place by 1. If the digits in the units place and the ten's place are interchanged, the
difference between the newly formed number and the original number is less than the original
number by 1, what is the original number?
[Janata Bank AEO-2015]
Solution:
Let,
Tenth’s digit = x,
Unit’s digit = 2x + 1,
So,
The number is =(10x + 2x + 1)
If the digit are interchange their place then alternate number
=10(2x + 1) + x
According to the question,
10(2x + 1)+x – {10x + (2x + 1)}=10x + (2x + 1) -1

Or, 20x + 10 + x - 10x - 2x - 1 = 12x

Or, 9x + 9 = 12x

Or, x = 3

So,
The Original number is
=(10*3 + 2*3 + 1) = 37

Answer :37

Question :A machine P can print 1 lakh books in 8 hour, Q can same in 10 hour & R can print
them in 12 hour. All the machine r started at 9 AM, while machine P in closed at 11 am and
the remaining two machine complete the work. Approximately at what time the work will be
finished?
[Janata Bank AEO-2015]

Solution:

(P + Q + R)'s
1 hour's work=(1/8 + 1/10 + 1/12) = 37/120
Work done by P, Q and R in 2 hours =(2*37/120) portion
= 37/60 portion
Remaining work
=(1 - 37/60) portion
= 23/60 portion
(Q + R)'s 1 hour's work
= (1/10 + 1/12) portion
= 11 /60 portion
Now, Q & R
11/60 part work is done in 1 hr.
1 part work is done by Q and R in
=1/(11/60)
= 60/11 hrs
So, 23/60 part work will be done by Q and R in
=(60 /11)*( 23/60)
= 23/11 hours
= (23*60)/11 mins
= 125 mins (approx.)
So, the work will be finished approximately 2 hours 5 mins. after 11 A.M., i.e., around 1.05
P.M. Answer:1 PM (Approximately)

Original Question:Joseph bought two varieties of rice,cost of 5 cent per ounce and 6 cent per
ounce each,and mixed them some ratio .Then he sold the mixture at 7 cent ounce,making a
profit of 20%.What was the ratio of the mixture.
[ Nove's GRE Math Bible]
Or
Modified Question:
Rahim bought two varieties of rice costing tk 5 & 6 per kg each.Then he sold the mixture at
Tk 7/kg,making profit of 20%. What was the ratio of the mixture?
[Bangladesh Bank AD-2015]
*Janata Bank EO -2012
*Rupali Bank SO-2013
*Bangladesh Bank AD-2014

Solution -1:
Let,
Rahim bought x kg rice at tk 5 & y kg rice at tk 6/kg
So, total cost =(5x + 6y) Tk
Selling price = (7x + 7y) Tk
20% profit = 20% of(5x + 6y)
= (5x + 6y)/5
According to question,
Selling price -Cost price =Profit
(7x + 7y) - (5x + 6y) = (5x + 6y)/5
Or, 2x + y = (5x + 6y)/5
Or, 10x + 5y = 5x + 6y
Or, 5x = y
Or, x/y = 1/5
Answer :1:5

Solution -2
Let,
Rahim bought x kg rice at tk 5 & y kg rice at tk 6/kg
So, total cost =(5x + 6y) Tk
Selling price = (7x + 7y) Tk
According to the question,
(7x+7y)=120% of (5x +6x)
======================
======================
Or, x/y=1/5
The ratio of the mixture=1:5
Answer:1:5

Solutions-3:
Preliminary Way
20% profit
Cost price=(100*7)/120
=70/12

5----------------------------------6

70/12

[6-70/12]-------------------[(70/12)-5]
=2-------------------------------10
=1:5
Answer:1:5

Question:In what ratio must a grocer mix two varieties of tea worthTk 60 a kg & 65 a kg so
that by selling the mixture at Tk 68.20 a kg he may gain 10% ?
[Islami Bank PO -2017] MCQ
More:
*GRE Big Book
*Exim Bank-2016
*Indian Bix
*Similar Bangladesh Bank
Solution:
Let
The grocer buy 60 Tk kg=x kg
The grocer buy 65 Tk kg =y kg
The cost price of (x+y) kg
=(60x+65y) Tk
And selling price of (x+y) kg
=68.20(x+y) Tk
10% profits selling price
=110%(60x+65y)
According to the question,
68.20(x+y)=110%(60x+65y)
=======================
=======================
========================
Or,22x =33y
X:Y=3:2
Answer:3:2
Alternative way====
According to the question,
Selling price-cost price=profit
68.20(X+Y)-(60X+65Y)=
10% of(60x+65y)
===========================
==========================
Or,X:Y=3:2
Answer:3:2

Short Cut:=====

10% profits sp=110 Tk


Cost price=(100*68.20)/110
=62 Tk

60----------------65

62

(65-62) (62-60)
=3. =2
Answer:3:2

Question :Salam used a part of TK 100,000 to purchase a TV of the remaining portion he


invested 1/3 of it at 4% simple annual interest and 2/3 of it at 6% simple annual interest, If
after a year the income from two investment total Tk 320. What was the purchase price of the
TV?
[One Bank SO-2017]]
Solution :
Let,
The purchase price of the TV =xTk
and
Remaining amount of money= (100,000 – x) Tk
First portion interest
=(100,000-x)*1/3 *4%
=(100000-x)/75
Second portion interest
=(100,000– x)*2/3 * 6%
=(100000-X)/25

According to the question


(100000-x)/75 +(100000-X)/25=320
Or,[(100000-X)(1/75+1/25)]=320
Or,[(100000-X)*4/75]=320
Or,400000-4x=320*75
Or, x = 94,000
Answer :94,000 Tk

Question :It takes 120 metric tone water to sink a ship.Through a hole in the full of the
ship,water is entering the ship at a rate of 2 metric tone per minute. At the same time,water is
being pumped out at the rate 1.5 metric tone per minute using one pump. After 1 hour and 20
minutes another pump of same capacity was started. How much more time will it take to
pump all the water out of the ship?
[BGFCL-2017]
Solution:
Here,
Water enter ship per minute
=2 metric tone
Water pumped out per minute
=1.5 metric tone
Water entered more per minute
=(2-1.5)=.5 metric tone
**After 1 hr & 20 minutes or 80 minutes
Water poured =(80*.5)=40 MT
After 80 minutes another capacity
pumped out pump started
So two pump pumped out water per minute=(1.5+1.5)=3 metric T
And 1 minute more water
pumped out=(3-2)=1 metric tone
Now,
1MT pumped out =1 minute
40 «»«»«»«»«»«»«=(40*1)/1
=40 minutes
Answer:40 minutes

Section-B: Math Review 1st Phase

===================================
1. The salary of Lamia and Farzin in the ratio of 7:5 and total of their salary is tk 12000. If
their annual increments are tk 200 and 150 respectively. What will be the ratio of their salary
after one year?

{Sonali Bank Officer(IT)-2016}


solution
________
Let,
Salary of Lamia=7x
Salary of Farzin=5x
According to the question,
7x+5x=12000
Or,x=1000
So, after one year salary of
Lamia=7*1000+200=7200
Farzin=5*1000+150=5150
Ratio=7200:5150=144:103
Ans:144:103
2. A man is standing on a railway bridge which is 180 meters long. He finds that a train
crosses the bridge in 20 seconds and crosses him in 8 seconds. Find the length of the train and
its speed.
{Sonali Bank Officer(IT)-2016}

solution-1:
Actually The Train
(20-8)=12 second goes 180 meters
So, The 20 sec goes=[(180*20)/12]
=300 meter
So,Length of train
=(300-180)=120 meter
And
Speed of train
=(120/8)m/s
=[(120/8)*18/5]km/hr
=54 km/hr
Answer:120 meter & 54 km/hr

Solution-2:=========
Let,
Length of the train =x
According to the question,
(180+x)/20=x/8
=>20x=144+8x
=>12x=1440
=>x=120
Solving equation x=120
Speed=[120*3600/8]
=54000 meter
=54 km
So length =120 meter
speed =54 km/hr
Answer:120 meter & 54 km/hr

3. A man goes upstream at 10 km hr to a place and back downstream to same point at 6


km/hr. What is his average sped his journey
{Sonali Bank Officer(IT)-2016}

Solution
________
Let, distance =30 km [LCM of 10 & 6]
Upstream time
=30/10
=3 hours
Downstream time
=30/6
=5 hours
Total distance
=30+30=60 km
Total time=5+3=8 hours
Average speed
=60/8=7.5 km/hr
Answer:7.5 km/hr
=========
4. The sum of the present age of father and son is 50 years, when son's age will be equal to
the father's present age then the sum of their age shall be 102, what is father's present age?
{Sonali Bank Officer(IT)-2016}

Solution-1:
Let,
Father present age=F years
Son's present age =S years
So,
F+S =50........(1)
When son age equal to father present age the sum of their age=102 years
Total age increased=(102-50)
=52 years
Individual age increased
=52/2=26 years
After 26 years later son's age equal to father age
According to the question,
S+26=F
or, F-S=26-----(2)
from Equations 1 & 2
Father =38 years
Son =12 years
Answer:38 years

Solution-2:
Let,
father's age=x
son's age=50-x
Difference of their age=x-(50-x) =2x-50
So after 2x-50 years son age will be equal to father's present age
According to the question,
x+x+2x-50=102
=>x=38
So father's age=38 years
son's age=50-38=12 years
Answer:38 years

5. By selling a table for tk 39 gain is as much percentage as its cost. what is the cost price?
{Sonali Bank Officer(IT)-2016}

Solution:
Let,
Cost price =x
x% gain selling price = x×x/100
According to the question,
x+ x*x/100=39
=> 100x+x^2 =3900
=> x^2+130x-30x-3900=0
=> (x+130)(x-30)=0
So, x=-130 [Not accepted]
or
x-30=0
So x=30
The cost price 30 Tk
Answer:30 Tk

=====================
6. A boat takes 3 hours to travel to a certain distance with the stream, and returns to the
original place in 5 hours. How long will it take to travel the same distance in stationary
water?
{Sonali Bank Senior Officer(IT)-2016}

Solution-1:
Let,
Speed of boat in stationary water=x
Speed of current=y
Downstream Speed=(x+y) km/hr
Upstream Speed=(x-y) km/h
We know,
Distance=speed*time
Downstream distance=3(x+y)
Upstream distance=5(x-y)
According to the question,
3(x+y)=5(x-y)
Or,3x+3y=5x-5y
Or,x=4y
Distance=3(x+y)
=3*5y
=15 y
Time taken to the same distance in stationary water
=15y/4y
=3 hrs 45 minutes
Answer:3 hrs 45 minutes

Solution-2:
Let, the speed of the boat = x kmph,
the speed of the stream = y kmph,
and the distance = d km.
According to the question:
x+y = d/3 ----------- (i)
&
x-y = d/5 ----------- (ii)
(i) + (ii) =>
2x = d/3 + d/5 = 8d/15
x = 4d/15
Required time = Distance/Speed = d/x= d/(4d/15) = 15/4 hrs = 3hrs and 45mins.
Ans: 3 hours and 45 minutes.

7. A man has Tk300000. He invests a part of the amount at 8%, and the remaining amount at
10% p.a. At the end of the year he earns a profit of Tk25600 from his entire investment. Find
the amount he invested under each rate.

{Sonali Bank Senior Officer(IT)-2016}

Solution:
Suppose,
He invested Tk x at 8% rate.
So, He invested Tk (300000-X) at 10% rate.
According to the question,
0.08X + 0.1(300000-X) = 25600
Or, 0.08X + 30000 - 0.1X= 25600
Or,0.02X = 4400
Or,X= 4400/0.02
Or, X= 220000
So, he invested Tk220000 at 8% rate,
and Tk(300000-220000) or Tk80000 at
10% rate.
Ans: Tk220000 at 8%, Tk80000 at 10%.

8. M purchased a 30-inch TV whose height was 18 inches. If the size of a TV is expressed as


the lengths of its diagonal, find the width of the TV M purchased.
-------------------------------------
{Sonali Bank Senior Officer(IT)-2016}

Solution:
Let,
The width of the TV = X inches.
According to the question:
X² + 18² = 30²
Or, X² = 900 - 324
Or, X² = 576
Or, X = 24
So, the width of the TV is 24 inches.
Ans: 24 inches.

9. The ratio of the girls and boys in a class was 2:5. If two new girls join the class, the ratio
becomes 1:2. What was the total number of students in the class?
{Sonali Bank Senior Officer(IT)-2016}

Solution:
Let,
the number of girls =2p, the number of boys=5p
And the total number of students in the class=2p+5p=7p
According to the question:
(2p+2) : (5p) = 1 : 2
Or, (2p+2)/(5p) = 1/2
Or, 5p = 4p + 4
Or, p = 4
So, the total number of students in the class was = 7×4 =28.
Ans: 28

10. How many liters of a solution that is 15% salt must be added to 5 liters of a solution that
is 8% salt so that the resulting solution is 10% salt?

{Sonali Bank Senior Officer(IT)-2016}

Solution:
Suppose,
X liters of 15% should be added.
According to the question:
15% of X + 5×0.08= 10% of (X+5)
Or, 0.15X + 0.4 = 0.10(X+5)
Or, 0.15X + 0.4 = 0.10X + 0.5
Or, 0.15X- 0.10X = 0.5 - 0.4
Or, 0.05X = 0.1
Or, 5X = 10
Or, X = 2
Ans: 2 liters.

Extra:
11.The time taken for covering a distance of x km in downstream is equal to the time taken to
cover (x-18) km in upstream. The speed of upstream is 6 km/hr less than the speed
downstream. If the speed of the boat in still water is 15km/hr, then find the value of x.

Solution:
The speed of the boat in fixed water = 15 kmph
Let, the speed of the water current = v kmph
Downstream Speed=(15+v) km/hr
Upstream Speed=(15-v)km/hr
So,
Then the difference between the upstream & downstream speed will be
= {(15+v)-(15-v)}
= 2v
Given,
2v= 6
=>v= 3 kmph
So, the downstream speed=18 kmph &
Upstream speed = 12 kmph
Now,
Time=Distance/Speed
According to the question,
(x-18)/12 = x/18
=> x= 54
So the value of x = 54
Answer:54 km
========================================
12.The sum of two numbers is 13 and their product is 30.What is the sum of the squares of
the two numbers?[Dhaka Bank cash-2016]
Solution:
Let,
Number are =x,y
Now,
x+y=13----(1)
xy=30------(2)
We know,
x^2+y^2=(x+y)^2-2xy
Or,x^2+y^2=(13)^2-2*30
Or, x^2+y^2=109
Answer:109

13. The simple interest on a sum of money will be tk 600 after 10 years. If the principal is
trebled after 5 years, what will be the total interest at the end of the tenth year?[Dhaka Bank -
2016]
[UCBL PO-2017]
Solution:
Here,
10 y. interest=600 Tk
1 yrs. interest=600/10=60 Tk
5 yrs interest=(60*5)=300 tk

When principal is trebled then interest will be also be trebled


=(60*3*5)
=900 tk( Last 5 five yrs)

Total interest in 10 years


=(300+900)
=1200 Tk
Answer:1200 Tk

14. 10% of the voters did not cast their vote in an election between two candidates. 10% of
the votes polled were found invalid. The successful candidate got 54% of the valid votes and
won by a majority of 1620 votes. Find the number of voters enrolled on the voters list.
[Bangladesh Bank IT-2016]
Solution-1:
Successful candidate gots=54%
Other candidate got=(100-54)=46%
Successful candidate got majority
=(54-46)%
=8%
According to the question,
8%---------------------=1620
100%-----------------=[(1620*100)/8]
=20250
Invalid vote=10%
So valid vote=(100-10)=90%
Now,
90% vote =20250
And 100% vote=[(20250*100)/90]
=22500
From question Did not cast vote=10%
Cast vote=(100-10)=90%
So,
90% cast vote=22500
100% or total voters
=[(22500*100)/90]
=25,000
Answer:25,000

15 .Two friends P and Q started a business investing in the ratio of 5: 6 R joined them after
six months investing an amount equal to that of Q 's .At the end of the year ,20% profit was
earned which was equal to tk 98000.What was the amount invested by R?
[Bangladesh Bank IT-2026]
Solution-1:
Suppose,
The total profit be z Tk

Then,
20% of z = 98000
Or, z =(98000∗100)/20
Or, z= 4,90,000
Total profit after one year =4,90,000 Tk
Let ,
the capitals of P, Q and R be
5x, Rs. 6x and Rs. 6x respectively.
Then,
5x*1+6x*1+6x * 1/2= 490000
Or, (10x +12x+6x)/2=490000
Or,28x=490000*2
Or, x=35000

So, R's investment=(6*35000)


=210000 Tk
Answer:210000 Tk

Solution-2:
Suppose,
The total profit be z Tk

Then,
20% of z= 98000
Z= (98000∗100)/20
Or,z= 4,90,000
Let ,
the capitals of P, Q and R be
5x, Rs. 6x and Rs. 6x respectively.
Then,
(5x * 12)+(6x * 12)+(6x * 6)= 490000 * 12
Or,168x = 490000 * 12
Or, x = (490000∗12168)
= 35000
R's investment = 6x =(6 * 35000) =210000 Tk

Answer:2,10,000 Tk

16. A person deposited money 6% & 7% respectively on fixed deposit. If he deposited a total
amount of Tk 4000 in the banks and earns 250 tk as interest in 1 year. How much money was
deposited in the bank with 7% interest rate?
[SBL TAO-2016]
1.
Let ,
7% interest he deposited=x Tk
And 6% interest he deposited=(4000-x) Tk
According to the question,
x*7%+(4000-x)6%=250
Or,7x/100+(24000-6x)/100=250
Or,x+24000=25000
Or,x =1000
So,he 6% interest deposited=1000 Tk
Answer:1000tk

17.A trader while selling an item,was asking for such a price that would enable him to offer a
20% discount and Still make a profit of 30% on cost price. If cost price is 50 tk, what was his
asking price?[SBL TAO-2016]
Solution:
Let ,
Cost price =100 Tk
20%discount selling price
=(100-20)=80 Tk
Given,
cost tk =50
30% profit on cost=(50+50*.30)
sp=tk. 65

when sp tk 80 asking price 100


when sp tk 65 " [(100*65)/80]
=tk. 81.25
Answer:81.25 Tk

18. A sales person receives daily wage of Tk 250 fixed and also gets 15% commission on all
sales he makes.How much take worth of sales does he need to make in order to bring his total
daily income of Tk 1000 ?[SBL TAO-2016]
Solution:
Sale's person got commission
=(1000-250) Tk
=Tk. 750
Let,
Total sales=x Tk
So,
X of 15%=750
Or, x=[(750*100)/15]
Or, x=5000 Tk
Answer:5000 Tk

19. The length of a rectangular field is 30 feet more than its breadth. If the perimeter of
rectangular field is 380 feet. What is the area of the field in square feet? [SBL TAO-2017]
Solution:
Let,
Breath = x & length = (x+30)
we know that,
perimeter of Rectangular
= 2( length + breath)
Or, 380 = 2( x+x+30)
Or, x = 80
Now,
Breath = 80 and
length = x+30 or 80+30, = 110
So,Area = (length*breath)
= (110*80)
= 8800 square feet

Answer:8800 square feet

20.There are two tapes in a water tank.The first tap pumps water in the tank and the second
one drains it out.The first tapes 30 minutes to make the full and the second tapes needs 40
minutes to drain that water out.If both taps are opened at the same time,how Long will it take
to make a half full tank?
[SBL TAO-2016]
Solution:
First pipe 1 minute can filled
=1/39 part
2nd pipe 1 minute can out
= 1/40 part
They together 1 minute can filled
=(1/30-1/40) part
=1/120 part

Now, 1/120 part filled in 1 minute


Then, 1/2 part """" """ =(120*1)/2
=60 minutes
Answer:60 minutes
========================================
21.এক ক ৭.৫% এক ক ১০% ক
এ ৩১ ক , ২০% ক ?[SBC
AM-2016]
Solution:
২০%
=(১০০+২০) =১২০ ক
এ ১০% ক
=(১০০-১০) = ৯০ ক

২০% ,
১০০ ক ১২০ ক
১ ক ১২০/১০০ ক
৯০ ক ১২০*৯০/১০০ = ১০৮ ক

৭.৫%
=(১০০-৭.৫) = ৯২.৫ ক
, এ ক
=(১০৮ -৯২.৫) = ১৫.৫ ক
খ ১৫.৫ ক খ = ১০০ ক
খ ১ ক খ = ১০০/১৫.৫ ক
খ ৩১ ক খ = (১০০*৩১/১৫.৫) = ২০০ ক

: ২০০ ক

22.এক ক ক এক
ক ৪ ৪৭৮০ ক এ ৭ ৫১৪০ ক ১২ ক
ক ?
[35th BCS Written]
Solution:
৪ ক = ৪৭৮০ ক
এ ৭ ক =৫১৪০ ক
, (৭-৪)=৩ =( ৫১৪০ - ৪৭৮০) = ৩৬০ ক
৩ = ৩৬০ ক
১ = ৩৬০/৩ = ১২০ ক
৪ গ
=[৪৭৮০-৪*১২০]
=৪৩০০ ক
, ১২ ক
=( + )
= (৪৩০০+১২*১২০) = ৫৭৪০ ক

: ৫৭৪০ ক
23. ১৪০০ ক ক এক ২০% ক ৮% ক
, ক ৮% ক ক ক ?
[EMBA DU-2017]
Solution:
Let,
Cost price of one thing is x
and another is (1400-x)
According to the question,
120x/100+(1400-x)92/100=1400
or, (120x+128800-92x)/100=1400
or, 28x=11200
or, x=400
So, 8% loss SP is
=(1400-400)*92/100
= Tk. 920

Ans: Tk.920

24.P এক ক এক ২০ এ Q ক এক ৩০ ক এক ক
ক ৫ গQ ক ক ক গ ?
Solution:
P ২০ ক ১ ক
P১ ক =১/২০
Q১ ক =১/৩০
P & Q এক এ ১ ক =১/১২
P ঑ Q এক Full ক ক ১২
৫ গQ
এক ৭ ক ক
১২ ক ১
এ এ ,৭ -------={৭/১২}
Remaining work ={১-৭/১২}
=৫/১২
*** ক ৫/১২ ক Pএ ক গ ক
P১ ক ২০
এ এ ,৫/১২-------{২০*৫/১২}
=৮.৩৩
঑ :৮.৩৩

25.Karim & Rahim have equal amount of money.Rumana has half of Rahim's money and
Amena has half of Rumana's money .If you add one Tk with all the money they have,it will
be Tk 100 .How much money Rahim had ?
[SBL TAO(Cash)-2016]
Solution:
Let,
The amount of Rahim be=x Tk
The amount of karim be=x Tk
Rumana has =x/2 Tk
Amena has =x/2*1/2=x/4
According to the question,
X+X+X/2+X/4+1=100
=================
=================
Or, X=36
Answer:36 Tk

26.On selling a chair at 7% loss and a table at 17% gain ,a man gain Tk 296.If he sells the
chair at 7% gain and the table at 12% gain,then he gains Tk 400 .The actual price of the table
& Chair individual and total is
[PBL MCQ -2016] [Modified Question]
Solution:
let,
Cost price of table=x tk
& cp of chair=y tk
According to the question,
17x-7y=296*100---------(I)
12x+7y=400*100----------(ii)
by adding equation no 1&2 we get,
x=2400.
now, putting the value of x in equation no ( 2) we get,
12*2400-7y=40000
Or,y =1600

Table price=2400tk
& chair price= 1600tk
Total price of table and chair
=(2400+1600)
=4000 Tk
Answer:2400;1600 & 4000 Tk

27.Mr X signed a contract for building a road for 1920 meters long within 120 days.He
employed 160 workers for this task.But after 24 days he found that only 1/8 of the task has
been finished .If Mr.X wants to finish the road in time how many additional workers he has
to employ ?[UCB PO-2017]
Solution:
Given that,
Total time to complete the task=120 days
Work already done=1/8 part
Now,
After 24 days Remaining time
=(120-24)=96 days
And
After 1/8 part work done,
Remaining task=(1-1/8)=7/8 part
24 days are needed to do 1/8 of the work by 160 workers
96 days are needed to do 7/8 of the work by
=[(160*24*8*7)/(1*96*8)]
=280 workers
Additional workers are needed
=(280-160)
=120
Answer:120

28.A book seller, while selling a book, was asking for prices that will enable him to offer a
25% discount and still make a profit of 20% on cost.If the adking price for the book was
tk.80, what eas the cost of the book?
Solution:
Let,
Cost price of the book=Tk. 100
At 20% profit on cost price then sp=120 Tk
And at 25% discount SP=(100-25)=75 Tk
Sp is 75 then asking price=100 Tk
SP is 120 ----------------=(120*10)/75
=160 Tk
When AP is 160 then Cp =100 Tk
So, "" 80. """" ""=(100*80)/160
=Tk. 50

So,the cost price of the book is 50 Tk


Answer:50 Tk

29.A can do a piece of work in 8 days which B can finish in 12 days. If they work at it on
alternate days with A beginning, In how many days, the work will be finished?
Solution:
(A+B)'s 2days work =(1/8+1/12)
=5/24
So (A+B)'s 8days work =(5*4/24)
=20/24
work left= (1-20/24)=4/24 =1/6..
at 9th day, A's 1days work = 1/8
work left =(1/6-1/8) =1/24

B can do 1/12 portion of work in= 1day


so 1/24 portion of work can be done in =12/24 days
=1/2 day..
so work will be finished in (8+1+1/2) = 9.5 days
Answer:9.5 days

30.When a producer allows 36% commission on the price of his product, earns a profit of
8.8% what would be his profit percent if the commission is reduced by 24%?[PBL SO/O-
2014]
Solution:
Let,
cost price of the product=Tk.100
8.8% profits on cost price then
Selling price of product
=(100+100 of 8.8%)=108.8 Tk
36% commission on cost price then Selling=(100-100 of 36%)=64 Tk
When
SP is 108.8 Tk then cp=100 Tk
SP-----64----------------=(100*64)/108.8
=58.82 Tk
If the commission is reduced by 24% then actual commission gave
=(36-34)=12%
12% commission on cost price then Selling price=(100-12)=88 Tk
Profit=(88-58.82)=29.18 Tk
Now,
Profit percent he got
=[(29.18*100)/58.82]=49.6%
Answer:49.6%

31.Before the budget, a businessmen had increased the price of his products by 12%.In the
budget,a 10% sales tax was imposed on that product.As a result,the profit ultimately
increased by Tk 64 .what was the original price of the product?
[EXIM BANK -2015]
Solution:
Let ,
The original price of the product is 100
at 12% increase price of the product before budget
=(100+100 of 12%)
=112 Tk
after deducting 10% sales tax price of the product
=(112- 112*10%)
=100.8 Tk
so profit (100.8-100)=.8
now original price of the product
=(64*100)/.8
=8000 Tk
Answer:8000 Tk

32.A room is half as long again as it is broad. The cost of carpeting the room at Tk. 5 per
sq.m is Tk.270 and the cost of papering the four walls at Tk.10 per sq.m is Tk.1720. If a door
and 2 windows occupy 8 sq.m.Find the dimensions of the room.
[PBL SO/OFFICER-2016]
Solution:
Let,
Breadth of room= x
Length of room= (x+x/2) = 3x/2
Area of floor
= Total carpet cost/ rate
= 270/5
= 54 square meter
According to the question,
3x*x/2 = 54
x=6
From the equation we find,
Breadth = 6m
Length= 3*6/2 = 9m
Now,
Papered area,
= (1720/10) sq. km
= 172 sqm
So,
Area of one door and two window = 8 sq.m
Total area of 4 walls
= (172+8)
= 180 sq.m.
Now,
Area of four wall
=[2(length+breadth)]*height
2(6+9)*h = 180
30h = 180
h=6
So,
Breadth= 6m
Length= 9m
Height = 6m
Answer: 6,9 & 6 meter

33.A man deposited Tk. 50000 at a certain interest for 1 year. After 1 year he received
Tk.55280 as both principal & interest after deduction of Tk.120 as government levy and 10%
on interest as government tax. what was the interest rate in percentage?
Solution:
Here,
Deposited amount=50000 Tk
Interest after deducting Tax and Levy
( 55280-50000)
= Tk.5280
Interest Before deducting levy
=5280+120
= Tk.5400
10%interest as government tax
Now,
(100-10)=90% =5400;
then 100% ≡ 5400*100/90
= Tk.6000
That is, Interest before deducting Tax and Levy = Tk.6000

So, Interest Rate


= (6000/50000)*100
= 12%
Answer:12%

34.The annual income and expenditure of a man and his wife are in the ratios of 5:3 and 3:1
respectively. If they decide to save equally and find a balance of tk 4000 at the end of year,
what was their income?[BB OFFICER-2015]
[এ ক ঑ confusion ঑ ১০০০০ &
৬০০০ ক Exam cpoy pest ক ক ঑
ক ক গ ক ক If they decide to save
equally and find a Total balance of tk 4000 at the end of year, what was their income?
঑ ৫০০০ & ৩০০০ ক
ক ঑ এক ]
Solution:
[ ক ৪০০০, ক ক ৪০০০
ক ক ৪০০০ ক ক , Save
঑ ক
ক ক language lackings , ঑এ ক এ গ
]
Suppose,
Man's income = 5x
Man's expenditure = 3x
Savings = 5x-3x = 2x
Again,
Woman's income = 3y
Woman's expenditure = y
Savings = 3y-y = 2y
According to the question ,They save equal balance
So,
2x=2y=4000
2x=4000
Or,x=2000
Again,
2y=4000
Or, y=2000

Man's income = 5*2000=10000 Tk


Woman's income=3*2000= 6000Tk
Answer: 10,000 & 6000 Tk

Back solve:
Total man income=10000 Tk
Save=4000 Tk
Expenditure=(10,000-4000)=6000 Tk
Man Income & Expenditures Ratio
=10000:6000
=5:3
Women Expenditure
=(6000-4000)
=2000 Tk
Women Ratio
=6000:2000
=3:1
*** খ ৪০০০**
Man's income = 5x
Man's expenditure = 3x
Savings = 5x-3x = 2x
Again,
Woman's income = 3y
Woman's expenditure = y
Savings = 3y-y = 2y
According to the question ,They save equal balance
So,
2x=2y=4000/2=2000
2x=2000
Or,x=1000
Again,
2y=2000
Or, y=1000

Man's income = 5*1000=50000 Tk


Woman's income=3*3000= 3000Tk
Answer: 5000 & 3000 Tk

35.Abul & Balam ran a race of 480 m. In the first heat, Abul gives Balam a head start of 48 m
and beats him by 1/10th if a minute. In the second heat, Abul gives Balam a head start of 144
m and is beaten by 1/30th of a minute. What is Balam's speed in m/s?
[DBBL PO-2016]
Solution:
In first heat, Balam gets a head start of 48 m
Thus the distance travelled by Balam in first heat
= (480-48)
= 432m
1/10th of a minutes= 6 seconds beat by him
In 2nd heat Balam travelled
= (480-144)
=336m
1/30th of a minutes= 2 seconds
Let,
Balam's speed= S m/s
According to the question,
(432/S)-6= (336/S)+2
=> 432/S-336/S= 6+2
=> 432-336/S= 8
=>96= 8S
=>S= 96/8
so, S= 12m/s
Answer:12 m/s

36. A 20 liter mixture of milk and water contains milk and water in the ratio 3:2. 10 liters of
the mixture is removed and replaced with pure milk and the operation is repeated once more.
At the end of the two removal and replacement, what is the ratio of milk and water in the
resultant mixture?
[DBBL PO-2016]
Solution:
Step-1:
Ratio of milk & water=3:2
Milk in the mixture= 20 x 3/5 = 12 ltr
Water in the mixture= 20 x 2/5 = 8 ltr
Ratio = 3 : 2

Step-2:
After removal of10 liters mixture,the amount of milk will be = 10 x 3/5 = 6 ltr
water = 10 x 2/5 = 4 ltr.

After pouring 10 ltr. Of pure milk,


New mixture
Milk = 12 - 6 + 10 = 16 ltr.
Water = 8 - 4 + 0 = 4 ltr
Ratio = 16 : 4 = 4 : 1.
Step-3
[ 10 liters replaced ]
milk = 10 x 4/5 = 8 ltr
water = 10 x 1/5 = 2 ltr
Now addition of pure milk 10 ltr.

Milk = 8 + 10 = 18 ltr.
Water = 2 ltr.
Ratio = 18 : 2 = 9 : 1

Ans: 9 : 1

37.During a special promotion a certain filling station is offering 10% discount on gas
purchased after 10 gallons.If kim purchased 20 gallons of gas and Zeny purchased 25 gallons
of gas the zenis total per gallon discount is what percent of kims total per gallon discount?
Solution:
Kim paid
=(10+10-10 of 10%)
=19 gallon,
Discount=20-19=1
So,the overall discount she got was =(1*100)/20=5%

Zeny's purchased 25 gallons of gas.


She paid for (10 + 0.9*15) = 23.5
gallons,
Discount=(25-23.5)=1.5 gallon
so the overall discount she got was 1.5*100/25 = 6%.
From question
P% of Kim=6
(P/100)*5=6
P=120%
Answer:120%
38.The simple interest on a certain sum of money for 2.5 years at 12% per annum is Tk. 40
less than the simple interest on the same sum for 3.5 years at 10% per annum. Find the sum.
[PBL CASH-2016]
Solution:
Suppose,
Sum be = x Tk
According to the question,
(3.5*10%)*x - (2.5*12%)*x = 40
Or, 35x-30x = 4000
Or,5x = 4000
Or,x = 800
Answer:800 Tk

Solution-2:by (Sarker Mamun)


Effective rate of interest for 3.5 years=3.5*10=35%
Effective rate of interest for 2.5 years=2.5*12=30%
Difference of interest rate=35-30=5%

Now, 5% is..........................40
Then, 100% is........40*100/5
=800 (Ans.)

39.A, B and C started a business by investing Tk 1,20,000, Tk. 1,35,000 and Tk. 1,50,000
respectively. Find the share of each out of an annual profit of Tk. 56,700[PBL CASH-2016]
Solution:
Ratio of their investment:
A,B & C=120000:135000:150000
=120: 135: 150
=8: 9: 10
Sum of their Ratio= 8+9+10=27
Share of profit;
A=(56700*8/27) = 16800 Tk
B=(56700*9/27) = 18900 Tk
C=(56700*10/27)=21000 Tk
Answer: 16800, 18900 & 21000 Tk

40. A retailer buys 40 pens at the market price of 36 pens from a wholesaler. If he sells these
pens giving a discount of 1%, what is the profit percent?
[PBL CASH-2016]
Solution:
Let,
40 of pens cost price=36 SP =x Tk
cp of each pen =x/40 Tk
sp of each pen=x/36 Tk

Deducting 1% discount, sp
= 99%*x/36
=99x/3600
profit =99x/3600 - x/40
=9x/3600

Profit percentage
=[(9x/3600)/(x/40)]*100
=10%
Answer:10%
Alternative Solution: by (Sarker Mamun)
Let, Market price of each pen be Tk.100,
So, market price of 36 pens
=Tk. 3600
Then, Buying price of 40 pens =Tk.3600
So, Buying price of each pen
= 3600/40
=Tk.90
At 1% discount, SP =99% of 100
=Tk. 99
So, Profit =99 –90
=Tk. 9
Profit %=9*100/90
=10%(Ans.)

41.In 2016, the number of pairs of the shoes that a company sold to retailers decreased by 20
percent, while the price per pair increased by 20 percent from that of the previous year. The
company's revenue from sales of the shoes in 2016 was taka 300000. What was the revenue
from the sale of the shoes in previous year?
[JBL AEO (IT)-2016]
Solution:
Let,
In 2015 The company sold 100 pair of shoes
and
Price was Tk. 100 per pair.
So,Total Revenue 2015 was
=(100×100)
= Tk. 10000 Tk.
So, Revenue in 2016 was
Tk. 120×80= Tk. 9600.
Thus, actual revenue in 2015 was Tk. (3,00,000×10000)/9600
= Tk. 3,12,500
Answer:3,12,500 Tk
Alternative Solution:
Let,
Previous year,
Shoes sold = x numbers
and
price per shoes=y Tk
Total revenue= xy Tk.
In 2016,
Shoes sold = (x- 0.20x) numbers and price per shoes=(y+0.20y) Tk.
According to the question,
(x- 0.20x)× (y+0.20y)= 300000
Or, 0.80x ×1.20y=300000
Or,xy = 312500
Total revenue= 3,12,500 Tk.
Answer:3,12,5000 Tk

42.A manufacturing company uses two machines A and B with different production
capacities. When working alone, machine A can produce a production lot in 5 hours and
machine B can produce the same lot in X hour. When the two machines operate
simultaneously to fill the same production lot, it takes them 2 hours to complete the job. How
many hours will the machine B take to produce the production lot alone?[JBL AEO (IT)-
2016]
Solution:
In x hours B can complete 1 part
In 1 hour B can complete 1/x part.
In 5 hours A can complete 1 part,
In 1 hour A can complete 1/5 part.
If they both worked together they can do in 1 hour=1/2 part
According to the question,
1/2-1/5=1/x
Or, (5-2)/10=1/x
Or, 3/10=1/x
Or, 3x=10
Or, x = 10/3
= 3.33 hours or 3 hours 20 min
Answer:3.33 hrs

43.If the sum of Interests on 500 Tk. for 4 years and 600 Tk. for 5 years is 500 Tk.Then What
will be the interest on Tk. 1000 for 3 years?[JBL AEO (IT)-2016]
Solution:
Let,
Rate of interest of is x%
On Tk 500 in 4 years interest
={(500*4*x)/100}
=20x Tk
On Tk 600 in 5 years interest
={(600*5*x)/100}
=30x Tk
According to the question,
20x+30x=500
Or, x=10
So rate of interest is 10%
Now on Tk 1000 in 3 years interest
={ (1000*3*10)/100}
=300 Tk
Answer:300 Tk

44.Sumon and Shimul two friends started a business with 5000 Tk and 4000 Tk respectively
in a business. After 3 months Mr. Sumon added 1000 Tk more and Dilip invested with 7000
Tk in the same business. What is the share of profit among them after one year if profit is
36000 Tk?[JBL AEO (IT)-2016]

Solution:
Equivalent amount of Sumon
= 5000×12+ 1000× 9= 69,000 Tk.
Equivalent amount of Shimul
= 4000×12= 48, 000 Tk.
Equivalent amount of Dilip= 7000×9= 63,000 Tk.
Sumon: Shimul : Dilip
= 69,000: 48,000 : 63,000
= 69: 48: 63
= 23: 16 : 21
Sum of the Ratios
= 23+ 16 + 21= 60
Profit share of,
Sumon= (36000× 23/60) Tk.
= 13800 Tk.
Shimul= (36000×16/60) Tk.
= 9600 Tk.
Dilip= (36000×21/60) Tk.
= 12600 Tk.
Ans: 13800 Tk, 9600 Tk and 12600 Tk

45.16 men can finish a piece of work in 49 days.14 men started working in 8 days they could
finish certain amount of work. If it is required to finish the remaining work in 24 days..How
many more men should be added to the existing workforce?
{Important Math}
Solution:
Let,
Total work=1 portion
16 men can do in 49 days 1 portion
16 men can do in 1 day 1/49 part
So,
1men can do in 1 day = (1/49)*16
14m can do in 8 days= [(14*8)/(49*16)]= 1/7 portion

Work Remaining
= (1 - 1/7)
= 6/7 portion

1/7 part of the work done in 8 days by 14 men


So, 6/7 part of the work done in 24 days by
= [(14*7*8*6)/7*24 ]
= 28 men

More men needed


= 28-14 = 14 men
Answer:14 m
46.16 ounces of fresh orange juice has 240 calories & 16 ounces of fresh grapefruit juice has
174 calories.If an 8 ounce mixture of these 2 juices contains 94 calories, what fraction of the
mixture is orange juice?
{Important Math}
Solution:
Let,
The mixture contain x ounces of orange juice and
( 8-x) ounces of grape juice
According to the question,
x* (240/16) + 174/16 (8-x) = 94
Or,240x + 1392 - 174x = 1504
Or,66x = 112
Or,x = 112/66
Or, x= 1.6969=1.7 ounces

Required Fraction = 1.7/8 = 0.2125


Answer:0.2125

47.A ship has faulty engine sailed only 7 hours period 2 day that covers 59 km. if first day
rate is 5 km less than second day but first day sailed 3 hours more than of second day. what's
is the avg speed of second day?
[DBBL PO-2017]
Solution:
Let,
First day rate=x km/h
2nd day rate=(x+5)km/h
2nd time taken=y hrs
So,First day time taken=(y+3)hrs
From question First condition
y+(y+3)=7
Or, y=2
Second day time taken 2 hrs
First day taken=5 hrs
According to the question,
X*5+2*(X+5)=59
{Distance=Speed*Time}
Or,x=7
Second speed=12 km/hr
Answer:12 km/hr

48.In a class 25 students, 10 have less than 6 marbles, 10 have more than 7 and 4 have more
than 8. how many have more than 5 less than 9 marbles?[DBBL PO-2017]
Solution:
Here,
The Number of Students has,
Less than 6 marbles=10
More than 7 marbles=10
More than 8 marbles=4
Let,
The Number of Students has more than 5 less than 9 marbles=x
Required condition= 5<x<9
So, x=25-10-4=11
Ans: 11 Students
NOTE:
Required condition... 5<x<9
Now 10 have less than 6, that means 10 have at least 5..(not fulfill condition) and 4 have
more than 8 that means 4 have at least 9..(not fulfill condition)
So student x = 25-(10+4)=11

49.Bluepaint and yellow paint ratio 1:2 cot 110 and 100per ltr. mixture of two paint made
green color sell 120 per ltr. if he get 2000 tk profit how much yellow paint kg use.?
Solution:[DBBL PO-2017]
Let,
Blue paint= x kg
Yellow paint=2x kg
Cost price of Blue & Yellow paint
=110*x+100*2x
=310 x
Selling price of Green paint
=3x*120
=360x
We know that
Selling price-cost price=profits
According to the question,
360x-310x=2000
On solving, we get x=40
So, Yellow paint=2x =2*40=80
Ans: 80 Kg

50.A man has 100 balls, 50 red and 50 black sell 48 each. if black sell 20% loss and red sell
20% profit. What is the net profit or loss in tk?
Solution:[DBBL PO-2017]
Selling price of Red balls=50*48=2400
Selling price of Black balls=50*48=2400
Total selling price=4800tk
CP of Black balls=(2400*100)/80
=3000 Tk
CP of Red balls=(2400*100)/120
=2000 Tk
Total cost price=5000 Tk
Net Loss=(5000-4800)= 200 Tk

Ans:200 tk.

50.A man has 100 balls, 50 red and 50 black sell 48 each. if black sell 20% loss and red sell
20%
profit. What is the net profit or loss in tk ? [DBBL PO-2017].
Solution :
Selling price of Red balls=50*48=2400
Selling price of Black balls=50*48=2400
Total selling price=4800tk
CP of Black balls=(2400*100)/80
=3000 Tk
CP of Red balls=(2400*100)/120
=2000 Tk
30
Total cost price=5000 Tk
Net Loss=(5000-4800)= 200 Tk
Ans:200 tk.
51.Mr Brown has a wheat business. He purchases wheat from a local wholesaler at a
particular per pound. The price of the wheat at her local is tk.30 per kg. His faulty spring
balance reads.0.90 per kg for a kg . Also in the festival season he gives 10% discount on the
wheat. He found that he made neither a profit nor a loss in the festival season. At what
price did Mr Brown purchase the wheat from the wholesaler? [Jamuna Bank MTO-2016].
Solution:
Here
Market price=30 Tk per kg
10% discount after price per kg
=(30-30 of 10%)
=27 Tk
But he sells .9 kg in stead of 1 kg
So,SP =(.9*27)
=24.3 Tk per kg
Which should be her cost price
We know,
2.2 pound =1 kg
Cost price per pound
=24.3/2.2
=Tk 11.045 per pound
Answer: Tk11.045 per pound
52. According to the stock policy of a company each employee in the division is given 15
shares of the company and each employee in the recruitment division is given 10 shares.
Employees belonging to both the communities get 25 shares each.There are 20 employees
in the company and each of them belongs to at least one of the division.The cost of each
share is tk.10.If the technical division has 15 employees and the recruitment division has 10
employees, then what is the total cost of the shares given by the company?
[Jamuna Bank MTO-2016]
Solution-1:
Given that,
The number of employees in the technical & recruitment division be 15 & 10 respectively
Agian
Each technical person & each recruitment person given 15 & 10 share respectively
Hence the net share given equals
=(15*15)+(10*10)
31
=325
So the net worth of the share
=(325*10)
=3250 Tk
Answer:3250 Tk
Solution-2:
Shares :
Technical -15
Recruiting -10
Both-25
No. of employees :
Technical -15
Recruitment -10
Total-20
Both=25-20=5
so, employees only in Technical =15-5=10
Employees in recruiting =10-5=5
Per share price: 10 Tk
Cost of both-25*5*10=1250
cost of recruiting -10*5*10=500
cost of technical -15*10*10=1500
Totals=1250+500+1500=3250
Ans: Tk.3250
Solution-3:
Let,
Employee of both is x
ATQ,
15+10-x=20
on solving, we get x = 5
only technical division employee= 15-5=10
only recruitment division employee= 10-5=5
Total price of shares = (10*15+ 5*10+5*25)10= 3250
Ans: Tk.3250

53.X sells an item to Y at a profit of 28% on his cost and Y sells the same item to Z at a loss
of 25% on his cost. If Y has thus sold the item to Z at Tk 2 less than the cost of the item to X,
what is the cost of the item to X in Tk ? (NCC Bank -2002)
Solution:
Cost price of X is 100 Tk
Selling price of X=100*128/100=tk.128
Cost price of Y is 128 Tk
Selling price of Y tk.
=128*75/100
= Tk.96
Cost price of Z tk.96
Difference=100-96= 4 tk less.
Now,
4 tk less when Cost price tk. 100
1 tk less when Cost price tk. 100/4
2 tk less when Cost price tk. 100*2/4= 50 tk
Answer:50 Tk
Ans: 50 tk.
54.A can do a piece of work in 10 days, while B can do it in 15 days.They work together for
5days and rest of the work done by C in 2 days.If they get tk 4500 for the whole work, how
should they divide the money? (RAKUB officer-2016)
Solution:
A can do in 1 day = 1/10 part work
A can do in 5 days =5/10= ½ part work
B can do in 1 day=1/15 part work
B can do in 5 days=5/15=1/3 part work
In 5 days (A+B) work
= (1/2+1/3)
= 5/6 part work
So, in 2 days C does work
= 1 –5/6
= 1/6 part work
Now,
Share of A =4500×(1/2)=2250 Tk
Share of B = 4500×(1/3)=1500 Tk
Share of C = 4500×(1/6)=750 Tk
Answer:2250 Tk,1500 Tk & 750 Tk

55. Arafat and Asif were each paid X taka in advance to do a certain job together. Arafat
worked on the job for 10 hours and Asif worked 2 hours less than Arafat. If Asif gave Arafat
33 Y taka of his payment so that they would have received the same hourly wage, what was
the amount in taka, in terms of Y, that Arafat was paid in advance? (Exim Bank-2016).
Solution:
Work ratio=1/10:1/8=4:5
Asif's wage=4x/9 tk
Arafat's wage=5x/9 tk
According to the question,
5x/9-4x/9=Y
x=9Y
Answer:9y Tk
56. A man started at 8 A.M from his home, walked at the rate of 3km/hr & reached his
office 45minutes late .The next day he started at the same point & walked at the rate of
5km/hr & reached his office 15 minutes earlier. What is the distance between his office &
home?
Solution:
Let,
The distance between his office & home be x km
According to the question,
x/3 -(45/60) =x/5 +(15/60 )[Time=distance/Speed]
Or, x/3 - x/5 = 15/60 + 45/60
x= 7.5
Ans: 7.5 km
57.If sales tax is payable at t% of gross sales value and you intend to make a profit of r%,
what would be the gross sales price inclusive of tax of an article which you bought for tk K?
(Basic Bank-2008).
Solution:
Given that,
Cost price of article=k Tk
Sales price r% profit(Excluding tax)
=(k + k of r%)
=k{(100+r)/100}
Now,
Adding t% tax on sales ,the tax is
=k{(100+r)/100}*t%
So,
Gross sales price inclusive of tax is
=k{(100+r)/100} +k{(100+r)/100}*t%
=k{(100+r)/100} *{(100+t)/100}
34
Answer:k{(100+r)/100} *{(100+t)/100}
Note:
Firstly
58. A train travelling at 48 kmph completely crosses another train having half its length
and travelling in opposite direction at 42 kmph, in 12 seconds. It also passes a railway
platform in 45 seconds. The length of the platform is ----
Solution:
Let,
The biggest train length=2x
The smallest train length=x
Since two train are opposite direction
So,Relative speed=(48+42)
=90kmph
=90*(5/18)
=25 m/s
We know,
12*25=(x+2x)
X=100
Biggest train length
=2*100=200 m
Again
Let,
Platform length=M
48kmph
=48*(5/18)m/s
=40/3 m/s
We know,
Time*speed=Distance
45*(40/3)=200+M
M=400 m
Answer:400 m
35
59. A task A and B work together in 10 days , B and C work together in 15 days ,C and A
work
together in 25 days. They work together for 4 days .Then A left the job and B and C work
for 4 days. Then B left the job. How many days C will finish the task? (KSB -2011).
Solution:
(A+B+C) in 1 day's work
=(1/10+1/15+1/25) portion
=(31/150*2) portion
=31/300 portion
Now (A+B+C)'S 4 day's work
=(31*4/300)
=31/75 Portion
Remaining work
=(1-31/75)
=44/75 portion
Then A left.
(B+C) 4 days work =4/15
Remaining work
=(44/75-4/15)
=24/75 portion
C's 1 days work
=31/300-1/10
=1/300 portion
C 1/300 part do in 1 day
24/75 " ".........(24*300/75)
=96 days
Answer:96 days

60. Tk.1500 is invested at a rate of 10% simple interest and interest is added to the principal
after every 5 years. in how many years will it amount to tk.2500 ?
Solution:
Here,
P=1500 Tk
T=5 years
R=10%
For first 5 year interest
=(1500*10*5)
=750
So principal after 5 year
=(1500+750)= 2250.
Now interest
=(2500-2250)
36
= 250 Tk
Again
I=250 Tk
P=2250 Tk
R=10%
Then,
T= (250*100)/(2250*10)
= 10/9 years
So total year= 5+10/9
= 6 years 1 month 10 days or 6.11 years taken to receive 2500 Tk
Answer: 6.11 year

61. In a two- digit number, the digit in the unit's place is more than twice the digit in the
ten's place by 1. If the digit in the unit's place and the ten' s place are interchanged,
differences between the newly formed number and original number is less than the
original number by 1. What is the original number ? [Janata Bank AEO-2015]
Solution:
Let,
Ten’s digit = x,
Unit’s digit = 2x+1
The number is
= 10x+(2x+1)
=12x+1
If the digit in the Unit place & tenths place are interchange
Then the number
=10(2x+1)+x
=20 x +10+x
=21x +10
According to the question,
(21x+10)–(12x+1)=(12x+1)-1
we get x = 3
So, the original number is
= 10×3+2×3+1
=37
Ans: 37

62. Mr John invested certain amount in three different schemes, A ,B and C with the rate of
interest 10% p.a, 12% p.a and 15% p.a respectively. if the total interest accrued in one year
was tk.3200 and the amount invested in scheme C was 150% of the amount invested in
37 scheme A and 240% of the amount invested in scheme B, what was the amount invested in
scheme B ? [Bangladesh Bank (cash)-2017]
Solution:
Let,
B investment be=x
ATO,
C=240% of B
Or, C =12x/5
And,
C =150% of A
A=8x/5
According to the question,
10% of (8x/5)+12% of x +15% of (12x/5)=3200
Or,(16x+12x+36x)/100=3200
Or, x =5000
Ans: Tk.5000
63. A, B & C can complete a work separately in 24, 36 & 48 days respectively. They started
together but C left after 4 days of start and A left 3 days before the completion of the
work. In how many days will the work be completed? {Very Important Math}.
Solution:
Let,
'x' day after work will be completed
Total work=1 portion
According to the question,
(x-3)/24+x/36+(4/48)=1
=>x=15
Ans:15 days

64. By walking at 3/4 of his usual speed, a man reaches his office 20 minutes later than his
usual time. Find the usual time taken by him to reach his office. [PBL -2013]
Solution:
Let,
His usual speed=x
Usual time=y
So,
Distance=xy---------:(1)
38
Now,
Present speed=3x/4
Present time=(y+20)
Distance=3x/4*(y+20)-----(1)
From (1) & (2) equations
xy=3x/4*(y+20)
y=60
Ans: 60min or 1hour.
65. By selling 100 bananas, a fruit-seller gains the selling price of 20 bananas. His gain
percentage is ? {Very Important Math}.
Solution:
Let,
selling price of a banana is 1 tk.
so, selling price of hundred banana is 100tk.
gain =20 tk
cost =100-20=>80tk
cost price 80 taka than profit 20
cost price 100taka than profit 20/80*100
=25%
Ans:25%
Alternative:
We know,
Gain = (S.P. of 100 bananas) - (C.P. of 100 bananas)
⇒ (S.P.of 20) = (S.P of 100) - (C.P. of 100)
⇒ S.P.of 80 = C.P. of 100
Let,
C.P. of each banana = Tk. 1
C.P. of 80 banana = Tk. 80
S.P. of 80 banana = Tk. 100
Gain=100-80=Tk.20
Required percentage = (20 / 80) x 100 % = 25%
Ans:25%

66. A train leaves sylhet at 6 a.m. & reaches Dhaka at 10 a.m. Another train leaves Dhaka at
8 am and reaches Sylhet at 11:30 a.m. At what time do the two trains cross one another
{Very Important Math}
Solution:
Let,
39
Distance between Sylhet to Dhaka x km
First train time taken=4 hrs
Second train time taken=3.5 hrs
first train speed = x/4 kmph
2nd train speed = x/3.5 kmph
suppose they meet 'y' hour after 8 am
so,
x(y+2)/4 +xy/3.5=x
Or, Y =56 minutes
so they meet 8.56 AM
Answer:8.56 AM

67. Three pipes A, B, C are attached to a tank. A and B can fill it in 20 and 30 minutes
respectively and C can empty it in 15 minutes. If A, B, C are kept open successively for 1
min.
each, how soon will the tank be filled?{Very Important Math}.
Solution:
LCM of 20,30,60 is =60
Let,
Total work=60 unit
1 min A pipe done=60/20=3 unit
1 min B pipe done=60/30=2 unit
1 min C pipe done=60/15=4 unit
So, 3 min A , B & C done
=3+2-4
=1 unit
So,
165 min A,B & C done
=(165*1)/3
=55 unit
166 min A turn on and done =3 unit
Total work=55+3=58 unit
167 min B turn on and done=2 unit
Total work=58+2=60 unit
So the tank wil be filled in 167 minutes
Answer:167 minutes

68. A dealer gives two successive discounts of 10% and 20% on the marked price of 5000 of
a Smartphone, he had to give a further discount equal to 20% of his cost price on his new
selling price, as a result he made neither a profit nor a loss. Find the cost price of the
mobile? {Very Important Math}
Solution:
Let,
40
Cost price= x tk
after 10% discount S.P
= 5000 - 10% of 5000
= 4500 tk
Again
After 20% discount S.P = 4500 - 20% of 4500
= 3600 tk
According to the question,
x = 3600 - 20% of x
x = 3600 - x/5
x = (5 * 3600)/6
Or, x = 3000 tk
Answer:3000 Tk

69. Mr John Wick is Rice businessmen. He two varieties of rice are mixed in a certain ratio.
The cost of the mixture per kg is Tk 0.50 less than that of the superior variety and Tk 0. 75
more than that of the inferior variety. What was the ratio of superior variety to inferior
variety in the mixture ? [BBA 2004-2005].
Solution:
Let,
The quantity of superior=x kg
And
The quantity of inferior=y kg
Again
Let,
The price per kg of superior variety=p Tk
The price per kg of mixed variety
=(p-0.5) Tk
The price per kg of inferior variety
=(P-0.5-0.75) Tk
According to the question,
(X*P)+Y(P-0.5-0.75)=(X+Y)*(P-0.5)
Or, X:Y=3:2
Answer:3:2

70. A hall is 15 m long & 12 m broad. If the sum of the areas of the floor & the ceiling is
equal
to the sum of the areas of the 4 walls, the volume of the hall is?
Solution :
We know that,
Floor & ceiling are equal length & breadth
41
So total Area of floor and ceiling
= (15*12)+(15*12)
=2*15*12
Area of four Wall = 2(15+12)* h
According to the question,
2*15*12 = 2(15+12)*h
=> h = 20/3
Volume= 15*12*20/3 = 1200

Answer:1200

Section-C: Math Review 2nd Phase


1: A man deposits 5000 tk. at 5% annual interest for six months. In every six months he
withdraws tk.500 from his principal plus interest earned. What is the total amount of interest
he received?
[BKB SO -2017][SEBL PO-2017]
Solution:
Given that,
Rate of interest is 5%
Since Interest Rate is semiannual,
=5/2=2.5%
So,
The total amount of Interest rate
he receive against Tk 5000
= Tk. (2.5% × 5000 + 2.5% × 4500 + 2.5% × 4000 + 2.5% × 3500 + 2.5% × 3000 + 2.5% ×
2500 + 2.5% × 2000 + 2.5% × 1500 + 2.5% × 1000 + 2.5% × 500)
= Tk. 2.5% × 27500 =Tk. 687.5 Tk. Answer:687.5 Tk

2: A man interest and wages from his investment tk. 5000. If he invests double then the
wages increased 50% and total amount is tk. 8000. What is his actual income in terms of
wages and interest?
[BKB SO -2017]
Solution:
Let, Interest be Tk. X and Wages be Tk. Y
According to the Question,
X +Y= 5000
Or, X =5000-Y------------(1)
When investment is doubled then interest will be doubled
So,
2x +150% of Y=8000
=================
=================
.·.Y=4000
From equation (I), X+ 4000= 5000 .·.X = 1000
Answer: Interest= Tk.1000 and wages= Tk. 4000.

3: 20 workers can finish a work in 30 days. After how many days should 5 workers leave the
job so the work is completed in 35 days?
[BKB SO -2017][SBC AM -2016]
Solution:
Let,
After x days 5 men should leave

20 workers do in 30 days 1 part


20. " 1 " 1/30
x " 1 " x/30

Now
worker Remaining=( 20-5)=15
Day left = (35-x)
And
Suppose
Total portion of work be =1

then,
20 men in 30 d. do 1 part
1 " 1 " " 1/20*30
15 men 35-x " {15*(35-x)/20*30}

According to the question,


x/30+15*(35-x)/20*30=1
=====================
x=15
Answer:15 days

4: A man goes to his office at a certain time. If his waking speed is 5 kmh then he is 7
minutes late.When his speed is 6 kmh he reaches 5 minutes before. How far his office from
his house?
[BKB SO -2017][JBL FF -2016]
Solution:
Let,
Total distance between home to office be X km
According to the Question,
X/5 -7/60 =x/6 +5/60 [Time=distance/Speed]
Or, X/5 - x/6 = 5/60 + 7/60
.·. X= 6 km
Answer: 6 km

5: In a mixture the ratio of apples, peaches and grapes is 6:5:2. If the total mixture is 39
pounds then what is the difference between apples and grapes?
[BKB SO -2017]
Solution:
Given that,
The ratio of Apples, Peaches and Grapes= 6:5:2
.·.The sum of the ratio
= (6+5+2) =13
.·.The quantity of Apple
= (39 × 6/13) = 18 pounds
.·.The quantity of Grapes
= (39 × 2/13) = 6 pounds
.·.The more pounds of apple than grape = (18-6) =12 Pounds
Answer:12 pounds

6.selling 12 candies at a price of tk 10 yields a loss of x% and selling 12 candies at a price of


tk 12 yields a profit of x% ..What is the value of x ?
[South East Bank PO-2017]
Solution:
Let,
12 candies cost price= Tk 100
x% loss selling price=(100-x)

Selling price(100-x) then cp 100 tk


sp 10. " {(100*10)/(100-x)}

x% profit selling price=(100+x) tk


Selling price tk(100+x) then cp tk100
Sp. 12. " {(100*12)/(100+x)}

According to the question,


{(100*10)/(100-x)}={(100*12)/(100+x)}
================
================
Or, x=9.09
So,the value of X is 9.09
Answer:9.09

7.The price of a balcony seat in a theater is 1/3 of the price of a seat in the orchestra. When
the theater is completely sold out, the total receipts from the 600 orchestra seats and the 450
balcony seats are Tk. 4500. What is the price of the orchestra seat?
[JBL EO -CIVIL-2017]
Solution:
Let,
Orchestra seat price = x tk
balchony seat price = x/3 tk
According to the question,
600x+ 450*x/3=4500
=> 600x+150x = 4500
=> 750x = 4500
=> x = 6 tk
Answer: 6 tk

8.A lamp is manufactured to sell for Tk. 35, which yields a profit of 25% of cost. If the profit
is to be reduced to 15% of cost, what will be the retail price of the lamp?
[JBL EO -ELECTRICAL-2017]
Solution:
Let,
Cost price of the lamp is 100 tk
Sp 125 tk then cp 100 tk
:
Sp 35 " (100*35)/125=28

15% profit on cost price then retail price


=(28+28*.15)
=32.20 tk
Answer: 32.20 tk

09. A man sells an article at a profit of 25%. If he had bought it at 20% less and sold it for Tk.
10.50 less, he would have gained 30%. Find the cost price of the article.
[Bangladesh Bank AD -2017]
Solution:
Suppose,
The cost price of the article is Tk. 100

At 25% profit, selling price


=(100+25% of 100)= Tk. 125

At 20% less, cost price


=(100-20% of 100)=Tk.80

At 30% profit, selling price


=(100+30% of 100)=Tk.130

When CP is Tk 100, SP is Tk.130


When CP is Tk 80, SP is (130*80/100)=Tk.104

Difference of two selling price


=(125-104)=Tk.21

When SP difference is Tk. 21 less, CP is Tk.100


When SP difference is Tk. 10.50 less, cost price is {(100*10.50)/21}
= 50 tk

Ans: Tk.50

10. A and B can do a piece of work in 18 days, B and C can do it in 24 days, A and C can do
it in 36 days. In how many days will A, B and C finishes it, working together and separately?
[Bangladesh Bank AD -2017
Solution:
(A + B)'s 1 day's work = 1/18
(B + C)'s 1 day's work = 1/24
(A + C)'s 1 day's work = 1/36
2(A + B + C)'s 1 day's work
=1/18 + 1/24 + 1/36=9/72
=1/8 portion of the work
(A +B + C)'s 1 day's work
=1/16 portion of the work

So,(A +B + C) together can finish the work in 16 days.

A’s 1 day's work


=1/16 – 1/24
= 1/48 portion of the work
A alone can finish the work in 48 days.

B's 1 day's work


= 1/16 – 1/36
=5/144 portion of the work
B alone can finish the work in 144/5=28.8 days

C’s 1 day work


=1/16-1/18
=1/144 portion of the work
C alone can finish the work in 144 days.

Ans: Together=16 days, A=48 days, B=28.8 days and C=144 days.

11.A alone can do a piece of work in 20 days, while B alone can do it in 30 days and C alone
can do it in 60 days.If in every third day B in every fourth day C help A in doing the work
,how many days will be required to complete the whole work?
[34 & 32 th BCS Written Exam]
Solution:
Let,
Total work=1 portion

LCM of 3 & 4= 12

Per 12 days A work= 12 days


Per 12 days B work=(12/3)=4 days
Per 12 days C work=(12/4)=3 days

In 12 days, (A+B+C)'s work


=(12/20+4/30+3/60)
=47/60 part of the work

Remaining work
=(1-47/60)
=13/60 part of the work

[Every third day B help A in doing the work]


Again, in 3 days (A+B)'s work
=(3/20+1/30)
=11/60 part of the work

Remaining work
=(13/60-11/60)
=2/60=1/30 part of the work

On the16th day, (A+C)'s work =1/20+1/60=1/15 part of the work

1/15 part of work is done by (A+C) in 1 day


1/30 part of work is done by (A+C) in 15*(1/30)=1/2 day

Total Time taken to complete the whole work


=(12+3+1/2)
=15.5 days

Ans: 15.5 days

12.The length of a rectangle flower garden is 60 meter & breadth is 40 meter.For nursing the
garden,there has two concrete crossroads with 5 meter width all along its length and the
breadth right at the middle of garden
[IBBL ATO/CASH -2017 English Version]
a)Determine the area of the road?

Solution:
Length of the garden=60 meters
Breadth of the garden=40 meters
[Where the two roads cross each other which length=width= 5 meters]

We know,
The area of the road
= (Length of garden*Length of crossroad)+ (Breadth of garden*Width of crossroad)- (Length
of crossroad* Width of crossroad)

={60*5+40*5-5*5}
= 475 sq.meters.

b)How much cost to build with cement two concrete crossroads ;if each square meter is
required tk 240 ?
Solution:
Cost of two concrete crossroads
=Tk.(475*240)=Tk.114000

Ans: a) 475 sq. meters ;


b) Tk.114000

13.A merchant sold a article for Tk 482 there is a certain amount profit and sold the same
article for Tk 318 loss incur equal to the previous profits.What is the selling price of the
article,If the merchant sold the article at 40% profits?
[IBBL ATO/CASH -2017 English Version]
Solution:
Cost of the Article is =Tk. X
Sold the article for Tk 482, Profit=Tk.(482-x)
Sold the article for Tk 318, Loss=Tk.(x-318)
[Profit=Loss]
According to the question,
(482-x)= (x-318)
So, x=400
At 40% profits on cost price then Selling price
=400*140/100= Tk.560

Ans: Tk.560

14.The costs of equities of symbol A and symbol B (in dollars) are two different positive
integers. If 4 equities of symbol A and 5 equities of symbol B together costs 27 dollars, what
is the total cost of 2 equities of symbol A and 3 equities of symbol B in dollars?
[Nova's GRE MATH Bible]
[IBBL PO-2017]
Solution:
Math Fact:
[এখ A&B Equities এ But এখ এ A and B
equities
এখ খ equities এ 3 Tk ঑ type A & Bএ 12 & 15 Tk
খ ক ]

X and Y be cost of the equities type of A & B


The cost of 4 equities of A=4 X
The cost of 5 equities of B=5Y
According to the question,
4X + 5 Y =27
Let,
4X=P
5Y=Q
So P is multiple of 4 & Q is multiple of 5
Now
P+Q=27
Or, P=27-Q
Since,Q is multiple of 5
Q=5,10,15,20-------etc

Q=5; P=27-5=22 is not multiple of 4


Q=10;P=27-10=17 is not multiple of 4
Q=15;P=27-15=12 is multiple of 4
Q=20;P=27-20=7 is not multiple of 4
So
Q=15
P=12
Now
P=4X
Or,X=12/4=3
Q=5Y
Or,Y=15/5=3
Therefore,
2 equities of A & 3 equities of B
=2X+3Y
=2*3+3*3
=6+9
=15
Answer:15

15.A number when divided successively by 4 and 5 leaves remainders 1 and 4 respectively.
When it is successively divided by 5 and 4, what will be the respective remainders?
[Janata Bank EO-2017]
[Indian Bix,GMATH]
Solution:
Math Fact:
*What is Successive division?
[Successive division খ ক এক ক খ গক এ এ গ
ক গ ঑ গ ক এক ক খ গক ]
Solution:
Let
The number be P
Quotient be X
First case
when divided by 4 quotient is X and remainder is 1
So P= 4x+1........(1)
Second case
Let Quotient be M
when divided by 5 quotient is
M and remainder is 4
X=5M+4
Putting X=5M +4 equation no (1)
P= 20M+17
considering M= 1,2,3,4........
P= 37,57,77....
37 or 57 or 77 etc number when divided successively 5 & 4 Remainder always 2 & 3
So respective remainder is 2 and 3
Answer:2,3
========Proof( ):============
**37/4
Remainder=1 & quotient=9
And
9/5
Remainder=4
So first condition is proof
Second case ====
37/5
Remainder=2 & Quotient=7
And
7/4
Remainder=3
So Answer:2,3

16:A contractor undertakes to do a piece of work in 40 days. He engages 100 men at the
beginning and 100 men after 35 days and completes the work in stipulated time. If he had not
engaged the additional men, how many days behind schedule would it be finished?
[Janata Bank EO-2017]
[Same as PBL JO MCQ-2014]

Solution:
100 Men 40 days done 1 portion W
100 Men 35 days done 35/40«»«»«
=7/8 portion
Remaining work=(1-7/8)
=1/8 portion
Remaining days=(40-35)=5 days
Total worker=(100+100)=200
200 worker 1/8 portion done 5 day
1========1/8========== 5*200
100======1/8=========(5*200/100)=10 days
If he had not engaged the additional men,Then he needed
(10-5)=5 days more to finish the whole work in stipulated time
Answer: 5 days

17:In a certain office, 1/3 of the workers are women, 1/2 of the women are married and 1/3 of
the married women have children. If 3/4 of the men are married and 2/3 of the married have
children, what part of the workers are without children?
[Janata Bank EO-2017]
[MTB MTO MCQ -2013]
Solution:
Let,
The number of workers in the office is x
Female=x/3
Female + Married
=1/2 of x/3
=x/6 portion
Female + Married+ Children
=(1/3 of x/6)
=x/18 portion
From question,
Male=(x-x/3)=2x/3 portion
Male + Married
=3/4 of 2x/3
=x/2 portion
Male + Married+ Children
=(2/3 of x/2)
=x/3
Total worker with children
=x/3+x/18
=7x/18 portion
Total worker without no children
=(x-7x/18)
=11x/18
Required worker no children
=(11x/18)/x
=11/18
Answer:11/18

18:The average weight of three men A, B, and C is 84 kg. Another man D joins the group and
the average now becomes 80 kg. If another man E, whose weight is 3 kg more than that of D,
replaces A, then the average weight of B, C, D and E becomes 79 kg. What is the weight of
A?
[Janata Bank EO-2017]
*[RAKUB SO-2014]
*[AB Bank -2002]
*[Al Arafa Bank-2016]
*[Meghan Bank-2014]
*[Premier Bank-2010]
*[One Bank-2008]
Solution:
The sum of Weight
A+B+C = 84×3 = 252 kg------(1)
If D join,total Weight of
A+B+C+D = 80×4 = 320 kg----(2)

So, the weight of D


=320 - 252 = 68 kg

And weight of E = 68+3 = 71 kg


Again, the sum of Weight of B+C+D+E = 79×4 = 316 kg
Weight of B+C+68+71 = 316 kg
B+C = 316-139 =177 kg
Putting the value of (B+C) equation no (1)
A+B+C=252
Or,A+177=252
Or, A=(252-177)=75
Answer: 75 kg

19:A shirt sold at 6% profits.If the purchase price was 4% less and selling price was Tk 4
more,the profit be 12.5%.What was the purchase price of the shirt.
[MODHUMOTI BANK PO-2017]
#More Written Exam Same As*
*Bangladesh Bank AD-2017
*SBC AM -2016
*BKB -2012
*NBL-2014
*BCS Written
*Non cader Written
*INDIAN BIX
*BAPEX
*Islamic Bank-2013
*BRC-2008
*So on
#Preliminary Exam:
*16th BCS
*MBA-2007-08;1999;1997
*NCC BANK-2002
*PBL-2000
Solutions-1:
Cost price of the article=100 TK
*6% profit first selling price
=(100+100 of 6%)=106TK
*4% less cost price
=(100-100 of 4%)=96 TK
*12.5% profit second selling price
=(100+100 of 12.5%)=112.5 TK
Now,
Cost price TK 100 then Sp=112.5 TK
»»»»»»»»»»TK1»»»»=(112.5/100)TK
»»»»»»»»»» TK80»»»=(112.5*96/100)
=108 TK
Differences of two selling price
=(108-106)=2 Tk
When
TK 2 more then cost price=100 TK
TK4»»»»»»»»»»»»»»»»=(100*4)/2=200TK
Answer:200TK

Solution-2:
Let,
Cost price be= x TK
First Selling price=(x+x of 6%)
=53x/50TK
4%% less cost price
=(100-4) or 96% of x=24x/25 TK
Second SP=(24x/25+24x/25 of 12.5%)
=216x/200 TK
According to the question,
(216x/200)-(53x/50)=4
=================
=================
Or, X=200
The cost price of the article=200 TK
Answer:200 TK

20:A man's running speed is 3 times of his walking speed.He runs a distance and come back
by walking total time taken 2 hrs.what was the distance if he runs 9 miles per hour.
[MODHUMOTI BANK PO-2017]
[PBL -2008]
Solution:
Let,
Distance be=D miles
Runs speed=9 m/h
Walking speed=9/3=3 m/h

According to the question,


D/3 +D/9=2
==========
==========
Or, D=4.5 miles
Answer:4.5 miles

21:A,B,C started a job which they can complete in 2 days.B can do the job in 5 days and C
can do it in 4 days.After working for 1 day,both B & C left. A complete testbed of the work
how many days?
Solution:
Let,
Total portion work be 1 portion
(A+B+C) 1 day do the job =1/2 portion
B & C 1 day do the job
=(1/5+1/4) portion
=9/20 portion
So,
A 1 day done the job
=(1/2-9/20) portion
=1/20 portion
Remaining work=(1-1/2)=1/2 portion
A 1/20 portion job done=1 day
A 1/2 portion job done=(20/2)
=10 days
Answer:10 days

22:Solve the equation:


1/(2x-5) + 1/(2x-11) = 1/(2x-7) + 1/(2x-9)
Answer:4

23.A number consists of three digits whose sum is 10.The middle digit is equal to the sum of
the other two and the number will be increased by 99 if these two digit are reversed.What is
the number?
[DBBL AO-2017]
Solution:
Let,
Unit digit = x
Tenth digit = y
&
Hundred digit = z
So, The original number
=100z+10y+x
1st condition,
x+y+z=10-------------------(1)
2nd condition,
y=x+z----------------------(2)
If unit digit & Hundred digit are interchanged their place the new number is =100x +10y +z
According to the question,
(100x +10y +z)-(100z+10y+x)=99
===========================
===========================
Or, z=x-1-------------------(3)
Putting the value of z in equation(2)
y= x+x-1
Or, y=2x -1----------(4)
again,
Putting the value of y and z in equation(1)
x+2x -1+x -1=10
Or, x=3
Now,putting the value of x in equation(4)
y=5
Putting the value of x & y in equation(1)
Z=2
Hence, the original Number is
= (100*2)+(10*5)+3
=253
Answer:253

24.A picnic was arranged by 'm' students.Total cost of picnic was estimated to be 'y'
Tk.Unfortunately, z students withdrew their names from the picnic.How many more tk would
each of the remaining students have to pay?
[DBBL AO-2017]
Solution:
Given that,
Total student = m
&
Total cost of picnic was estimated to be= y tk
So Cost per student was = y/m Tk
Since z student withdrew their name from picnic,
Now remaining student=(m-z)
And
Nwe cost per student is = y/(m-z)
So, Extra cost needed for rest of the stds now = y/(m-z) -y/m
= yz/m(m-z)

Answer: yz/m(m-z)

25. A worker was hired for 7 days. The 2nd day he was paid tk.10 more each day than what
he was paid the previous day of the work. The total amount he paid in the 1st 4 days of work
equal to the total amount he was paid in last days, what is starting pay?
[Bangladesh Tourism Board-AD-2017]
Solution:
Let,
Salary of the 1st day = X
.·. 2nd to 7th day salary was (x+10), (x+20), (x+30), (x+40), (x+50), (x+60) respectively
According to the question, x+x+10+x+20+x+30=
x+40+x+50+x+60
Or, 4x+60= 3x+150
Or, x= 90
.·. Starting pay was= Tk.90
Answer: Tk 90

26.Arif’s salary is twice that of Babu’s salary. Kabir’s salary is 1/3 of Arif’s and Malek’s
salary is 2/3 of that Babu’s salary.Total salary of kabir and Malek are what proportion of
Babu’s salary?
[Bangladesh Tourism Board-AD-2017]
Solution:
Let,
Babu’s salary= X
.·. Arif’s salary= 2X
Kabir’s salary= 1/3 × 2X = 2x/3 Malek’s salary= 2/3× X = 2x/3
.·. Required proportion
= (Kabir+ Malek): Babu
= (2x/3 + 2x/3): X
= 4:3
Answer:4:3

27. A, B and C of them working alone can complete a job in 6,8,12 days respectively. If all
three of them work together to complete a job and earn Tk. 2340 what will be C’s share
[Bangladesh Tourism Board-AD-2017]
Solution:
In 1 day,
A, B and C alone can do
1/6, 1/8 and 1/12 part of the work respectively
Ratio of their working rate
.·. A: B: C= 1/6:1/8:1/12
= 4:3:2 [Multiply by 24]
.·. Sum of the ratio= 4+3+2= 9
.·. C’s share=Tk. (2340 × 2/9)
= Tk.520
Answer: Tk.520

28.Each person in a group of 110 investors has investments in either equities or securities or
both. Exactly 25% of the investors in equities have investments in securities, and exactly 40%
of the investors in securities have investments in equities.How many have investment in
equities?
{Very Important Math}
Solution:
The investors can be categorized into three groups:
(1) Those who have investments in equities only.
(2) Those who have investments in securities only.
(3) Those who have investments in both equities and securities.
Let ,
x, y, and z denote the number of people in the respective categories. Since the total number of
investors is 110, we have
x + y + z = 110 ------------- (1)

Also,
The number of people with investments in equities is x + z
and
The number of people with investments in securities is y + z.
Since exactly 25% of the investors in equities have investments in securities, we have the
equation
25% of (x+z)=z
x = 3z ------------------- (2)

Since exactly 40% of the investors in securities have investments in equities, we have the
equation
40% of (y+z)=z
y = {3z}/2 -------------------- (3)

Substituting equations (2) and (3) into equation (1) yields


3z+(3z/2)+z=110
===============
Or, Z=20

Hence,
the number of people with investments in equities is:
=x+z
=3z+z
=3×20+20
=60+20
=80
Answer:80

29. A drink contains 20% mango juice, 20% guava juice and 60% apple juice. You added
250ml of water to 750 ml of the drink. Now what will be the ratio of water to apple juice in
the diluted drink?
[BAPEX ASSISTANT MANAGER GENERAL 2017]
Solution:
Given that,
A drink contains 20% mango juice, 20% guava juice
and
60% apple juice.
.·. The ratio of mango, guava and apple juice
= 20: 20: 60
=2:2:6
= 1:1:3
.·. The sum of the ratio
= (1+1+3) =5
.·. The quantity of apple juice
= (750 x 3/5) = 450 ml
Here also given that, 250 ml water is added in 750 ml of the drink.
So the ratio of water and apple juice in the drink
= 250:450
=25:45
=5:9
Answer:5:9

30.Mr Rahman invested a certain sum of money in a bank that paid simple interest. The
amount grew to tk. 240 at the end of 2 years. He waited for another 3 years and got a final
amount of Tk. 300. What was the principal amount that he invested at the beginning?
[BAPEX ASSISTANT MANAGER GENERAL 2017]
Solution:
Given that,
5 years interest+Principal=Tk.300
2 years interest+Principal=Tk.240
-----------------------------------------------------
[Minus] 3 years interest =Tk.60
2 years interest =[(60*2)/3]
= Tk 40
Hence,
Principal =Tk.(240-40)=Tk.200
Answer:200 Tk

31.In a class of 40 students, each student plays at least one of the games: chess, carom and
table tennis. Among the students, 18 play chess, 20 play table tennis and 27 play carom.
Further, 7 students play both chess and table tennis, 12 play both table tennis and carom and
4 play chess, carom and table tennis together. Find the number of students who play chess
and carom but not table tennis.
[Janata Bank AEO-RC-2017]
Solution:
Let ,
Number of students play chess=n(A)
Number of students play carom =n(B)
And
Number of students play table tennis=n(C)
Given that,
n(U)=40
n(A)=18
n(B)=27
n(C)=20
n(A^C) = 7
n(B^C)=12
and
n(A^B^ C)=4
Suppose,
n(A^B)=?
We know that
n(U)= n(A)+ n(B)+n(C)-n(A^B) -n(B^C)- n(A^C)+n(A^B^C)
Or,40=18+27+20-7-12-n(A^B)+4
Or,40=50-n(A^B)
Or, n(A^B)=10
But there are 4 people who play all three games.
So,
The number of who play chess and carom but not table tennis
=n(A^B)-n(A^B^C)
=10-4
=6
Answer: 6

32. Aslam and Babul invested in a business in the ratio 3:2. Assume that 5 percent of total
profit goes to workers' provident fund. If Aslam's share is Tk. 855000, what is the amount of
total profit?
[Janata Bank AEO-RC-2017]
Solution:
Suppose,
Aslam's profit share is = 3x
And Babul's profit share is = 2x
So, total profit after Worker Profodend Fund deduction (i.e: 95% of total profit)
= 3x+2x = 5x
According to the question:
3x = 855000
Or, x=855000/3
So,
5x = 855000×5/3 = 1425000

According to the question, 5% of total profit goes to workers profident Fund


So, the remaining 95% of profit is distributed between Aslam and Babul.

So, 95% of total profit = 1425000


100% of total profit = 1425000×100/95 = 1500000

Ans: Tk. 1500000

33. Alam invested an amount of Tk. 13900 divided in two different deposits at Bank X and
Bank Y at the simple interest rate of 14 percent and 11 percent per annum, respectively. If the
total amount of simple interest earned in two years is Tk. 3058, what was the amount invested
in the deposit of Bank Y?
[Janata Bank AEO-RC-2017]
Solution:
Suppose, the amount invested in Bank Y was = Tk. M
So, the amount invested in Bank X was = Tk. (13900-M)
.
According to the question:
2{14(13900-M)/100 + 11M/100}= 3058
(194600 - 14M + 11M)/100 = 1529
Or, M = 13900

Ans: Tk. 13900.


34. Rashed is driving at a speed of 120 km per hour. Mahi started one hour late and started
driving at a speed of 140 km per hour. How long will it take for Mahi to catch up Rashed?
[Janata Bank AEO-RC-2017]
Solution:
When Mahi starts,
Rashed is 120×1 = 120 km away from his
Relative speed of Mahi to Rashed
= (140-120) kmph
= 20 kmph
Required time to catch up Rashed = 120/20 hr
= 6 hr.

Ans: 6 hours.

35. A table is three times as long as it is wide. If it were 5 feet shorter and 5 feet wider, it
would be a square. How long and how wide is the table?
[Janata Bank AEO-RC-2017]
Solution:
Let,
The width of the table is x feet.
So,
The length of the table is 3x feet.
According to the question,
3x-5 = x+5
2x = 10
x=5
So, the length of the table is
= 3×5 = 15 feet,
and the width of the table is
= 5 feet.
Answer: 15 feet, 5 feet.

36. Given a = 3 + 2√2, find the value of (√a - 1/√a)


[Janata Bank AEO-RC-2017]
Solution:
Given that,
a = 3 + 2√2
Or, a = 2 + 1 + 2√2
Or, a= (√2)² + 2×√2×1 + 1²
Or, a = (√2 + 1)²
so, √a = (√2+1)
And,
1/√a = 1/(√2+1)
= (√2-1)/{(√2+1)(√2-1)}
= (√2-1)

Therefore,
√a - 1/√a = (√2+1) - (√2-1) = 2.
Answer: 2
37.In a shop, the cost of 4 shirts, 4 pairs of trousers and 2 hats is Tk. 5600. The cost of 9
shirts, 9 pairs of trousers and 6 hats is Tk. 12900. What is the total cost of 1 shirt, 1 pair of
trousers and 1 hat?
[Janata Bank AEO-RC-2017]
Solution:
Let,
The cost of a shirt is x Tk
A pair(Two) of trousers is y Tk
and a hat is z
According to the question,
4x+4y+2z = 5600 ----------------- (i)
9x+9y+6z = 12900 --------------- (ii)
From (ii) no equation,
Or, 9x+9y+6z= 12600
Or, 3x +3y+2z =4300------------(iii)

(i) no equation--(ii) no equation=>


4x + 4y + 2z =5600
3x + 3y + 2z =4300
---------------------------------
Or, x + y =1300 ---------(iv)
(I) --(ii)*4 =
4x + 4y + 2z=5600
4x +4y. =1300*4
-----------------------------------
Or , 2 z =400
Or, z =200

So, x+y+z = 1300+200 = 1500


And Total cost of 1 shirt, 1 pair of trousers and 1 hat is Tk. 1500.

Answer: Tk. 1500

38.Mr. X borrowed Tk. 500 at 5% simple interest per year. After some time, he borrowed Tk.
400 at 3.5% simple interest per year for the second time. Six months after the second time
borrowing, he repaid both the borrowed money along with interest and the amount repaid was
Tk. 994.50. How many years after the first time borrowing Mr. X repaid the borrowed
money?
[Bank ASIA MTO -2017]
[KSB SO-2013]
[23th BCS WRITTEN]
Solution:
Total amount=(400+500)=900 Tk
Total interest=(994.5-900)=94.5 Tk

3.5% interest rate


400 Tk 1/2 years interest
=[(7*400)/(2*100*2)]
= 7 Tk.
Remaining interest=(94.5-7)=87.5 Tk
5% rate of interest
500 Tk 1 years interest
=(5*500/100)=25 Tk

25 Tk interest for 1 years


87.5 Tk interest(87.5/25)=3.5 years

So after 3.5 years then he took second amount


Answer:3.5 years

39..Rahim can do a piece of work in 80 days.Rahim works for 10 days and Karim alone
finishes the rest of the work in 42 days.How much time would it take for the two of them
together to complete the whole work?
[Bank ASIA MTO -2017]
Solution:
Let,
.
Total portion work=1
Rahim 80 days done=1 portion
Rahim 10 days done=(10/80)=1/8 portion
Remaining work=(1-1/8)
=7/8 portion
Karim 7/8 portion done=42 days
Karim 1 or full portion work done
=(42*8/7)=48 days
Both Rahim & Karim 1 day done
=(1/80+1/48) portion
=1/30 portion
Both
1/30 portion done=1 days
Full work done=(1*30)/1=30 days
Answer:30 days

40. Solve the equation


(x-4)/(x-1) + (x-7)/(x-3) + (x-2)/(x-9)=3
Solution:
{(x-4/x-1)-1}+{(x-7/x-3)-1}+{(x-2/x-9)-1}=0
Or, { -3/x-1}-{4/x-3}+{7/x-9} =0
Or, {7/(x-9)}={3(x-3)+4(x-1))/(x-1)(x-3)}
Or, 7(x^2 -4x+3)=(x-9)(7x-13)
Or, 7x^2 -28x+21= 7x^2 -76x+117
Or, 48x=96
Or, x=2
Answer:2

41.The length of a rectangle field is 1.5 times of width.An amount of Tk 10,260 was needed
to cover the field with grass at the rate of 1.9 Tk per square meter.How much would it cost to
fence the four sides of the rectangular field at the rate of Tk 2.5 per meter?
[Bank ASIA MTO -2017]
[One Bank (SCO)-2017]
[Lankabangla Finance -MTO-2017]
Solution:
Area of the rectangle
=(10260/1.9)
=5400 square meter
Let,
Width of the rectangle=x meter

Length of rectangle=(3x/2) meter


We know,
(3x/2)*x=5400
===========
==========
Or, x=60
And
Length=(3*60/2)=90 meter
Perimeter of rectangle
=2[90+60]
=300 meter
So total Tk needed around rectangle fence=(300*2.5)=750 Tk
Answer:750 Tk

42.A, B, C can complete a piece of work in 16, 32, 48 days respectively.They started working
together but C left after working 4 days and B left 2 days before the completion of the
work.How many days require to complete the work?
[Social Islamic Bank-PO-207]
[LankaBangla Finance Ltd.MTO-2017]
Solution-1:
Let,
Total time taken to complete the whole work is x days
According to the question,
x/16+(x-2)/32+(4/48)=1
Or,6x+3x-6+8=96
Or,9x=94/9
Answer:94/9 or 10.44 days

Alternative Solution-2:
Suppose,
Total work be 96 units
[LCM of 26,32 & 48 is 96]
Efficiency of work rate per day of A,B & C is
A= 96/16=6 units
B= 96/32=3 units
C=96/48=2 units
Worked done by A,B,C in 4 days
= {(6+3+3)*4}=44 units
Worked done by A in 2 days
= 2*6= 12 units
Remaining work
= 96-(44+12)
=40 units
A & B 1 day done=(6+3)=9 units
Remaining worked done by A & B
= 40/9
Total days required to finish the whole work={4+2+(40/9)}=10(4/9) days

Answer:10.44 days

43.The total cost price of two watches is Tk. 840. One is sold at a profit of 16% and the other
at a loss of 12%. There is no loss or gain in the whole transaction. The cost price of the watch
on which the shopkeeper gains, is-
[Social Islamic Bank-PO-2017]
Solution:
Let,
Cost price of first watch is Tk x
So,
Cost price of Second watch
=Tk (840-x)
16% profits Selling price of first watch={(116*x)/100} Tk
12% loss SP of second watch
={88*(840-x)/100} Tk
According to the question,
(116x/100)+{(88*840-88x)/100}=840
or, (116x+ 73920-88x)/100=840
or, 28x+73920= 84000
or, 28x = 84000-73920
or, x= 100840/28
or, x= 360
Now, Cost price of first watch
= 360 Tk
C.p. of second = (840-x) = (840-360)= 480Tk.(answer
Solution-2:
Let,
Cost price of first watch is tk.x
And cost price of second watch is tk(840-x)
According to the question,
16% of x = 12% of (840-x)
Or, 16x = 12*(840-x)
Or, 16x = 10080 -12x
Or, 28x = 10080
Or, x = 360
And cost price of other watch is
=(840-360)
=480 Tk
So , cost price of two watches are 360 & 480
Answer: 360 Tk ; 480 Tk

44.Solved the equation


1/(x+2) + 1/(x+5)=1/(x+4) + 1/(x+3)
[Social Islamic Bank-PO-207]
[LankaBangla Finance Ltd.MTO-2017]
Solution:
1/( x+2)-1/(x+4)=1/(x+3)-1/(x+5)
or, ( x+4-x-2)/(x+2)(x+4)=(x+5-x-3)/ (x+3)(x+5)
or, 2/( x+2)((x+4)=2/(x+3)(x+5)
or, 1/ ( x+2)(x+4) =1/(x+3)(x+5)
or, (x+2)(x+4)=(x+3)(x+5)
or, x^2+4x+2x+8=x^2 +3x+5x+15
or, 6x+8=8x+15
or, 2x= -7
or, x= -7/2
Answer: -(7/2)

45.The perimeter of a square field is equal to the perimeter of a rectangle field.Length of the
rectangle fields is 3 times of its width and the area is 768 square meter.How many square
sized tiles of 80 centimetre wide will be required to cover the square field?
[Social Islamic Bank-PO-2017]
Solution:
Let,
The width of the rectangle be x meter
So,
The length of the rectangle be 3x meter
According to the question,
3x*x=768
0r, x =16
So,
The perimeter of the rectangle,
=2(3*16+16)
=128
So, the perimeter of the square is 128 meters
Suppose,
Each Side of square is a
From question,
4a=128
Or, a =32
Area of the square is
=(a)^2
=(32)^2
=1024 square meter
=10240000 square cm
Area of the tiles
=(80*80) square cm
=6400 square cm
Number of tiles
=10240000/6400
=1600 piece
Answer:1600
46. Salam used a part of Tk.100000 to purchase a television. Of the remaining portion, he
invested 1/3 of it at 4% simple annual interest and 2/3 of it at 6% simple annual interest. If,
after a year, the income from two investments totaled tk.
320, what was the purchase price of the television?
[ONE Bank SCO-2017]

Solution-1:
Let,
The purchase price of the TV =xTk
and
Remaining amount of money= (100,000 – x) Tk
First portion interest
=(100,000-x)*1/3 *4%
=(100000-x)/75
Second portion interest
=(100,000– x)*2/3 * 6%
=(100000-X)/25

According to the question


(100000-x)/75 +(100000-X)/25=320
Or,[(100000-X)(1/75+1/25)]=320
Or,[(100000-X)*4/75]=320
Or,400000-4x=320*75
Or, x = 94,000
Answer :94,000 Tk
Solution-2:
Let,
The purchase price of the TV=Tk x
Given that,
Total amount=100000 Tk
Remaining amount of money
=Tk(100,000 – x)
According to the question
{(100,000 – x)*1/3 *4%}+{(100,000 –x)*2/3 * 6%} = 320
Or,(1000/3*4+12000/3)–(4x/300) (12x/300)= 320
Or,4000/3+(12000/3)–(4x/300)– 12x/300 = 320
Or, 400000+1200000 -4x–12x= 320*300
Or,16x =1600000–96000
Or, 16x = 1504000
Or, x = 94,000
Answer:94,000 Tk

47.A man to go 10 km to catch a bus .He walks part of the way at 7 km/hr and runs the rest of
the way at 12 km/hr.If he takes 1 hr 15 minutes to complete his journey,Find how far he
walked.
[Dhaka BANK-MTO-2004]
[LankaBangla Finance Ltd.MTO-2017]
Solution:
Let,
He walked x km
and
He runs(10-X) km
Total time taken=1 hr 15=5/4 hrs
According to the question,
X/7 + (10-X)/12=5/4
=================
=================
Or, x=7
So, He walks 7 km
Answer:7 km

48.There are 87 balls in a jar.Each ball is painted with at least one of two colors,red or
green.It is observed that 2/7 of the ball that have red colors also have green color,while 3/7 of
the balls that have green color also have red color.What fraction of the balls in the jar has
both red and green colors?
[UCB PO-2017][DBBL PO-2016]
{Important for Written}
Solution:
Suppose,
Only Red painted=R
Only Green painted=G
Both Red & Green painted=X
So,
R+G+X = 87-----------(1)
Also,
Red painted=(R+X)
Green painted=(G+X)
So,
(R+X)*2/7=X
Or, R =5x/2-------------(2)
And,
(G+X)*3/7=X
Or, G =4x/7------------(3)
From equations(1)
R+G+X =87
Or, (5x/2)+(4x/3)+x =87
Or, x =18
Required fraction
=18/87
=6/29
Answer:6/29

49.A,B and C started a business by investing the amount of Tk 600 ,Tk 800 and Tk 900
respectively.After few months,A invested an amount of Tk 300 in the same business.At the
end of the year,they profited of Tk 300.After how many months did A invest Tk 300 if C's
share of profits be Tk 108 ?
[Cotton Development Board-1997]
[Very Important Math]
Solution:
Let,
For x month A invest 300 tk.
So,
Investment of A ,B & C
{(600*12)+300*X} : 800*12 : 900*12
=(24+x):32:36
Sum of Ratio
=24+X+32+36
=92+x
C' share
300*36/(92+x)=108
Or, x=8

A invest Tk 300 for 8 months


SO,
A invest 300 tk after
(12-8)=4 months
Answer: 4 months

50. A sum of money is borrowed and paid back in two annual installments of TK.882 each
allowing 5% compound interest .The sum borrowed was
[JBL EO MCQ-2017]
{Important for Written}
Solution:
Let,
The sum is =x tk
In first years compound
882=x(1+5/100)^1
Or,882=21x/20
Or,x=840
Second year,
Amount=(840+882)=1722 tk
Second year compunted,
1722=x(1+5/100)^1
Or,1722=21x/20
Or,x=1640
The sum borrowed was=1640 tk
Answer:1640 tk

51. Solve the equation : 4/(2x+3)+15/(5x+4)=35/(7x+6)


[One Bank(SCO)-2017]
Solution :
4/(2x+3) + 15/(5x+4)=35/(7x+6)
Or,4(2x+3)+15/(5x+4)=21/(7x+6)+14/(7x+6)
Or,4/(2x+3)- 14/(7x+6)=
21/(7x+6)-15/(5x+4)
Or,( 28x+24-28x-42)/(2x+3)= (105x+84-105x-90)/(5x+4)
Or,-18/(2x+3) = -6/(5x+4)
Or, 3/(2x+3)= 1/(5x+4)
Or, 15x +12= 2x+3
Or, 13x =-9
Or, x =-9/13
Answer:-(9/13)

52. The ratio between the length and the breadth of a rectangular park is 3:2. If a man cycling
along the boundary of the park at the speed of 12 km/hr completes one round in 8 minutes,
then what is the area of the park (in sq. m)?
[City Bank MTO -2017 ]
[Al Arafah Islamic Bank-2013]
Solution:
Let,
Length=3x
Breadth=2x
From the question,
60 minutes the man covere=12km=12000 m
So,
In 8 minutes the man covere=[(12000*8)/60]
=1600 meters
Perimeter of the rectangle is
2(3x+2x) = 1600
x= 160
The Area of the rectangle
=(3x*2x)
= (3*160)(2*160)
=153600 square meter
Ans: 153600 sq.m

53.During the next tree plantation week, Mr. X is considering planting trees in one of its own
rectangular piece of land which is 90 feet long 66 feet wide. This is suspended by boundary
wall of 5 feet height. It has been decided that trees will be planted leaving 5 feet and free
from the wall in all four sides. It was been decide that the distance from one tree to another in
both row and column will be 4 feet. What is the maximum numbers of trees that can be
planted in the land?
[City Bank MTO -2017 ]
[Standard Bank MTO-2016]
Solution:
Given that,
The length of the field=90 feet
Possible length of the rectangle
= 90-(5*2)
= 80 feet
Possible Width of the rectangle
= 66-(5*2)
= 56 feet
Possible trees in row
= (56/4 )+ 1
= 15
And
Possible trees in Column
= (80/4) + 1
=21
Maximum tree
= 21*15
= 315

Ans: 315

54. If 9 engines consume 24 metric tons of coal, when each is working 8 hours a day, how
much coal will be required for 8 engines, each running 13 hours a day, it being given that 3
engines of former type consume as much as 4 engines of latter type?
[BB Officer(Cash)-2017]
Solution:
According to the question,
3 Engine former = 4 Engine later.
1 Engine former = 4/3 of later
So, 9 Engine of former =(4*9/3)
=12 Engine of later type

12 engine later type each working 8 hrs a day consume 24 metric tons coal
So,
8 engine later type each working 13 hrs a day consume
={(24*8*13)/(12*8)}
=26 metric tons

Answer: 26 metric tons

55.Shakil started a business investing Tk. 25000 in 2009. In 2010, he invested an additional
amount of Tk. 10000 and Raihan joined him with an amount of Tk. 35000. In 2011, Shakil
invested another additional amount of Tk. 10000 and Jafor joined them with an amount of
Tk.35000. What will be Raihan's share in profit of Tk. 150000 earned at the end of 3 years
from the start of the business in 2009?
[BB Officer(Cash)-2017]
Solution:
According to Question,
Shakil Investment Amount in 3 years
=(25000×3+10000×2+1000×2) =105000 Tk
Raihan Investment Amount in 2 years
=(35000×2) Tk
= 70000 Tk
Jafor Investment Amount in 1 years
=(35000×1) Tk
= 35000 Tk
Now their Investment Ratio
=105000: 70000: 3500
= 3:2:1
Sum of their Investment Ratio
=(3+ 2 + 1)
=6
Raihan’s Profit share
=(150000×2/6)
= 50000 Tk
Answer: 5000 Tk

56.Dawood invested certain amount in three different schemes A, B and C with the rate of
10% p.a., 12% p.a. and 15% p.a. respectively. If the total interest accrued in one year was Tk.
3200 and the amount invested in Scheme C was 150% of the amount invested in Scheme A
and 240% of the amount invested in Scheme B, what was the amount invested in Scheme B ?
[BB Officer(Cash)-2017]
Solution:
Let,
A's investment was x Tk
From question conditions,
C' investment was
=150% of A
=(150/100) of X
=3x/2 Tk
And
C =240% of B
Or, 3x/2=(240/100) of B
Or, B =5x/8
According to the question,
10%of x+(5x/8)of 12%+(3x/2)of 15%=3200
Or, (10x/100)+(12*5x)/(8*100)+(15*3x)/(2*100)=3200
============================
============================
Or, x =8000
The amount of B invested in the scheme
={(5*8000)/8}
=5,000 Tk
Answer: Tk 5000

57.Rahim bought two varietis of rice costing taka 5 per kg and 6 per kg each,and mixed them
in some ratio, then he sold the mixture at 7 tk per kg making a profit of 20%, what was the
ratio of the mixture of two variety rice?
[Bangladesh Bank AD-2015]
[Janata Bank EO -2012]
[Rupali Bank SO-2013]
[Bangladesh Bank AD-2014]
Solution -1:
Let,
Rahim bought x kg rice at tk 5 & y kg rice at tk 6/kg
So, total cost =(5x + 6y) Tk
Selling price = (7x + 7y) Tk
20% profit = 20% of(5x + 6y)
= (5x + 6y)/5
According to question,
Selling price -Cost price =Profit
(7x + 7y) - (5x + 6y) = (5x + 6y)/5
Or, 2x + y = (5x + 6y)/5
Or, 10x + 5y = 5x + 6y
Or, 5x = y
Or, x/y = 1/5
Answer :1:5

Solution -2
Let,
Rahim bought x kg rice at tk 5 & y kg rice at tk 6/kg
So, total cost =(5x + 6y) Tk
Selling price = (7x + 7y) Tk
According to the question,
(7x+7y)=120% of (5x +6x)
======================
======================
Or, x/y=1/5
The ratio of the mixture=1:5
Answer:1:5

Solutions-3:
Short Cut/Rules of Allegation:
20% profit
Cost price=(100*7)/120
=70/12

5----------------------------------6

70/12

[6-70/12]-------------------[(70/12)-5]
=2-------------------------------10
=1:5
Answer:1:5

59.In what ratio must a grocer mix two varieties of tea worthTk 60 a kg & 65 a kg so that by
selling the mixture at Tk 68.20 a kg he may gain 10% ?
[Islami Bank PO -2017]
[GRE Big Book]
[Exim Bank-2016]
[Indian Bix]
Solution:
Let
The grocer buy 60 Tk kg=x kg
The grocer buy 65 Tk kg =y kg
The cost price of (x+y) kg
=(60x+65y) Tk
And selling price of (x+y) kg
=68.20(x+y) Tk
10% profits selling price
=110%(60x+65y)
According to the question,
68.20(x+y)=110%(60x+65y)
Or,22x =33y
X:Y=3:2
Answer:3:2
Alternative way====
According to the question,
Selling price-cost price=profit
68.20(X+Y)-(60X+65Y)=
10% of(60x+65y)
===========================
==========================
Or,X:Y=3:2
Answer:3:2

Short Cut:=====

10% profits sp=110 Tk


Cost price=(100*68.20)/110
=62 Tk

60----------------65

62

(65-62) (62-60)
=3. =2
Answer:3:2

58.It takes 120 metric tone water to sink a ship.Through a hole in the full of the ship,water is
entering the ship at a rate of 2 metric tone per minute. At the same time,water is being
pumped out at the rate 1.5 metric tone per minute using one pump. After 1 hour and 20
minutes another pump of same capacity was started. How much more time will it take to
pump all the water out of the ship?
[BGFCL-2017]
Solution:
Here,
Water enter ship per minute
=2 metric tone
Water pumped out per minute
=1.5 metric tone
Water entered more per minute
=(2-1.5)=.5 metric tone
**After 1 hr & 20 minutes or 80 minutes
Water poured =(80*.5)=40 MT
After 80 minutes another capacity
pumped out pump started
So two pump pumped out water per minute=(1.5+1.5)=3 metric. tones
And 1 minute more water
pumped out=(3-2)=1 metric tone
Now,
1MT pumped out =1 minute
40 «»«»«»«»«»«»«=(40*1)/1
=40 minutes
Answer:40 minutes

60.Twenty-four men can complete a work in sixteen days.Thirty-two women can complete
the same work in twenty-four days. Sixteen men and sixteen women started working for
twelve days. How many more men are to be added to complete the work remaining work in 2
days?
[PBL SO/Officer-2017]
Solution:
Here,
24 men can complete in 16 days 1 portion work
1 man can complete in 1 day=1/(16*24)
=1/384 portion
Again,
32 women can complete in 24 days 1 portion
1 woman can complete in 1 day=1(32*24)
=1/768 portion
Both,
1man +1 woman work in 1 day=(1/384+1/768)
16 men +16 women work 12 days
=[(16*12)/384 + (16*12)/768] portion
=(1/2+1/4) portion
=3/4 portion

Remaining work
=1-3/4
=1/4 portion
Let,
X more men should be added then the remaining work complete in 2 days
Total men =(16+x)
1 man in 1 day done=1/384 portion
(16+x) men in 2 days done=(16+x)*2/384 portion

Similarly,
16 women in 2 days done=(16*2)/768 portion

According to the question,


(16+x)*2/384 +32/768=1/4
Or,(32+2x)/384=1/4-32/768
Or,(32+2x)/384=(192-32)/768
Or,(32+2x)/384=160/768
Or, 32+2x=160/2
Or,2x=80-32
Or,2x=48
Or, x=24
Answer:24 men
61.The average speed of Train in the onward Journey is 25% more than that is the return
journey .The train halts for an hours on reaching the destination. The total time taken for the
complete journey to and fro17 hours, covering a distance of 800 km. Find the speed of the
Train is the onward journe?
[PBL SO/Officer-2017]
Solution:
Let,
Return speed of the train is x kmh
So,
Onward speed of the train be(x+x of 25%) =5x/4
The train halts for an hour
So,Actual time taken the train
(17-1)=16 hours

According to the question,


400/x + 400/(5x/4)=16
Or,400/x +400*4/5x=16
Or,(400*5+400*4)/5x=16
Or,x=(400*9)/(16*5)
Or, x=45

So, Onward speed of the train is =5x/4


=(5*45)/4
=56.25 kmh
Answer:56.25 km/h

62.The population of a town increased from1,75,000 to 2,62,500 in a decade.Find the average


percent increase of population per year is?
[PBL TAT -2017]
Solution:
Population increased in a decade or 10 years
=262500-175000
=87500
Per year increased
=(87500/10)
=8750

Required percentage
={(8750*100)/175000}
=5%
Answer:5%

63.A,B,C enter into partnership.A invests 3 times as much as B invests two-third of What C
invests. At the end of the year, the profit earned is Tk 6600.What is the share of B?
[PBL TAT -2017]
Solution:
Let,
The amount of investment of C be Tk. x
So,
The amount of investment of B be
Tk (2x/3)
The amount of investment of A be
=(3*2x/3)
=Tk 2x
The ratio of investment of A,B & C
=2x:2x/3:x
=6:2:3
Sum of the ratio=6+2+3=11
So,
The amount of share
=(66000*2/11)
=Tk.1200
Answer:1200 Tk

64.Mr X invested his capital in two parts,one at 6% and another at 7% .At the end of two
years he received Tk 354 as interest at all.If one fourth of first part of his investment is equal
to one fifth of second part of investment.What was his total investment in his business?
[KSB SO-2017]
[Bangladesh Gas Field Assistant Account/Finance-2017]
Solution:
Suppose,
First portion=x
Second portion=y
According to the First condition,
x/4=y/5
Or, x=4y/5-------------(1)
And,
Second condition,
{(x*6*2)/100}+{(y*7*2)/100}=354
===========================
===========================
Or, y=1500
From (1) equation,
x=(4*1500)/5=1200
So,Total amount he invest
=(1200+1500) Tk
=2700 Tk
Answer:2700 Tk

65.A and B together can do a piece of work in 12 days, which B and C can do in 16 days.
After A has been working at it for 5 days and B for 7days C finishes in 13 days. In how many
days C alone will do the work?
{Very Important Math}
Solution:
Work done by A & B in 1 day=1/12
Work done by B & C in 1 day=1/16
Let,
Work done by C in 1 day=x
From question,
Work done by A in 5 days+ B in 7 days+ C in 13 days=1
So,
(A+B)'s 5 days work+(B+C)'s 2 days work + C's 11x days work=1
[See Picture Combination work ]
Or, (5/12)+(2/16)+11x=1
Or, x =1/24
So, C alone can finish the work it in 24 days
Answer:24 days

66.Two outlets of a motor manufacturing company reported that revenue from outlet X in
2013 was down 11 percent from 2012 and revenue from outlet Y in 2013 were up 7 percent
from 2012. If total revenue from outlet X and Y in 2013 was up 1 percent from 2012, what is
the ratio of revenue from outlet X in 2012 from outlet Y in 2012?
[Jamuna Bank PO-2014]
Solution:
Let,
In 2012, X & Y outlets income was 100x & 100 y
Total Revenue of X & Y outlets in 2012 was
=100X +100Y
=100(X+Y)
In 2013, X outlets income down 11%
So, X outlets net income in 2013 was
=(100-11)X
=89X
In 2013,Y outlets income were up 7%
So, Y outlets net income in 2013 was
=(100+7)Y
=107Y
In 2013, X and Y outlets total revenue was
=(89X+107Y)
But,
2013 total revenue increased 1% from 2012 total revenue
=100(X+Y)+100(X+Y) of 1%
=101X +101Y
According to the question,
101X +101Y=89X+107Y
===================
Or, X:Y=1:2
Answer:1:2

67.Suppose you deposited Tk 10000 on January 1. 2012 at 12.50% interest rate for 1 year. On
July 1 2013 Tk15000 at 12% interest rate for 6 months and on October 1 2013 Tk 20000 at
11.5% interest rate for 3 months(assume that the stated interest rates are simple and annual).
Suppose you withdrew all deposits including due interests on December 31. 2013. Calculate
the overall annual rate of interest you have received.
[MTB MTO-2014][SBL MTO-2016]
Solution:(Collected)
#Investment_01:
•• Tk. 10000 from 01.01.2016 to 31.12.2016 @12.50%.
Total Interest Earned = Tk. 10000 × 12.50% × 12/12 = Tk. 1250.
Weighted average annual investment = Tk. 10000 × 12/12 = Tk. 10000.
#Investment_02:
•• Tk. 15000 from 01.07.2016 to 31.12.2016 @12%.
Total Interest Earned = Tk. 15000 × 12% × 6/12 = Tk. 900.
Weighted average annual investment = Tk. 15000 × 6/12 = Tk. 7500.

#Investment_03:
•• Tk. 20000 from 01.10.2016 to 31.12.2016 @11.50%.
Total Interest Earned = Tk. 20000 × 11.50% × 3/12 = Tk. 575.
Weighted average annual investment = Tk. 20000 × 3/12 = Tk. 5000.

ow:
◘ Total Interest Earned = Tk. 1250 + 900 + 575 = Tk. 2725.
◘ Weighted Average Annual Investment To Earn The Interest = Tk. 10000 + 7500 + 5000 =
Tk. 22500.

Therefore:
Overall Annual Rate of Interest = 2725/22500 = 0.121111.... = 12.11%.
(Solution By: Sultan Mahmud Cxlvii Bhai)

঑ :
( :খ ক )
৬০০ ক গক ...এ ক ক এক ঘ :
৩০০ ক গক
====================
১০০০০ ক ১২ গক ১০০০০ ক ১২ গক
১৫০০০ ক ৬ গক ৭৫০০ ক ১২ গক
২০০০০ ক ৩ গক ৫০০০ ক ১২ গক
====================
= [১০০০০*১২.৫০%] + [৭৫০০*১২.০০%] + [৫০০০*১১.৫০%] = ১২৫০+৯০০+৫৭৫ =
২৭২৫ ক
= ১০০০০+৭৫০০+৫০০০ = ২২৫০০ ক
====================
঑ ক
= ২৭২৫/২২৫০০ = ১২.১১%

«»«»MATH REVIEW SECOND PHASE«»«»


{Day-30}
¤ ::::::::::
খ ১০০ ক PDF ঑ ।

¤MATH REVIEW FIRST PHASE + MATH REVIEW SECOND PHASE ১৭০ ক


এ এ খ
¤ কএ ঑ ক
=======================================
68.A part time employee whose hourly wage was increased by 25 percent decided to reduce
the number of hours worked per week so that the employees total weekly income would
remain unchanged.By what percent should the number of hours work be reduced?
[ICB SO-2011]
Solution:
Let,
The Hourly wage =W
Initially working hours=H
Reduced working hour=H1
According to the question,
H*W =25% of W*H1
Or, H=1.25 H1
Or, H1=0.8H
The number of hours that should be reduced
={(1-0.8)/1}*100
=20%
And:20%
Solution-2:
Let,
Original hourly wages=10
Original working hours =10
Total wage the employee income before change his wage & Working time
=(10*10)
=100
New hourly wages
=10 +10 of 25%
=12.5
Reducing working hours=H
According to the question,
H*12.5=100
Or, H=8
So,
Working hour reduce
=10-8
=2
Required hours reduced
=(2/10)*100
=20%
Answer:20%

69.Four person each of home work at the same rate where to complete job according to a
schedule. However because of an agreement, two of them four quit after working one
day.The remaining person finished the job, but it required two more days, that what had
originally been scheduled.How many days were originally scheduled for completion of the
job ?
[DBBL-PO-2003]
Solution:
Let,
Originally time taken=X days
4 Men 1 day's work=1/X portion
Remaining work
=(1-1/X)
={(X-1)/X } portion
Remaining person=4-2=2
Remaining days
=(X-1+2)
=(X+1) days
1 man 1 day's work=1/4X
2 men (X+1) days work
=(X+1)/2X
According to the question,
(X+1)/2X={(X-1)/X}
===============
===============
Or, X=3
Schedule time originally was 3 days
And: 3 days

70.A dishonest merchant make a 15% profit at the time of buying and a 10% loss at the time
of selling the goods. By doing so if the said merchant made a profit of Tk. 3500 on a
particular item, what was the real cost of the item sold
[Agrani Bank Senior Officer-2017]
Solution-1:
Let,
Cost Price=X Tk
15% profit on cost price
Then market price=X + X of 15%
=1.15 X Tk
Again 10% loss on market price
Then Selling price
=1.15x -1.15x of 10%
=1.035x Tk
Now profits=Selling - Cost Price
=(1.035x - x) Tk
=0.035x Tk
When,
Profit.035x Tk then Cost Price=X Tk
--------3500Tk--------------------------
=[(3500*x)/.035x]
=1,00,000 Tk
Answer:1,00,000 Tk

Solution-2
Let, cost price = 100 Tk.
At 15% profit the market price of the product will be=100+15=115 Tk.
At 10% loss, the sales price will be=115-(115*10%)=Tk. 103.5 Tk.
So profit = 103.5-100= 3.5 Tk.
Now,
when profit 3.5 Tk. then cost 100 Tk.
when profit 3500 Tk. then cost (100/3.5)*3500=1,00,000 Tk.
So, real cost is Tk. 1,00,000 (Ans.)
71.ক এক ক ৩০ ,খ ১৫ এ গ ১০ ক ২ খএ ৩
গ,ক ক ক ঐক ক ?[BKB SO-2015][Agrani Bank SO-2017]
[A alone can do a piece of work in 30 days,while B alone can do it in 15 days & C alone can
do it in 10 days.If in every second day B and in every third day C help A in doing the
work,how many days will be required to complete the whole work?]
Solution:1
Let,
Total work=1 portion
LCM of 2 & 3=6
Per 6 days A work= 6 days
Per 6 days B work=(6/2)=3 days
Per 6 days C work=(6/3)=2 days
So,
6 days (A+B+C)'s work
=(6/30+3/15+2/10) portion
=3/5 portion
Remaining work
=(1-3/5) portion
=2/3 portion
After 2 days(A+B)'s work
=(2/30+1/15)portion
=2/15 portion
[Every second days B help A]
Remaining work
=(2/5-2/15) portion
=4/15 portion
Total time=(6+2)=8 days
9th day(A+C) work
=(1/30+1/10) portion
=2/15 portion
Remaining work
=(4/15-2/15)portion
=2/15 portion
Another 2 days(A+B)'s work
=(2/30+1/15) portion
=2/15 portion
Remaining work
=(2/15-2/15)
=0
So total time taken to finish the work=(9+2)=11 days
Answer:11 days
Solution:2==================
এখ ,
২঑৩এ =৬
৬ ক ক ক,খ,গ এক এ ৬ ক ক
=(৬/৩০)+(৩/১৫)+২/১০)
=৩/৫
ক ক =(১-৩/৫)
= ২/৫
২/৩<৩/৫
ক,খ,গ এক এ ৩ ক ক =(৩/৩০+১/১৫+১/১০)
=৪/১৫
ক ক =(২/৫-৪/১৫)
=২/১৫
২/১৫<৪/১৫
ক এক ১/৩০ ক ক
ক ক =(২/১৫-১/৩০)
=১/১০
এখ ক, খ এক এ ক ১/১০ ক
ক, খ এক এ ১ ক ক =(১/৩০+১/১৫)=১/১০ ক
ক ক :(১/১০-১/১০)=0

=৬+৩+১+১=১১
Answer :11 days

72.Two trains ,one from Dhaka and another from chittagong simultaneously started to
proceed towards each other at the speed of 16 km and 21 km per hour respectively.As the
trains met each other it was found that one train travelled 60 km more than the
other.Calculate the distance between Dhaka to Chittagong.
[Agrani Bank Senior Officer-2017]
Solution:
Let,
Two trains met each other 't' hrs later
First trair covered=(16*t) km
Second train covered=(21*t)km
Differences of distance two trains
=60 km
According to the question,
21t-16t=60
Or, 5t=60
Or, t=12
Total distance Dhaka to Chittagong
=(16*12+21*12)
=444 km
Answer:444 km

73.Sakib and Labib individually borrowed different amount of money from a particular bank
on the same day at rate of 20% simple interest.The total money paid by Sakib in 3 years as
principal plus interest was the same amount Labib paid in 2 years as principal plus
interest.Find the ratio of their individual loan amount.
[Agrani Bank Senior Officer-2017]
[Sonali Bank Senior Officer-2015]
Solution:
Sakib and Labib borrowed amount
X & Y Tk respectively
According to the question,
X+ X of 20%*3=Y+Y of 20%*2
Or,X+0.6 X =Y +0.4Y
Or, 1.6 X=1.4Y
Or, X:Y=1.4:1.6 or 7:8
Answer:7:8

74.The perimeter of a square is equal to the perimeter of rectangle. The lenght of the
rectangle is three times longer than its width having total area of 1200 sq.meter. what will be
the total cost cost if the total area of the square is covered with stones having a dimension of
50 centimeter square each & if tk.50 is charged for placing a stone in the square?
[Agrani Bank Senior Officer-2017]
[Sonali Bank Senior Officer-2015]
Solution:
Let,
Breadth of rectangle=x m
And length of Rectangle= 3x m
According to the question,
3x^2 = 1200
x= 20
Length of rectangle = 60m
Perimeter of rectangular
= 2(60+20)=160m
As per question,
Perimeter of rectangular= perimeter of squire = 160m
Side of square = 160/4
=40m or 4000 cm
area of square =(4000*4000)sq.cm
Square of stone
= (50)^2
= 2500 sq.cm
number of stones = (4000*4000)/2500=6400
Total cost = 6400*50=320000
Answer:3,20,000 Tk
NOTE:৫০ . গ গ এক ঘ ৫০ .
৫০ গ . এক গ ৫০ গ .

75.A cistern 6 meter long & 4 meter wide contains water up to a depth of 1 metre 25
centimeters.The total area of the wet surface is
[Agrani Bank Senior Officer-2017]
Solution:
Given that,
Length(l)=6 m
Width(b)=4 m
Depth(h)=1 m 25 cm=1.25 m
We know that,
Area of the wet surface
=[2(lb+bh+hl)-lb]
=[2(4*6+4*1.25+1.25*6)-4*6]
=73-24
=49 m^2
Answer:49 m^2
Note:
Wet surface:
**Wet surface এক ক ঑
** এক ৬ ক wet ঑ এ
঑ ।

76.Movenpick's ice-cream cones are of 12 cm in diameter and 18 cm height.To cope with the
price hike of the ingredients,the management planned to reduce the quality of ice-cream per
cone by adjusting the cone diameter and/or height.If they reduce diameter by 2 cm while
maintaining the height,how much ice-cream can they save per cone
[FSIBL PO-2016 MCQ]
Solution:
Give that,
Initial diameter of ice-cream=12 cm
So, Radius of ice-cream
R=12/2=6 cm
After 2 cm reduce diameter then new diameter=(12-2)=10 cm
New Radius r=10/2=5 cm
Total Ice-cream saved
=Initial Volume - Final Volume
=1/3 π(R^2)h - 1/3 π(r^2)h
=6π*(36 - 25)
=66 π
Answer:66π
:
ক ক ক
=১/৩ π(r^২)h
এ এ

77. Proof of identity of 115 people was verified. 65 of them had Passport, 30 of them had
both Passport and Voter ID. However, 15 of them could not produce any identity documents.
How many of them showed up only with Voter ID?
[BKB OFFICER-2017]
Solution:
Total people=115
Undocments people=15
So,
Documents people=(115-15)=100
Both voter ID and passport have
=30
Let,
Voter ID have=x people
Only passport have=(65-30)=35 people
Only voter ID have=(x-30) people
So,
Only passport or Voter ID or Both
=35+(x-30)+30
According to the question,
35+(x-30)+30=100
Or, x =65
So,
Only voter ID have=(65-30)=35
Answer: 35.

78. Rafiq, Shafiq, and Arif can alone complete a project in 10 days, 20 days, and 10 days
respectively. Rafiq started working on the project alone. Shafiq joined the project after 2
days. After working together for 4 days both Rafiq and Shafiq left the project and Arif got in.
How many days it took to complete the entire project?
[BKB OFFICER-2017]
Solution:
Rafiq can do in 1 day = 1/10 of the work
Shafiq can do in 1 day = 1/20 of the work
Arif can do in 1 day = 1/10 of the work

Rafiq first 2 days work done


=2/10 portion
=1/5 portion
Remaining work
=(1-1/5) portion
=4/5 portion
Both Rafiq & shafiq 4 days done
=(4/10)+(4/20)
=3/5 portion
Remaining work
=(4/5)-(3/5)
=1/5
Arif full work done in 10 days
Arid 1/5 portion work done
=10*1/5
=2 days
So, Total time taken to complete the whole work
=(2+4+2)=8 days

Answer: 8 days.

79. Mr. Rashid has saved Taka 1200 from his first month's salary. He plans to increase his
monthly savings by Taka 100 in every following month. How much time would it take to
save Taka 106200?
[BKB OFFICER-2017][36th BCS]
Solution:
Let,
'n' months be required to save Tk 106200.

Here,
the savigns in the first month,
a = 1200 Tk
Increase in savings in each month, d = 100 Tk

According to the question:


n/2{2a+(n-1)*d}=106200
Or,(n/2){2×1200 + 100(n-1)} = 106200
Or,n(1200 + 50n - 50) = 106200
Or,n² + 23n = 2124
Or,n² + 59n - 36n - 2124 = 0
Or, (n+59)(n-36) = 0
Either,
n = 36
Or,
n=-59[It is not accepted]

Ans: 36 months or 3 years.

80. Solve the equation : 2{(x+3)/(x-3)}² - 7{(x+3)/(x-3)} + 6 = 0


[BKB OFFICER-2017]
Solution:
Suppose,
(x+3)/(x-3) = P---------(1)
So,
2{(x+3)/(x-3)}² - 7{(x+3)/(x-3)} + 6 = 0
Or, 2p² - 7p + 6 = 0
Or, 2p² - 4p - 3p + 6 = 0
Or, 2p(p-2) - 3(p-2) = 0
Or, (p-2)(2p-3) = 0

So, either
P-2=0
(x+3)/(x-3) = 2
2x - 6 = x + 3
x=9
or,
2p-3=0
(x+3)/(x-3) = 3/2
3x - 9 = 2x + 6
x = 15

Ans: 9 or 15.

81. The area of a rectangle is 1200 square meter. If the length of the rectangle is reduced by
10 meters, it becomes a square. Calculate the length of the diagonal of the rectangle. [BKB
OFFICER-2017]
Solution:
Let,
Length of the rectangle is=x
Width of the rectangle is=y
If the length of the rectangle is reduced by 10 meters, it becomes a square.
y=x-10--------(1)

We know that,
xy = 1200
Or, x (x-10)=1200
x² - 10x - 1200 = 0
(x-40)(x+30) = 0
Either,
x = -30 [It's not accepted]
Or,
x-40=0
Or, x=40

So, the width of the rectangle is

=40-10= 30 meters,
and
the length of the rectangle is = 40 meters.

So, the length of the diagonal of the rectangle is = √(40²+30²) = 50 meters.

Answer: 50 meters

82. A Rhombus has an area of 120 square meters and the length of its one diagonal is 10
meters. Calculate the perimeter of the Rhombus.
[BKB OFFICER-2017]
Solution:
Let
The length of a side of the rhombus is = a meters,
and that of the other diagonal is
=d meters.

We know that,
area of a rhombus =1/2* (product of the lengths of the diagonals)
Or, 120 = (10×d)/2
Or, d = 24 meters.

Again, We know:
a² = (24/2)² + (10/2)²
a² = 169
a = 13

So,
the perimeter of the rhombus is = 4a = 4×13 = 52 meters.

Answer: 52 meters.

83. The area of an equilateral triangle is 3√3 square meter. Determine the area of a circe
inscribed within the triangle.
[BKB OFFICER-2017]
Solution:
Area of equilateral triangle = (√3/4)a²
So, (√3/4)a² = 3√3
=> a² = 12
=> a = 2√3
So, Semi-Perimeter of triangle = 3*2√3/2 = 3√3
Now, Radius* Semi-perimeter = Area
=> Radius = 3√3/3√3 = 1
Then, Area of circle = πr² = π*1² = π or 3.14 sqm (Ans.)

84:In first 1000 natural numbers,how many integers exist such that they leave a reminder 4
when divided by 7 and a reminder 9 when divided by 11?
[BHBFC SO-2017]
Solution:
Let,
Integer be X & Quotient be q
When integer X is divided by 7 , it leaves a remainder of 4
X=7q +4
X can take values 4 , 11 , 18 , 25 , 32 , 39 , 46 , 53 , 60
When integer X is divided by 11 , it leaves a remainder of 9
X= 11p + 9
X can take values 9 , 20 , 31 , 42 , 53 , 64
The first integer which fulfills the given criteria is 53 .
Similarly,we will get the next such after an interval of LCM of 7 and 11,that is 77
The numbers are
53 ,
53+77*1=130
53+77*2=207
53+77*3=284
==========
==========
77*12 + 53=977

Number of such integers


= {(977-53)/77}+1
=13
Ans:13

85.A sum of Rs. 725 is lent in the beginning of a year at a certain rate of interest. After 8
months, a sum of Rs. 362.50 more is lent but at the rate twice the former. At the end of the
year, Rs. 33.50 is earned as interest from both the loans. What was the original rate of
interest?
{Very Important}
Solution:
Let,
Original rate =X
After 8 months rate=2X
According to the question,
{(725*x*1)/100}+{(362.50*2x*4/12)/100}=33.50
==========================
X=3.46%
So, the original rate is 3.46%
Answer:3.46%

86.A financier claims to be lending money at simple interest, But he includes the interest
every six months for calculating the principal. If he is charging an interest of 10%, the
effective rate of interest becomes
{Very Important}
Solution:
Let,
Lending money Tk. 100
Interest for 1st 6 months
=100*1/2*10%
=5 tk
Next 6 month balance=105 tk
Interest for the next 6 months=105*1/2*10%=5.25

So,
Effective interest rate
=(5+5.25)=10.25%
Answer:10.25%

87.A does a work in 45 days whereas B does the same work in 36 days. A starts the work and
was joined by B after 18 days. If A gets total 2350 as his wage, find the money B will get?
Solution:
Traditional way/Exam Approach:
A's 18 day's work=18/45=2/5
Both (A+B)'s 1 day work
=(1/45)+(1/36)
=1/20 portion
Both Full work done
=(1*20*3)/5
=12 days
A 12 days work
=12/45=4/15 portion
B's12 days work
=12/36
=1/3
Ratio of work A & B
=(2/5 +4/15):1/3
=2:1
So,
2 portion work for paid =2350 tk
1 portion paid for
=(2350/2)=1175 tk
So, B will get =1175 tk
Answer:1175 tk
Smart or Faster method:
Let,
Total work is 180 units
[LCM of 45 & 36]
A's working efficiency per day
=180/45
=4 units
B's working efficiency per day
=180/36
=5 units
A's 18 day work done
=18*4=72 units
Remaining work
=(180-72)
=108 units
A & B 1 day done=(4+5)=9 unit
A & B 108 unit done
=108/9=12 days
A 12 days done=12*4=48 unit
B ---------:------------=12*5=60 unit
A & B working portion ratio
=(72+48):60
=120:60
=2:1
So, B's got =2350/2
=Tk.1175
Answer:1175 tk

88.Two trains A and B start from X and Y towards Y and Xrespectively. After passing each
other,they take 4 hours 48 minutes and 3 hours 20 minutes to reach Y and X respectively.If
train is moving at 45 km/hr, then speed train B is?
{Very Important}
Solution:
Let,
Speed of B train=Z
Give that,
4 hrs 48 m=24/5 hrs
3 hrs 20 m=10/3 hrs
After they meet the first train travelled=(45*24/5)=216 km
[Formula:Distance=Time*Speed ]
And
Second train travelled
=Z*10/3
=10Z/3 km
::
Now,Before they meet second train travelled/cover the distance 216 km with speed Z km/h
and First train cover 10Z/3 km with the speed 45 km/h
::
According to the question,
216/Z ={(10Z/3)/45}
Or, 216/Z=10Z/45*3
Or, 10*Z^2=45*3*216
Or, Z^2=2916
Or, Z=54
The speed of the second train is 54 km/h
Answer:54 km/h

89. A bank offers 5% compound interest calculated on half-yearly basis. A customer deposits
Tk.1600 each on 1st January and 1st July of a year. At the end of the year, the amount he
would have gained by way of interest is:
Answer:121 tk

90.A box contains only marbles. If 1/4th of the marble were removed from the box would be
filled to 1/3 of its capacity.If instead of 100 marbles were added,the box would be full.How
many marbles are there in the box?
[Jamuna Bank MTO/PO-2017]

Solution:
Let,
Total capacity of the box=1 portion

The marble in the box begging=x

First1/4 th of the marble removed from the box


=x*1/4 portion
=X/4 portion
Remaining marbles in the box
={x-(x/4)}
=3x/4 portion

According to the question,


1/3 of the capacity of box =3x/4 marbles
Full capacity of the box =9x/4 marbles
::
Now ,100 marbles were added to the "x" marbles then the box full its capacity
So,
x+100=9x/4
Or, (9x/4)-x =100
Or, 5x =400
Or, x=80
Hence,The marble in the box 80
Answer:80
91.A box only contains marbles. If you remove 1/4th of the marble from the box, then 1/3rd
of the box is empty. Now if you added 100 marbles, the box will be full. How many marbles
were there in the box?

Solution:
Total capacity of the box=1 portion
Let,
The marble in the box begging=x
1/4 th of the marble removed =x*1/4 portion
=X/4 portion
Remaining marble
={x-(x/4)}
=3x/4 portion
According to the question,
(1-1/3)=2/3 of the capacity of box =3x/4 marbles
Full capacity of the box =9x/8 marbles
::
Now ,100 marbles were added to the "x" marbles then the box full its capacity
So,
x+100=9x/8
Or, (9x/8)-x =100
Or, x/8 =100
Or, x=800
Hence,The marble in the box 800
Answer:800

91:While out on picnic,a group of boys came upon an apple tree.One of the boys climbed up
tree and picked enough apples for each boy to have three,with none left over,the along with
came three boys,making it impossible to divide the picked apples evenly. However, after
picking one more apple and adding it to the total, every boy had two apples with none left
over.How many apples were finally divided ?
[Meghan Bank -MTO-2017]
Solution:
Let,
The total boys were in the picnic=X
and
Picked up total apples from tree=Y
Form first condition,
Y/X =3 ---------(1)
Second condition,
(Y+1)/(X+3)=2 ----------(2)

From equation no (1)=»


Y=3X-------(3)
Putting the value Y=3X in equation (2)
we get X=5
From equation(3)=»
Y=15
Hence,The total apples picked up from the tree 15
But (15+1)=16 apples were distributed among the boys then every boys got Two apple
Answer:16
92:Mr. Zakir gave 40 % of the money he had, to his wife.He also gave 20% of the remaining
amount to each of his three sons.Half of the amount now left was spent on miscellaneous
items and the remaining amount of tk 12000 was deposited in the bank .How much did Mr.
Zakir Have initially ?
[Meghan Bank -MTO-2017]
Solution:
Let,
Mr Zakir's total initially amount was= tk x
He gave his wife
= 40% of x
=2x/5 tk
Mr Zakir's remaining amount
={x-(2x/5)}
=3x/5 tk
Each son got=20%of 3x/5
=3x/25
So three son got
={3*3x/25}
=9x/25
Remaining amount is
(3x/5-9x/25)
=6x/25
According to the question,
(6x/25)*1/2=12000
Or, X=100,000

Answer:100,000 Tk

93.If 7% of the sale price of a product equal to 8% of cost price and 9% of the sale price
exceeds 10% of the cost price by Tk 1, find the amount of profit and cost of the product?
[Dhaka BANK-MTO-2017]
Solution:
Let,
Selling price be =X Tk
And
Cost price be =Y Tk
First condition,
7% of X =8% of Y
7x/100=8y/100
Or, x =8y/7-----------:--(1)
Second condition,
9%x -10%y=1
Or, (9X/100)--(10Y/100)=1
======================
======================
Or, Y =350
From equations no(1)
X =(8*350)/7
Or, X =400
So,
Selling price of the product is 400 Tk
Profit=(Selling Price--Cost price)
=(400-350)=50 Tk
Answer: Tk.350 & Tk. 50
:::::
::::
94.Mr Amin invests Tk 24000 in Dhaka Bank at 7.50%. How much additional money must
he invest at 10% so that the total income will be equal to 9.25% of his entire investment?
[Dhaka BANK-MTO-2017]
Solution:
Let,
He investment additional amount =x
According to the question,
24000*7.50%+x * 10%= (24000+ x) 9.25%
Or,[(24000*7.50)/100] +10x/100=
[(24000+x)*9.25]/100
Or, 180000 +10x =222000+9.25x
Or, 10x -9.25x =222000-180000
Or, 0.75 x =42000
Or, x=56000
So, He invest additional amount is
56000 Tk
Answer:56000Tk
:::::::
:::::::
95.Find the value of x^4+1/x^4 ; If x=√5-√4
[Dhaka Bank-MTO-2017]
Solution:
Given that,
x=√5-√4----------(1)
1/x= 1/(√5-√4) [Reverse both side]
Or, 1/x= (√5+√4)/[(√5-√4)(√5-√4)]
Or, 1/x=√5+√4---------(2)
From (1)&(2)
x+(1/x)=√5-√4+√5+√4=2√5

Now,
x^4+1/x^4
=(x^2)^2+(1/x^2)^2
= (x^2+1/x^2)^2-2*x^2*(1/x^2)
= (x^2+1/x^2)^2-2
= [(x+1/x)^2-2*x*1/x]^2-2
=[(2√5)^2-2]^2-2
=324-2
=322
Answer: 322
::::::
::::::
96.{3/(x+1)}+{6/(2x+1)}={18/(3x+1)}
[Dhaka Bank-MTO-2017]

Answer: -(3/5)
{Try yourself}

97.In an organization, 30% of the employees live over 10 miles from the work. 60% of
employees who live who live over 10 miles from work,use company transport. If 40% of
employees of the company use company transport, what percentage of the employees live 10
miles or less from work and use company transport?
[Bangladesh Bank AD-2001]
[Estarn Bank MTO-2007]

Solution:
Math Fact:
[Note: ক ক ক ১০ এ ক ক
৭০ ক X/ ১০০ ক ।]
Let,
Total employee be=x

Live 10 miles over employee be


=x of 30%
=3x/10

Live 10 miles or less employee


=(x-3x/10)
=7x/10

Employees Live ove 10 miles and using company transport


=3x/10 of 60%
=9x/50

Total number of employees using company transport


=x of 40%
=2x/5

Employees using transport and live 10 miles or less from work


=(2x/5 - 9x/50)
=11x/50

Among x employees using transport and live 10 miles or less from work=11x/50 employees
So,
100 employees using transport and live 10 miles or less from work
=[(11x * 100)/50*x]
=22%
Answer:22%

Solution-2:
Let,
.
.
The total number of employees be=100

Employees live10 miles or less from work=100-30=70

Employees live over 10 miles and using transport=60% of 30=18

Total number of employees using transport=40% of100=40


Employees using transport and live 10 miles or less from work
=40-18=22

Answer:22%

98.এক এক ঑ ১০% ক ৬৩০ ক


, ক ?
[Sonali Bank officer -2014]
[Same As SEBL-2015]
Solution-1:
The market price of 1 unit goods be Tk x
So, 100 units of goods =100 x Tk
But ,at the time of buying,the merchant paid 100x Tk and received
=(100 + 100 of 10%)
=110
So, the real cost of 110 units to the merchant
=100 x Tk
At the time of selling,the merchant charged
=110x/100
So,
Sold of 110 units
=110*(110x/100)
=121x Tk
Net profit,
=121x-100x
=21x Tk
According to the question,
21x=630
Or, x =30 Tk
So,
The real cost price of the product
=100*30
=3000 Tk
Answer:3000 Tk

Solution-2:
Let,
Cost Price=X Tk
10% profit on cost price Then market price
=X + X of 10%
=1.1 X Tk
Again 10% profits on market price
Then Selling price
=1.1x + 1.1x of 10%
=1.21x Tk
Now profits=Selling - Cost Price
=(1.21x - x) Tk
=0.21x Tk
When,
Profit .21x Tk then Cost Price=X Tk
--------630 Tk--------------------------
=[(630*x)/.21x]
=3000 Tk

Solution-3:
Let,
Cost Price of the article=100 Tk
10% profit on cost price then
Market price=(100 + 100 of 10%)
=110 Tk
Again,
10% profits on market price then selling price=(110+110 of 10%) Tk
=121 Tk
Now,
Profit=(121-100)=21 Tk
When
Profit 21 Tk then Cost Price=100 tk
--------630 Tk -----------------------------------
=[(100*630)/21]
=3000 Tk
Answer:3000 Tk

Solution-4:====
Short Cut/MCQ Way
Let , Cost Price=100 Tk
10% profits on both way
Profit
=10+10+[(10*100)/100]
=21 Tk
Profit 21 then CP =100 TK
-------630 TK --------=[100*630/21]
=3000 TK
Answer:3000 Tk

99.ক এক ক ৩০ ,খ ১৫ এ গ ১০ ক ২ খএ ৩
গ,ক ক ক ঐক ক ?[BKB SO-2015]
[A can do a piece of work in 30 days,B can do it in 15 days and C can do it in 10 days.Every
second days B and every third days C help A .How many days will finish the whole work?]
[Agrani BANK SO-2017]
[Bangladesh Krishi Bank SO-2015]
More:Same As
**34th BCS WRITTEN
**33th BCS WRITTEN
**21th BCS WRITTEN
**PSC Non cader
Precaution:=========
Most of the Guide Book this question answer is wrong
Such as 12/10 etc
Correct Answer:11 Days
Solution:1
Let,
Total work=1 portion
LCM of 2 & 3=6
Per 6 days A work= 6 days
Per 6 days B work=(6/2)=3 days
Per 6 days C work=(6/3)=2 days
So,
6 days (A+B+C)'s work
=(6/30+3/15+2/10) portion
=3/5 portion
Remaining work
=(1-3/5) portion
=2/3 portion
After 2 days(A+B)'s work
=(2/30+1/15)portion
=2/15 portion
[Every second days B help A]
Remaining work
=(2/5-2/15) portion
=4/15 portion
Total time=(6+2)=8 days
9th day(A+C) work
=(1/30+1/10) portion
=2/15 portion
Remaining work
=(4/15-2/15)portion
=2/15 portion
Another 2 days(A+B)'s work
=(2/30+1/15) portion
=2/15 portion
Remaining work
=(2/15-2/15)
=0
So total time taken to finish the work=(9+2)=11 days
Answer:11 days
Solution:2==================
এখ ,
২঑৩এ =৬
৬ ক ক ক,খ,গ এক এ ৬ ক ক
=(৬/৩০)+(৩/১৫)+২/১০)
=৩/৫
ক ক =(১-৩/৫)
= ২/৫
২/৩<৩/৫
ক,খ,গ এক এ ৩ ক ক =(৩/৩০+১/১৫+১/১০)
=৪/১৫
ক ক =(২/৫-৪/১৫)
=২/১৫
২/১৫<৪/১৫
ক এক ১/৩০ ক ক
ক ক =(২/১৫-১/৩০)
=১/১০
এখ ক, খ এক এ ক ১/১০ ক
ক, খ এক এ ১ ক ক =(১/৩০+১/১৫)=১/১০ ক
ক ক :(১/১০-১/১০)=0

=৬+৩+১+১=১১
Answer :11 days

100.In a flight of 600 km ,an aircraft was slowed down due to bad weather.Its average speed
for the trip was reduced by 200 km/hr and the time of flight increased by 30 minutes.The
duration of the flight is.[Indian Bix]
[Rupali Bank Written-2013]
Answer Type
A.1 hour B.2 hrs
C.3. hrs D.4 hrs
Answer:1 hour
**Most of JoB Guide the answer is
(60+30) minutes=1.5 hrs which is incorrect
-
*Duration of the flight always fixed
* ক কঘ ড় ৎ গ

Solution:1==================
Let,
The duration of the flight be=x hrs
Original distance be=600 km
According to the question,
Original speed-Reduced speed=200
600/x-600/(x+1/2)=200
Or,600/x-1200/(2x+1)=200
Or,2/x-{6/(2x+1)}=1
Or,(6x+3-6x)/{x(2x+1)}=1
Or,2x^2+3x-2x-3=0
Or,x(2x+3)-1(2x-3)=0
Or,(2x+3)(x-1)=0
Now,
X-1=0
Or,X=1
And
2x+3=0
Or,X=-(3/2)
[neglecting the negative value]
Answer:1 hour
============================
Solution:2
Let,
Original time be= t hrs
Original speed be=S km/hrs
Here,Distance (D)=600 km
According to the first condition,
D=st
Or,s=D/t
Or,s=600/t=======(1)
According to the 2nd condition,
600=(s-200)*(1/2+t)
{(Putting value s=600/t) 2nd condition}
======================
======================
======================
Or,(2t+3)(t-1)=0
Now,
2t+3=0
Or,t=-(3/2)
[neglecting Negative value]
And
t-1=0
Or,t=1
So Duration of the flight 1 hour
Answer:1 hour

Section-D: Confusion Math


Question-1:In an organization, 30% of the employees live over 10 miles from the work. 60%
of employees who live who live over 10 miles from work,use company transport. If 40% of
employees of the company use company transport, what percentage of the employees live 10
miles or less from work and use company transport?
[Bangladesh Bank AD-2001]
[Estarn Bank MTO-2007]

Solution:
Math Fact:
[Note: ক ক ক ১০ এ ক ক
৭০ ক X/ ১০০ ক ]
Let,
Total employee be=x

Live 10 miles over employee be


=x of 30%
=3x/10
Live 10 miles or less employee
=(x-3x/10)
=7x/10

Employees Live ove 10 miles and using company transport


=3x/10 of 60%
=9x/50

Total number of employees using company transport


=x of 40%
=2x/5

Employees using transport and live 10 miles or less from work


=(2x/5 - 9x/50)
=11x/50

Among x employees using transport and live 10 miles or less from work=11x/50 employees
So,
100 employees using transport and live 10 miles or less from work
=[(11x * 100)/50*x]
=22%
Answer:22%

Solution-2:
Let,
.
.
The total number of employees be=100

Employees live10 miles or less from work=100-30=70

Employees live over 10 miles and using transport=60% of 30=18

Total number of employees using transport=40% of100=40

Employees using transport and live 10 miles or less from work


=40-18=22

Answer:22%

Question-2 :এক এক ঑ ১০% ক


৬৩০ ক , ক ?
[Sonali Bank officer -2014]
[Same As SEBL-2015]
Solution-1:
The market price of 1 unit goods be Tk x
So, 100 units of goods =100 x Tk
But ,at the time of buying,the merchant paid 100x Tk and received
=(100 + 100 of 10%)
=110
So, the real cost of 110 units to the merchant
=100 x Tk
At the time of selling,the merchant charged
=110x/100
So,
Sold of 110 units
=110*(110x/100)
=121x Tk
Net profit,
=121x-100x
=21x Tk
According to the question,
21x=630
Or, x =30 Tk
So,
The real cost price of the product
=100*30
=3000 Tk
Answer:3000 Tk

Solution-2:
Let,
Cost Price=X Tk
10% profit on cost price Then market price
=X + X of 10%
=1.1 X Tk
Again 10% profits on market price
Then Selling price
=1.1x + 1.1x of 10%
=1.21x Tk
Now profits=Selling - Cost Price
=(1.21x - x) Tk
=0.21x Tk
When,
Profit .21x Tk then Cost Price=X Tk
--------630 Tk--------------------------
=[(630*x)/.21x]
=3000 Tk

Solution-3:
Let,
Cost Price of the article=100 Tk
10% profit on cost price then
Market price=(100 + 100 of 10%)
=110 Tk
Again,
10% profits on market price then selling price=(110+110 of 10%) Tk
=121 Tk
Now,
Profit=(121-100)=21 Tk
When
Profit 21 Tk then Cost Price=100 tk
--------630 Tk -----------------------------------
=[(100*630)/21]
=3000 Tk
Answer:3000 Tk

Solution-4:====
Short Cut/MCQ Way
Let , Cost Price=100 Tk
10% profits on both way
Profit
=10+10+[(10*100)/100]
=21 Tk
Profit 21 then CP =100 TK
-------630 TK --------=[100*630/21]
=3000 TK
Answer:3000 Tk

Question-3:ক এক ক ৩০ ,খ ১৫ এ গ ১০ ক ২ খএ
৩ গ,ক ক ক ঐক ক ?[BKB SO-2015]
[A can do a piece of work in 30 days,B can do it in 15 days and C can do it in 10 days.Every
second days B and every third days C help A .How many days will finish the whole work?]
[Agrani BANK SO-2017]
[Bangladesh Krishi Bank SO-2015]
More:Same As
**34th BCS WRITTEN
**33th BCS WRITTEN
**21th BCS WRITTEN
**PSC Non cader
Precaution:=========
Most of the Guide Book this question answer is wrong
Such as 12/10 etc
Correct Answer:11 Days
Solution:1
Let,
Total work=1 portion
LCM of 2 & 3=6
Per 6 days A work= 6 days
Per 6 days B work=(6/2)=3 days
Per 6 days C work=(6/3)=2 days
So,
6 days (A+B+C)'s work
=(6/30+3/15+2/10) portion
=3/5 portion
Remaining work
=(1-3/5) portion
=2/3 portion
After 2 days(A+B)'s work
=(2/30+1/15)portion
=2/15 portion
[Every second days B help A]
Remaining work
=(2/5-2/15) portion
=4/15 portion
Total time=(6+2)=8 days
9th day(A+C) work
=(1/30+1/10) portion
=2/15 portion
Remaining work
=(4/15-2/15)portion
=2/15 portion
Another 2 days(A+B)'s work
=(2/30+1/15) portion
=2/15 portion
Remaining work
=(2/15-2/15)
=0
So total time taken to finish the work=(9+2)=11 days
Answer:11 days
Solution:2==================
এখ ,
২঑৩এ =৬
৬ ক ক ক,খ,গ এক এ ৬ ক ক
=(৬/৩০)+(৩/১৫)+২/১০)
=৩/৫
ক ক =(১-৩/৫)
= ২/৫
২/৩<৩/৫
ক,খ,গ এক এ ৩ ক ক =(৩/৩০+১/১৫+১/১০)
=৪/১৫
ক ক =(২/৫-৪/১৫)
=২/১৫
২/১৫<৪/১৫
ক এক ১/৩০ ক ক
ক ক =(২/১৫-১/৩০)
=১/১০
এখ ক, খ এক এ ক ১/১০ ক
ক, খ এক এ ১ ক ক =(১/৩০+১/১৫)=১/১০ ক
ক ক :(১/১০-১/১০)=0

=৬+৩+১+১=১১
Answer :11 days
Question-4: In a flight of 600 km ,an aircraft was slowed down due to bad weather.Its
average speed for the trip was reduced by 200 km/hr and the time of flight increased by 30
minutes.The duration of the flight is.[Indian Bix]
[Rupali Bank Written-2013]
Answer Type
A.1 hour B.2 hrs
C.3. hrs D.4 hrs
Answer:1 hour
**Most of JoB Guide the answer is
(60+30) minutes=1.5 hrs which is incorrect
-
*Duration of the flight always fixed
* ক কঘ ড় ৎ গ

Solution:1==================
Let,
The duration of the flight be=x hrs
Original distance be=600 km
According to the question,
Original speed-Reduced speed=200
600/x-600/(x+1/2)=200
Or,600/x-1200/(2x+1)=200
Or,2/x-{6/(2x+1)}=1
Or,(6x+3-6x)/{x(2x+1)}=1
Or,2x^2+3x-2x-3=0
Or,x(2x+3)-1(2x-3)=0
Or,(2x+3)(x-1)=0
Now,
X-1=0
Or,X=1
And
2x+3=0
Or,X=-(3/2)
[neglecting the negative value]
Answer:1 hour
============================
Solution:2
Let,
Original time be= t hrs
Original speed be=S km/hrs
Here,Distance (D)=600 km
According to the first condition,
D=st
Or,s=D/t
Or,s=600/t=======(1)
According to the 2nd condition,
600=(s-200)*(1/2+t)
{(Putting value s=600/t) 2nd condition}
======================
======================
======================
Or,(2t+3)(t-1)=0
Now,
2t+3=0
Or,t=-(3/2)
[neglecting Negative value]
And
t-1=0
Or,t=1
So Duration of the flight 1 hour
Answer:1 hour

Section-E: All Written Math-2017


Question :An amount of Tk. 7200 is spent to cover the floor of a room by carpet. An amount
of Tk. 576 would be saved if the breadth were 3 meters less. What is the breadth of the room?
[BDBL SO-2017]
(এক ক ক ড় ৭২০০ ক খ ক ৩ ক খ
৫৭৬ ক ক ক ?)
:
ক ,
ঘ x .
y
গ খ z ক
,
xyz=৭২০০--------(১)
xz(y-৩)=৭২০০-৫৭৬
=৬৬২৪-----(২)

ক (১) ক (২) গ ক =>

xyz/xz(y-৩)=৭২০০/৬৬২৪

, y=৩৭.৫

:৩৭.৫

:
৫৭৬ ক ক খ ৩ এ
১ ক ক খ ৩÷৫৭৬
৭২০০ ক ক খ (৩*৭২০০)÷৫৭৬
=৩৭.৫
:৩৭.৫
#New_Year_Celebration_2018

#Three Written Math Solution


::
::
Question-1:The cost of raw materials of a product increases by 30%, the manufacturing cost
increases by 20% and the selling price of the product increases by 60%. The raw material and
the manufacturing cost originally formed 40% and 60% of the total cost respectively. If the
original profit percentage was one-fourth the original manufacturing cost, find the new profit
percentage.

Solution:
Let,
The total initially cost of the product =tk.100
From Question condition,
Raw material cost =100*40%=40 tk
Manufacturing cost=100*60%=60 tk
Original Selling price=SP+Profit
=100+60*1/4=115 tk
Now,
New raw material cost
=40 +40*30%=52 tk
New manufacturing
=60+60*20=72 tk
New total cost=52+72=124. Tk
New selling price=115+115*60%=184 tk
New profit =184-124=60 tk

Required New percent


=60*100/124
=48.39%
Answer:48.39%

Question-2:A pipe can fill up an empty tank in 12 minutes. Another pipe flows out 14 litre
per minute. If the two pipes are opened together and the empty tank is filled up in 96 minutes,
how much water does the tank contain??

Let,
Water flow out x litre per minuter and
Total Water contains y litre in tank
This,
From Question condition,
y=12x..............(1)
Again when opened two pipe in the same time then the tank fill it in 96 mintues...
So,
y= 96x - 96*14...........(2)
From equation (I) and (ii) we get
96x - 96*14=12x
or, 84x=96*14
or, x=16
Putting the value in equation (I) we get,
y=12*16=192

The tank contains 192 litres of water


Ans:192 litres

Question-3:A number when divided by a divisor, leaves a remainder of 24. When twice the
number is divided by the same divisor, the remainder is 11. What is the divisor??
Solution:
Let,
The original number be 'N'
Let, the divisor be 'd'

Let the quotient of the division of a by d be 'x'

Therefore, we can write the relation

As,
N/d=x
and the remainder is 24.

So, N=dx+24----------(1)

When twice the original number is divided by d, 2N is divided by d.


We know that N=dx+24. Therefore, 2N=2dx+48

The problem states that


{(2dx+48)/d}

leaves a remainder of 11.


2dx is perfectly divisible by d
and will therefore, not leave a remainder.

The remainder of 11 was obtained by dividing 48 by d.


When 48 is divided by 37, the remainder that one will obtain is 11.
Hence, the divisor is 37

Ans:37

Question: A box contains only marbles. If 1/4th of the marble were removed from the box
would be filled to 1/3 of its capacity.If instead of 100 marbles were added,the box would be
full.How many marbles are there in the box?
[Jamuna Bank MTO/PO-2017]

Solution:
Let,
Total capacity of the box=1 portion

The marble in the box begging=x


First1/4 th of the marble removed from the box
=x*1/4 portion
=X/4 portion
Remaining marbles in the box
={x-(x/4)}
=3x/4 portion

According to the question,


1/3 of the capacity of box =3x/4 marbles
Full capacity of the box =9x/4 marbles
::
Now ,100 marbles were added to the "x" marbles then the box full its capacity
So,
x+100=9x/4
Or, (9x/4)-x =100
Or, 5x =400
Or, x=80
Hence,The marble in the box 80
Answer:80

* এ :
Question:A box only contains marbles. If you remove 1/4th of the marble from the box, then
1/3rd of the box is empty. Now if you added 100 marbles, the box will be full. How many
marbles were there in the box?

Solution:
Total capacity of the box=1 portion
Let,
The marble in the box begging=x
1/4 th of the marble removed =x*1/4 portion
=X/4 portion
Remaining marble
={x-(x/4)}
=3x/4 portion
According to the question,
(1-1/3)=2/3 of the capacity of box =3x/4 marbles
Full capacity of the box =9x/8 marbles
::
Now ,100 marbles were added to the "x" marbles then the box full its capacity
So,
x+100=9x/8
Or, (9x/8)-x =100
Or, x/8 =100
Or, x=800
Hence,The marble in the box 800
Answer:800

¤Meghan Bank -MTO-2017


¤Date:27.10.2017
=======================================
Question-1:While out on picnic,a group of boys came upon an apple tree.One of the boys
climbed up tree and picked enough apples for each boy to have three,with none left over,the
along with came three boys,making it impossible to divide the picked apples evenly.
However, after picking one more apple and adding it to the total, every boy had two apples
with none left over.How many apples were finally divided ?
Solution:
Let,
The total boys were in the picnic=X
and
Picked up total apples from tree=Y
Form first condition,
Y/X =3 ---------(1)
Second condition,
(Y+1)/(X+3)=2 ----------(2)

From equation no (1)=»


Y=3X-------(3)
Putting the value Y=3X in equation (2)
we get X=5
From equation(3)=»
Y=15
Hence,The total apples picked up from the tree 15
But (15+1)=16 apples were distributed among the boys then every boys got Two apple
Answer:16

Question-2:Mr. Zakir gave 40 % of the money he had, to his wife.He also gave 20% of the
remaining amount to each of his three sons.Half of the amount now left was spent on
miscellaneous items and the remaining amount of tk 12000 was deposited in the bank .How
much did Mr. Zakir Have initially ?
Solution:
Let,
Mr Zakir's total initially amount was= tk x
He gave his wife
= 40% of x
=2x/5 tk
Mr Zakir's remaining amount
={x-(2x/5)}
=3x/5 tk
Each son got=20%of 3x/5
=3x/25
So three son got
={3*3x/25}
=9x/25
Remaining amount is
(3x/5-9x/25)
=6x/25
According to the question,
(6x/25)*1/2=12000
Or, X=100,000

Answer:100,000 Tk

«»«»Written MATH Solution-2017«»«»


¤Dhaka BANK-MTO-2017
¤Date:29.09.2017
===========================================
Question-1:If 7% of the sale price of a product equal to 8% of cost price and 9% of the sale
price exceeds 10% of the cost price by Tk 1, find the amount of profit and cost of the
product?
[Dhaka BANK-MTO-2017]
Solution:
Let,
Selling price be =X Tk
And
Cost price be =Y Tk
First condition,
7% of X =8% of Y
7x/100=8y/100
Or, x =8y/7-----------:--(1)
Second condition,
9%x -10%y=1
Or, (9X/100)--(10Y/100)=1
======================
======================
Or, Y =350
From equations no(1)
X =(8*350)/7
Or, X =400
So,
Selling price of the product is 400 Tk
Profit=(Selling Price--Cost price)
=(400-350)=50 Tk
Answer: Tk.350 & Tk. 50
:::::
::::
Question-2:Mr Amin invests Tk 24000 in Dhaka Bank at 7.50%. How much additional
money must he invest at 10% so that the total income will be equal to 9.25% of his entire
investment?
[Dhaka BANK-MTO-2017]
Solution:
Let,
He investment additional amount =x
According to the question,
24000*7.50%+x * 10%= (24000+ x) 9.25%
Or,[(24000*7.50)/100] +10x/100=
[(24000+x)*9.25]/100
Or, 180000 +10x =222000+9.25x
Or, 10x -9.25x =222000-180000
Or, 0.75 x =42000
Or, x=56000
So, He invest additional amount is
56000 Tk
Answer:56000Tk
:::::::
:::::::
Question-3:Find the value of x^4+1/x^4 ; If x=√5-√4
[Dhaka Bank-MTO-2017]
Solution:
Given that,
x=√5-√4----------(1)
1/x= 1/(√5-√4) [Reverse both side]
Or, 1/x= (√5+√4)/[(√5-√4)(√5-√4)]
Or, 1/x=√5+√4---------(2)
From (1)&(2)
x+(1/x)=√5-√4+√5+√4=2√5

Now,
x^4+1/x^4
=(x^2)^2+(1/x^2)^2
= (x^2+1/x^2)^2-2*x^2*(1/x^2)
= (x^2+1/x^2)^2-2
= [(x+1/x)^2-2*x*1/x]^2-2
=[(2√5)^2-2]^2-2
=324-2
=322
Answer: 322
::::::
::::::
Question-4:{3/(x+1)}+{6/(2x+1)}={18/(3x+1)}
[Dhaka Bank-MTO-2017]

Answer: -(3/5)
{Try yourself}

#WRITTEN_MATH_SOLUTION_2017

#BHBFC_SO_2017
:

==========================================
Question-1:Twice the width of a rectangle is 10 meters more than its length. If the area of the
region enclosed by the rectangle is 600 square meters then find its perimeter.

Solution:
Let,
Width of rectangle is x m
And
Length of the rectangle is 2x-10 m
According to the question,
x*(2x-10)=600
or, x^2-5x-300=0
or, (x-20)(x+15)=0

Here, x=20
Either
x=-15 [it's not acceptable]
So,the width of rectangle is 20 m
Length=(20*2-10)=30 m

Perimeter of the rectangle is =2(30+20)


=100 m
Ans:100 m

Question-2:A boat running upstream takes 8 hours 48 minutes to cover a certain distance,
while it takes 4 hours to cover the same distance running downstream. What is the ratio
between the speed of the boat and speed of the water current respectively?

Solution :
Let,
Speed of the boat is x
Speed of the current is y
Speed of the downstream=(x+y)
Speed of the upstream=(x-y)
According to the question,
{8.8(x-y)}={4(x+y)}
or, 8.8x-8.8y=4x+4y
or, 4.8x=12.8y
or, 24x=64y
or, x:y=8:3
And:8:3

Question-3:A customer bought 5 pencils and 6 erasers at tk. 80. Next week, the price of each
pencil increases by 20% but the price of erasers remains unchanged. Now the customer buys
2 pencils and 3 erasers at tk. 39. Find the new price of each pencil.
:::
Solution:
Let,
Price of each pencil be P
Price of each eraser be E
New price of each pencil
=P*6/5
=6P/5
So,price of 2 pencil
=2*6P/5
=2.4P
According to the question,
5P+6E=80-------------(i)
2.4P+3E=39---------(ii)
(i)-(ii)*2
5P+6E=80
4.8P+6E=78
---------------------
0.2P=2
Or,P=2/0.2
=10
So,the new price of the pencil
10*6/5=12
Ans:12 tk

Question-4:A, B and C can complete a work in 12, 15 and 25 days respectively. A and B
started working together whereas C worked with them in every third day. Find the number of
days required to complete the work.
:::
Solution:
A and B together can complete in 1 day
= 1/12 + 1/15
= 9/60
= 3/20 of the work.
A and B with the help of C can complete in 1 day
=(3/20 + 1/25) part of the work
= 19/100 part of the work.
So, their 3 days' work
= 2×3/20 + 19/100
= (30+19)/100
= 49/100 of the work.
So, their 3×2 = 6 days' work
= 49×2/100
= 49/50 of the work.
Remaining work
=1 - 49/50
= 1/50 of the work.
on the 7th day,
A and B will take 1/15 portion work
={20/50*3}=0.133 day's

Therefore, the required number of days


=(6 +.133)=6.133 days.
Answer: 6.133 days.

Question-5:The price of a shirt and a pant together is Tk. 1300. If the price of the shirt
increases by 5% and that of the pant by 10%, it costs Tk. 1405 to buy those two things. Find
the respective price of a shirt and a pant.
:::
Solution:
Suppose,
The prices of a shirt and a pant are Tk. x and Tk. y respectively. According to the question:
x+y = 1300 ----------------------- (i)
1.05x + 1.1y = 1405 ---------- (ii)
Subtracting from
(i)*1.1 - (ii)=»
0.05x =25
Or, x = 500
Substituting the value of x in equation (i) we get,
500+y = 1300 y = 800.

Answer: Shirt Tk. 500, Pant Tk. 800.

Question-6: 3 coins are tossed at random. Show the sample space and find the probability of
getting: - (i) one head two tails
(ii) One tail
(iii) One tail and two heads
Solution:
::::
Total Sample Space after tossed 3 coins randomly
=HHH, HHT, HTH, THH, HTT, THT, TTH, TTT
Probability of getting one head and two tails: In the sample space we can see, a total of 8
types of outcome is possible.
Among these 8 types of outcomes, the combinations with one head and two tails are ----
HTT, THT, TTH,
3 outcomes.
So, the required probability is
3/8

(ii) Probability of getting one tail:


Above the sample space we can see that, a total of 8 types of outcome is possible.
Among these 8 types of outcomes, the combinations with one tail are -- HHT, HTH, THH,
3 outcomes.
So, the required probability is 3/8

(iii) Probability of getting one tail and two heads:Above the sample space we can see that, a
total of 8 types of outcome is possible. Among these 8 types of outcomes, the combinations
with one tail and two heads are : HHT, HTH, THH, i.e: 3 outcomes. So, the required
probability is 3/8.

Answer:
Total Sample Space = {HHH, HHT, HTH, THH, HTT, THT, TTH, TTT),
(i) 3/8, (ii) 3/8, (iii) 3/8.
======================================

«»«»Written Math Solution-2017«»«»

⊙Dhaka Stock Exchange MTO-2017


==========================================
::::::::
::::::::
::::::::
Question-1:The compound interest on a sum for 2 years is Rs. 832 and the simple interest on
the same sum for the same period is Rs. 800. The difference between the compound and
simple interest for 3 years will be?
[DSE, MTO-2017]
::::
::::
Solution:
Given that,
Simple interest for 2 years is Rs.800
So,
Simple interest for 1st year is Rs.400
and
Simple interest for 2nd year is also Rs.400
:
In Similarly,
Compound interest for 1st year will be 400
and Compound interest for 2nd year will be (832 - 400)= 432

So,we can see that compound interest for 2nd year is more than simple interest for 2nd year
by
(432 - 400) = Rs.32

i.e, Rs. 32 is the interest obtained for Rs.400 for 1 year

Rate, R =100×32*400×1=8%

Difference between compound and simple interest for the 3rd year
= Simple Interest obtained for Rs.832
P=800+32=832
R=8%
T=1 year
We know,
SI=PRT/100
=832×8×1/100
=Rs. 66.56

Total difference between the compound and simple interest for 3 years
= 32 + 66.56
= Rs.98.56

Answer: Rs.98.56
Note:
এ এক ক( ৎ ক )। ২ ক

Suppose,
P=100
T=1
R=5%
SI=(100*5*1)/100=5
CI={100(1+5/100)^1-100}=5

Question-2:Price of category A share is Tk. 105, and it's dividend rate is 12%. On the other
hand price of category B share is Tk. 88 and it's dividend rate is 8%. If one wants to earn
same amount of dividend from both option what could be the ration of investment? (DSE
MTO 2017)
Solution:
Let,
Category of A share face value=A
Category of B share face value=B
So, Dividend got both share
12% of A = 8% of B
Or, 12A = 8B
Or, A:B = 2:3
So, investment, A:B
= 2*105 : 3*88
=35:44
Ans:35:44
Note:
[Dividend ঑ Face value এ Market Price & Face Value এক
এক এ Face Value 100 tk but Market Price 120 tk
But dividend ঑ 100 tk এ ।

#Written_Math_Solution_2017«»«»

#National_Bank_PO_2017
=============================================
Question-1:A, B, C can complete a work in 16, 32, 48 days respectively.After starting work 4
days later C left and B left 2 days before the completion of the work.How many days require
to complete the work?
::::::::::::::
Solution:
Let,
Total time taken is x days
According to the question,
x/16+(x-2)/32+4/48=1
Or,6x+3x-6+8=96
Or,9x=94/9
Answer:94/9 or 10.44 days

Alternative Solution:
LCM of 26,32&48 is 96
Let,
Total work=96
efficiency of
A= 96/16=6 units
B= 96/32=3 units
C=96/48=2 units
worked done by A,B,C in 4 days = (6+3+3)*4=44 units
worked done by A in 2 days = 2*6= 12 units
Remaining work
= 96-(44+12)
=40 units
remaining worked done by A&B
= 40/9
Total days = (4+2+40/9)=10(4/9) days
Answer:10.44 days

Question-2:The perimeter of a square is equal to the perimeter of a rectangle field. The length
of the rectangle is three times of its width and the area is 768 sq.meter. How many square
sized tiles of 80 cm width will be required to cover the square field ?
:::::::::
Solution:
Let,
The width of the rectangle is x m
And the length of the rectangle is 3x meter
According to the question,
x*3x=768
Or, x=16
So,
The width of rectangle is 16 meter
And length of rectangle is (3*16)=48 meter
Perimeter of the rectangle
=2(16+48)
=128 m
Suppose,
Each side of the square is a meter
Since the perimeter of the square is equal to the perimeter of the rectangle
4a=128
Or, a=32
Area of the square
=32*32
=1024 square meter
=10240000 square cm
Each tiles wide is 80 cm
Area of the tiles
=(80*80) square cm
Required tiles need
=(10240000/6400)
=1600
Ans:1600

Question-3:Aman,Belal and Chad started a small business with a total amount tk


28,000.Aman paid tk 4500 more than Belal and Belal paid tk 7000 less than Chad.If the
company made a profit of tk 5600 .How much profit should Belal receive?
::::::::::
Solution:
Let,
Belal paid be x tk
Aman paid be (x+4500) tk
Chad paid be (x+7000) tk
According to the question,
x+x+4500+x+7000=28000
Or, x=5500
Belal paid=5500 tk
Aman paid=10,000 tk
Chad paid =12500 tk
Ratio of investment is
Aman:Belal:Chad
=10,000:5500:12500
=20:11:25
Sum of ratio=56
Total profit =5600 tk
Belal should receive profit
=5600*(11/56)
=1100 tk
Ans:1100 tk

Question-4:8/(2x-1) + 9/(3x-1)=7/(x+1)
:::::::
Solution:
8/(2x-1)+9/(3x-1)=7/(x+1)
Or,8/(2x-1)+9/(3x-1)=3/(x+1)+4/(x+1)
Or,8/(2x-1)-4/(x+1)=3/(x+1)-9/(3x-1)
Or,{8(x+1)-4(2x-1)}/(2x-1)(x+1)={3(3x-1)-9(x+1)}/(x+1)(3x-1)
Or,1/(2x-1)=-1/(3x-1)
Or, 3x-1=-2x+1
Or, 5x=2
Or x=2/5

Ans: 2/5
=======================================

Question:A team of workers was employed by a contractor who undertook to finish 360
pieces of an article in a certain number of days. Making four more pieces per day than was
planned, they could complete the job a day ahead of schedule. How many days did they take
to complete the job?
[BHBFC OFFICER-2017]

Solution:
Let,
X number of article produce in a day
Days taken in the schedule time
= 360/X
Days taken when 4 articles are prepared extra per day
= 360/(X + 4)
:
The difference in the day is 1,
:
According to the question,
360/X - 360/(X+4)=1
X^2+ 4X–1440 = 0
X = 36

i.e. number of item prepared in general time is 36, and where 4 articles prepared extra is 40.
Therefore no of days taken to complete the job = 360/40 = 9
Schedule time taken=360/36=10 day's
Reduce time taken=360/40=9 day's

Alternative solution:
Let,
Schedule time taken x day's
But the job actually complete(x-1) day's
when 4 more articles produce in a day

Complete per day in schedule time


=(360/x) pieces
Complete per day in actual time
=360/(x-1)

Difference of pieces per day is 4

According to the question,


{360/(x-1)}-(360/x)=4
Or, (x-10)(x+9)=0
Here
X+9=0
Or, X=-9 [it's not acceptable]

Either
X-10=0
Or, X=10
So,
Schedule time taken 10 day's
But 4 more articles is produce in a day
then actually time taken to finish the job(10-1)=9 day's
Ans: According to the JK khurana Book 10 day's
AT Website 9 days

««»«»Islamic Bank(PO)-2017 Written Math«»«»

Question :The costs of equities of symbol A and symbol B (in dollars) are two different
positive integers. If 4 equities of symbol A and 5 equities of symbol B together costs 27
dollars, what is the total cost of 2 equities of symbol A and 3 equities of symbol B in dollars?
Solution:
X and Y be cost of the equities type of A & B
The cost of 4 equities of A=4 X
The cost of 5 equities of B=5Y
According to the question,
4X + 5 Y =27
Let,
4X=P
5Y=Q
So P is multiple of 4 & Q is multiple of 5
Now
P+Q=27
Or, P=27-Q
Since,Q is multiple of 5
Q=5,10,15,20-------etc

Q=5; P=27-5=22 is not multiple of 4


Q=10;P=27-10=17 is not multiple of 4
Q=15;P=27-15=12 is multiple of 4
Q=20;P=27-20=7 is not multiple of 4
So
Q=15
P=12
Now
P=4X
Or,X=12/4=3
Q=5Y
Or,Y=15/5=3
Therefore,
2 equities of A & 3 equities of B
=2X+3Y
=2*3+3*3
=6+9
=15
Answer:15
======================================
Solutions & Edited
By
Yousuf Ali
Special Thanks:Rubina Akhter

«»«»Written MATH Solution-2017«»«»«»«»«»

¤Janata Bank EO-2017

¤Modhumoti Bank PO-2017

=======================================
Solution,Edited & Complete
By
Yousuf Ali
Special Thanks:Rubina Akhter
=======================================
Question:A number when divided successively by 4 and 5 leaves remainders 1 and 4
respectively. When it is successively divided by 5 and 4, what will be the respective
remainders?
[Indian Bix,GMATH]
Solution:
Math Fact:
*What is Successive division?
[Successive division খ ক এক ক খ গক এ এ গ
ক গ ঑ গ ক এক ক খ গক ]
Solution:
Let
The number be P
Quotient be X
First case
when divided by 4 quotient is X and remainder is 1
So P= 4x+1........(1)
Second case
Let Quotient be M
when divided by 5 quotient is
M and remainder is 4
X=5M+4
Putting X=5M +4 equation no (1)
P= 20M+17
considering M= 1,2,3,4........
P= 37,57,77....
37 or 57 or 77 etc number when divided successively 5 & 4 Remainder always 2 & 3
So respective remainder is 2 and 3
Answer:2,3
========Proof( ):============
**37/4
Remainder=1 & quotient=9
And
9/5
Remainder=4
So first condition is proof
Second case ====
37/5
Remainder=2 & Quotient=7
And
7/4
Remainder=3
So Answer:2,3

Question:A contractor undertakes to do a piece of work in 40 days. He engages 100 men at


the beginning and 100 men after 35 days and completes the work in stipulated time. If he had
not engaged the additional men, how many days behind schedule would it be finished?
[Same as PBL JO MCQ-2014]
Solution:
100 Men 40 days done 1 portion W
100 Men 35 days done 35/40«»«»«
=7/8 portion
Remaining work=(1-7/8)
=1/8 portion
Remaining days=(40-35)=5 days
Total worker=(100+100)=200
200 worker 1/8 portion done 5 day
1========1/8========== 5*200
100======1/8=========(5*200/100)=10 days
If he had not engaged the additional men,Then he needed
(10-5)=5 days more to finish the whole work in stipulated time
Answer: 5 days

Question:In a certain office, 1/3 of the workers are women, 1/2 of the women are married and
1/3 of the married women have children. If 3/4 of the men are married and 2/3 of the married
have children, what part of the workers are without children? [MTB MTO MCQ -2013]
Solution:
Let,
The number of workers in the office is x
Female=x/3
Female + Married
=1/2 of x/3
=x/6 portion
Female + Married+ Children
=(1/3 of x/6)
=x/18 portion
From question,
Male=(x-x/3)=2x/3 portion
Male + Married
=3/4 of 2x/3
=x/2 portion
Male + Married+ Children
=(2/3 of x/2)
=x/3
Total worker with children
=x/3+x/18
=7x/18 portion
Total worker without no children
=(x-7x/18)
=11x/18
Required worker no children
=(11x/18)/x
=11/18
Answer:11/18

Question:The average weight of three men A, B, and C is 84 kg. Another man D joins the
group and the average now becomes 80 kg. If another man E, whose weight is 3 kg more than
that of D, replaces A, then the average weight of B, C, D and E becomes 79 kg. What is the
weight of A?
*[RAKUB SO-2014]
*[AB Bank -2002]
*[Al Arafa Bank-2016]
*[Meghan Bank-2014]
*[Premier Bank-2010]
*[One Bank-2008]
Solution:
The sum of Weight
A+B+C = 84×3 = 252 kg------(1)
If D join,total Weight of
A+B+C+D = 80×4 = 320 kg----(2)

So, the weight of D


=320 - 252 = 68 kg

And weight of E = 68+3 = 71 kg


Again, the sum of Weight of B+C+D+E = 79×4 = 316 kg
Weight of B+C+68+71 = 316 kg
B+C = 316-139 =177 kg
Putting the value of (B+C) equation no (1)
A+B+C=252
Or,A+177=252
Or, A=(252-177)=75
Answer: 75 kg

«»«»MODHUMOTI BANK PO-2017«»«»«»«»«

Question:A shirt sold at 6% profits.If the purchase price was 4% less and selling price was Tk
4 more,the profit be 12.5%.What was the purchase price of the shirt.

#More Written Exam Same As*


*Bangladesh Bank AD-2017
*SBC AM -2016
*BKB -2012
*NBL-2014
*BCS Written
*Non cader Written
*INDIAN BIX
*BAPEX
*Islamic Bank-2013
*BRC-2008
*So on
#Preliminary Exam:
*16th BCS
*MBA-2007-08;1999;1997
*NCC BANK-2002
*PBL-2000

Solutions-1:
Cost price of the article=100 TK
*6% profit first selling price
=(100+100 of 6%)=106TK
*4% less cost price
=(100-100 of 4%)=96 TK
*12.5% profit second selling price
=(100+100 of 12.5%)=112.5 TK
Now,
Cost price TK 100 then Sp=112.5 TK
»»»»»»»»»»TK1»»»»=(112.5/100)TK
»»»»»»»»»» TK80»»»=(112.5*96/100)
=108 TK
Differences of two selling price
=(108-106)=2 Tk
When
TK 2 more then cost price=100 TK
TK4»»»»»»»»»»»»»»»»=(100*4)/2=200TK
Answer:200TK

Solution-2:
Let,
Cost price be= x TK
First Selling price=(x+x of 6%)
=53x/50TK
4%% less cost price
=(100-4) or 96% of x=24x/25 TK
Second SP=(24x/25+24x/25 of 12.5%)
=216x/200 TK
According to the question,
(216x/200)-(53x/50)=4
=================
=================
Or, X=200
The cost price of the article=200 TK
Answer:200 TK

Question:A man's running speed is 3 times of his walking speed.He runs a distance and come
back by walking total time taken 2 hrs.what was the distance if he runs 9 miles per hour.
[PBL -2008]
Solution:
Let,
Distance be=D miles
Runs speed=9 m/h
Walking speed=9/3=3 m/h

According to the question,


D/3 +D/9=2
==========
==========
Or, D=4.5 miles
Answer:4.5 miles

Question:A,B,C started a job which they can complete in 2 days.B can do the job in 5 days
and C can do it in 4 days.After working for 1 day,both B & C left. A complete testbed of the
work how many days?
Solution:
Let,
Total portion work be 1 portion
(A+B+C) 1 day do the job =1/2 portion
B & C 1 day do the job
=(1/5+1/4) portion
=9/20 portion
So,
A 1 day done the job
=(1/2-9/20) portion
=1/20 portion
Remaining work=(1-1/2)=1/2 portion
A 1/20 portion job done=1 day
A 1/2 portion job done=(20/2)
=10 days
Answer:10 days

Question:Solve the equation:


1/(2x-5) + 1/(2x-11) = 1/(2x-7) + 1/(2x-9)
Answer:4

«»«»Bank ASIA MTO -2017«»«»«»«»»


Written MATH Solution
==================================================================
============

Solution,Edited & Complete


By
Yousuf Ali
=======================================
Question:A man borrow tk 500 @5% interest. After some years he again borrow 400@3.5%
interest. Then after 6 month he paid borrowed amont along with interest totalling 994.5 taka.
After how many years of his first borrowing he paid his borrowings and interest?

Solution:
Total amount=(400+500)=900 Tk
Total interest=(994.5-900)=94.5 Tk

3.5% interest rate


400 Tk 1/2 years interest
=[(7*400)/(2*100*2)]
= 7 Tk.
Remaining interest=(94.5-7)=87.5 Tk
5% rate of interest
500 Tk 1 years interest
=(5*500/100)=25 Tk

25 Tk interest for 1 years


87.5 Tk interest(87.5/25)=3.5 years

So after 3.5 years then he took second amount


Answer:3.5 years

Question:A can do a work in 80 days. A work for 10 days After working 10 days, B
completed the task within 42 days. If A & B work together, how many days it would need to
complete the task?
Solution:
Let,
.
.
Total portion work=1
A 80 days done=1 portion
A 10 days done=(10/80)=1/8 portion
Remaining work=(1-1/8)
=7/8 portion
B 7/8 portion done=42 days
B full portion done=(42*8/7)=48 days
Both A & B 1 day done
=(1/80+1/48) portion
=1/30 portion
Both
1/30 portion done=1 days
Full work done=(1*30)/1=30 days
Answer:30 days

Question:A rectangular shapped fieds length is 1*1/2 or 1.5 of its width. To cover its fully
with grass it needs 10260 taka, 1.9 taka per square meter. To give fence around the field,
costing 2.5 tk per meter, how much tk will be needed?
Solution:
Area of the rectangle
=(10260/1.9)
=5400 square meter
Let,
Width of the rectangle=x meter

Length of rectangle=(3x/2) meter


We know,
(3x/2)*x=5400
===========
==========
Or, x=60
And
Length=(3*60/2)=90 meter
Perimeter of rectangle
=2[90+60]
=300 meter
So total Tk needed around rectangle fence=(300*2.5)=750 Tk
Answer:750 Tk

«»«»City Bank (MTO) -2017 Written MATH«»«»

®Exam Held-13.05.2017

==================================================================
============

1) The ratio between the length and the breadth of a rectangular park is 3:2. If a man cycling
along the boundary of the park at the speed of 12 km/hr completes one round in 8 minutes,
then what is the area of the park (in sq. m)?
[Al Arafah Islamic Bank-2013]
Solution:
Let,
Length=3x
Breadth=2x
From the question,
60 minutes the man covere=12km=12000 m
So,
In 8 minutes the man covere=[(12000*8)/60]
=1600 meters
Perimeter of the rectangle is
2(3x+2x) = 1600
x= 160
The Area of the rectangle
=(3x*2x)
= (3*160)(2*160)
=153600 square meter
Ans: 153600 sq.m

2) During the next tree plantation week, Mr. X is considering planting trees in one of its own
rectangular piece of land which is 90 feet long 66 feet wide. This is suspended by boundary
wall of 5 feet height. It has been decided that trees will be planted leaving 5 feet and free
from the wall in all four sides. It was been decide that the distance from one tree to another in
both row and column will be 4 feet. What is the maximum numbers of trees that can be
planted in the land?[SBL MTO-2016]
Solution:
Given that,
The length of the field=90 feet
Possible length of the rectangle
= 90-(5*2)
= 80 feet
Possible Width of the rectangle
= 66-(5*2)
= 56 feet
Possible trees in row
= (56/4 )+ 1
= 15
And
Possible trees in Column
= (80/4) + 1
=21
Maximum tree
= 21*15
= 315

Ans: 315

#Written_MATH_Solution_2017«»«»«»

¤PUBALI BANK SENIOR OFFICER-2017


¤PUBALI BANK OFFICER-2017
¤Exam Date:09.06.2017
:
:
:
:

Question-1:Twenty-four men can complete a work in sixteen days.Thirty-two women can


complete the same work in twenty-four days. Sixteen men and sixteen women started
working for twelve days. How many more men are to be added to complete the work
remaining work in 2 days?
Solution:
Here,
24 men can complete in 16 days 1 portion work
1 man can complete in 1 day=1/(16*24)
=1/384 portion
Again,
32 women can complete in 24 days 1 portion
1 woman can complete in 1 day=1(32*24)
=1/768 portion
Both,
1man +1 woman work in 1 day=(1/384+1/768)
16 men +16 women work 12 days
=[(16*12)/384 + (16*12)/768] portion
=(1/2+1/4) portion
=3/4 portion

Remaining work
=1-3/4
=1/4 portion
Let,
X more men should be added then the remaining work complete in 2 days
Total men =(16+x)
1 man in 1 day done=1/384 portion
(16+x) men in 2 days done=(16+x)*2/384 portion

Similarly,
16 women in 2 days done=(16*2)/768 portion

According to the question,


(16+x)*2/384 +32/768=1/4
Or,(32+2x)/384=1/4-32/768
Or,(32+2x)/384=(192-32)/768
Or,(32+2x)/384=160/768
Or, 32+2x=160/2
Or,2x=80-32
Or,2x=48
Or, x=24
Answer:24 men

Question-2:The average speed of Train in the onward Journey is 25% more than that is the
return journey .The train halts for an hours on reaching the destination. The total time taken
for the complete journey to and fro17 hours, covering a distance of 800 km. Find the speed of
the Train is the onward journe?

Solution:
Let,
Return speed of the train is x kmh
So,
Onward speed of the train be(x+x of 25%) =5x/4
The train halts for an hour
So,Actual time taken the train
(17-1)=16 hours

According to the question,


400/x + 400/(5x/4)=16
Or,400/x +400*4/5x=16
Or,(400*5+400*4)/5x=16
Or,x=(400*9)/(16*5)
Or, x=45

So, Onward speed of the train is =5x/4


=(5*45)/4
=56.25 kmh
Answer:56.25 km/h

=======================================

«»«»«»«»Written MATH Solution-2017«»»«»«»»


¤Agrani Bank Senior Officer -2017
¤Mark-70 [5*14=70]
Date:14.07.2017
========================================
Question-1:A dishonest merchant make a 15% profit at the time of buying and a 10% loss at
the time of selling the goods. By doing so if the said merchant made a profit of Tk. 3500 on a
particular item, what was the real cost of the item sold
Solution-1:
Let,
Cost Price=X Tk
15% profit on cost price
Then market price=X + X of 15%
=1.15 X Tk
Again 10% loss on market price
Then Selling price
=1.15x -1.15x of 10%
=1.035x Tk
Now profits=Selling - Cost Price
=(1.035x - x) Tk
=0.035x Tk
When,
Profit.035x Tk then Cost Price=X Tk
--------3500Tk--------------------------
=[(3500*x)/.035x]
=1,00,000 Tk
Answer:1,00,000 Tk

Solution-2
Let, cost price = 100 Tk.
At 15% profit the market price of the product will be=100+15=115 Tk.
At 10% loss, the sales price will be=115-(115*10%)=Tk. 103.5 Tk.
So profit = 103.5-100= 3.5 Tk.
Now,
when profit 3.5 Tk. then cost 100 Tk.
when profit 3500 Tk. then cost (100/3.5)*3500=1,00,000 Tk.
So, real cost is Tk. 1,00,000 (Ans.)

Question-2:ক এক ক ৩০ ,খ ১৫ এ গ ১০ ক ২ খএ
৩ গ,ক ক ক ঐক ক ?[BKB SO-2015][Agrani Bank SO-
2017]
[A can do a piece of work in 30 days,B can do it in 15 days and C can do it in 10 days.Every
second days B and every third days C help A .How many days will finish the whole work?]
Solution:1
Let,
Total work=1 portion
LCM of 2 & 3=6
Per 6 days A work= 6 days
Per 6 days B work=(6/2)=3 days
Per 6 days C work=(6/3)=2 days
So,
6 days (A+B+C)'s work
=(6/30+3/15+2/10) portion
=3/5 portion
Remaining work
=(1-3/5) portion
=2/3 portion
After 2 days(A+B)'s work
=(2/30+1/15)portion
=2/15 portion
[Every second days B help A]
Remaining work
=(2/5-2/15) portion
=4/15 portion
Total time=(6+2)=8 days
9th day(A+C) work
=(1/30+1/10) portion
=2/15 portion
Remaining work
=(4/15-2/15)portion
=2/15 portion
Another 2 days(A+B)'s work
=(2/30+1/15) portion
=2/15 portion
Remaining work
=(2/15-2/15)
=0
So total time taken to finish the work=(9+2)=11 days
Answer:11 days
Solution:2==================
এখ ,
২঑৩এ =৬
৬ ক ক ক,খ,গ এক এ ৬ ক ক
=(৬/৩০)+(৩/১৫)+২/১০)
=৩/৫
ক ক =(১-৩/৫)
= ২/৫
২/৩<৩/৫
ক,খ,গ এক এ ৩ ক ক =(৩/৩০+১/১৫+১/১০)
=৪/১৫
ক ক =(২/৫-৪/১৫)
=২/১৫
২/১৫<৪/১৫
ক এক ১/৩০ ক ক
ক ক =(২/১৫-১/৩০)
=১/১০
এখ ক, খ এক এ ক ১/১০ ক
ক, খ এক এ ১ ক ক =(১/৩০+১/১৫)=১/১০ ক
ক ক :(১/১০-১/১০)=0
=৬+৩+১+১=১১
Answer :11 days

Question-3:Two trains ,one from Dhaka and another from chittagong simultaneously started
to proceed towards each other at the speed of 16 km and 21 km per hour respectively.As the
trains met each other it was found that one train travelled 60 km more than the
other.Calculate the distance between Dhaka to Chittagong.
Solution:
Let,
Two trains met each other 't' hrs later
First trair covered=(16*t) km
Second train covered=(21*t)km
Differences of distance two trains
=60 km
According to the question,
21t-16t=60
Or, 5t=60
Or, t=12
Total distance Dhaka to Chittagong
=(16*12+21*12)
=444 km
Answer:444 km

Question-4:Sakib and Labib individually borrowed different amount of money from a


particular bank on the same day at rate of 20% simple interest.The total money paid by Sakib
in 3 years as principal plus interest was the same amount Labib paid in 2 years as principal
plus interest.Find the ratio of their individual loan amount.
Solution:
Sakib and Labib borrowed amount
X & Y Tk respectively
According to the question,
X+ X of 20%*3=Y+Y of 20%*2
Or,X+0.6 X =Y +0.4Y
Or, 1.6 X=1.4Y
Or, X:Y=1.4:1.6 or 7:8
Answer:7:8

Question-5:The perimeter of a square is equal to the perimeter of rectangle. The lenght of the
rectangle is three times longer than its width having total area of 1200 sq.meter. what will be
the total cost cost if the total area of the square is covered with stones having a dimension of
50 centimeter square each & if tk.50 is charged for placing a stone in the square?
Solution:
Let,
Breadth of rectangle=x m
And length of Rectangle= 3x m
According to the question,
3x^2 = 1200
x= 20
Length of rectangle = 60m
Perimeter of rectangular
= 2(60+20)=160m
As per question,
Perimeter of rectangular= perimeter of squire = 160m
Side of square = 160/4
=40m or 4000 cm
area of square =(4000*4000)sq.cm
Square of stone
= (50)^2
= 2500 sq.cm
number of stones = (4000*4000)/2500=6400
Total cost = 6400*50=320000
Answer:3,20,000 Tk
NOTE:৫০ . গ গ এক ঘ ৫০ .
৫০ গ . এক গ ৫০ গ . ।

«»«»Written MATH Solution- 2017«»«»


::::
¤Post:BKB OFFICER-2017
¤Date:14.09.2017

========================================
1. Proof of identity of 115 people was verified. 65 of them had Passport, 30 of them had both
Passport and Voter ID. However, 15 of them could not produce any identity documents. How
many of them showed up only with Voter ID?

Solution:
Total people=115
Undocments people=15
So,
Documents people=(115-15)=100
Both voter ID and passport have
=30
Let,
Voter ID have=x people
Only passport have=(65-30)=35 people
Only voter ID have=(x-30) people
So,
Only passport or Voter ID or Both
=35+(x-30)+30
According to the question,
35+(x-30)+30=100
Or, x =65
So,
Only voter ID have=(65-30)=35
Answer: 35.

2. Rafiq, Shafiq, and Arif can alone complete a project in 10 days, 20 days, and 10 days
respectively. Rafiq started working on the project alone. Shafiq joined the project after 2
days. After working together for 4 days both Rafiq and Shafiq left the project and Arif got in.
How many days it took to complete the entire project?

Solution:
Rafiq can do in 1 day = 1/10 of the work
Shafiq can do in 1 day = 1/20 of the work
Arif can do in 1 day = 1/10 of the work

Rafiq first 2 days work done


=2/10 portion
=1/5 portion
Remaining work
=(1-1/5) portion
=4/5 portion
Both Rafiq & shafiq 4 days done
=(4/10)+(4/20)
=3/5 portion
Remaining work
=(4/5)-(3/5)
=1/5
Arif full work done in 10 days
Arid 1/5 portion work done
=10*1/5
=2 days
So, Total time taken to complete the whole work
=(2+4+2)=8 days

Answer: 8 days.

3. Mr. Rashid has saved Taka 1200 from his first month's salary. He plans to increase his
monthly savings by Taka 100 in every following month. How much time would it take to
save Taka 106200?

Solution:
Let,
'n' months be required to save Tk 106200.

Here,
the savigns in the first month,
a = 1200 Tk
Increase in savings in each month, d = 100 Tk

According to the question:


n/2{2a+(n-1)*d}=106200
Or,(n/2){2×1200 + 100(n-1)} = 106200
Or,n(1200 + 50n - 50) = 106200
Or,n² + 23n = 2124
Or,n² + 59n - 36n - 2124 = 0
Or, (n+59)(n-36) = 0
Either,
n = 36
Or,
n=-59[It is not accepted]

Ans: 36 months or 3 years.

4. Solve the equation : 2{(x+3)/(x-3)}² - 7{(x+3)/(x-3)} + 6 = 0

Solution:
Suppose,
(x+3)/(x-3) = P---------(1)
So,
2{(x+3)/(x-3)}² - 7{(x+3)/(x-3)} + 6 = 0
Or, 2p² - 7p + 6 = 0
Or, 2p² - 4p - 3p + 6 = 0
Or, 2p(p-2) - 3(p-2) = 0
Or, (p-2)(2p-3) = 0

So, either
P-2=0
(x+3)/(x-3) = 2
2x - 6 = x + 3
x=9
or,
2p-3=0
(x+3)/(x-3) = 3/2
3x - 9 = 2x + 6
x = 15

Ans: 9 or 15.

5. The area of a rectangle is 1200 square meter. If the length of the rectangle is reduced by 10
meters, it becomes a square. Calculate the length of the diagonal of the rectangle.
Solution:
Let,
Length of the rectangle is=x
Width of the rectangle is=y
If the length of the rectangle is reduced by 10 meters, it becomes a square.
y=x-10--------(1)

We know that,
xy = 1200
Or, x (x-10)=1200
x² - 10x - 1200 = 0
(x-40)(x+30) = 0
Either,
x = -30 [It's not accepted]
Or,
x-40=0
Or, x=40
So, the width of the rectangle is

=40-10= 30 meters,
and
the length of the rectangle is = 40 meters.

So, the length of the diagonal of the rectangle is = √(40²+30²) = 50 meters.

Answer: 50 meters

6. A Rhombus has an area of 120 square meters and the length of its one diagonal is 10
meters. Calculate the perimeter of the Rhombus.
Solution:
Let
The length of a side of the rhombus is = a meters,
and that of the other diagonal is
=d meters.

We know that,
area of a rhombus =1/2* (product of the lengths of the diagonals)
Or, 120 = (10×d)/2
Or, d = 24 meters.

Again, We know:
a² = (24/2)² + (10/2)²
a² = 169
a = 13

So,
the perimeter of the rhombus is = 4a = 4×13 = 52 meters.

Answer: 52 meters.

7. The area of an equilateral triangle is 3√3 square meter. Determine the area of a circe
inscribed within the triangle.

Solution:By Mainul Maksud Quaes Sir


Area of equilateral triangle = (√3/4)a²
So, (√3/4)a² = 3√3
=> a² = 12
=> a = 2√3
So, Semi-Perimeter of triangle = 3*2√3/2 = 3√3
Now, Radius* Semi-perimeter = Area
=> Radius = 3√3/3√3 = 1
Then, Area of circle = πr² = π*1² = π or 3.14 sqm (Ans.

«»«»Written MATH Solution-2017«»«»


¤Dhaka BANK-MTO-2017
¤Date:29.09.2017
===========================================
Question-1:If 7% of the sale price of a product equal to 8% of cost price and 9% of the sale
price exceeds 10% of the cost price by Tk 1, find the amount of profit and cost of the
product?
[Dhaka BANK-MTO-2017]
Solution:
Let,
Selling price be =X Tk
And
Cost price be =Y Tk
First condition,
7% of X =8% of Y
7x/100=8y/100
Or, x =8y/7-----------:--(1)
Second condition,
9%x -10%y=1
Or, (9X/100)--(10Y/100)=1
======================
======================
Or, Y =350
From equations no(1)
X =(8*350)/7
Or, X =400
So,
Selling price of the product is 400 Tk
Profit=(Selling Price--Cost price)
=(400-350)=50 Tk
Answer: Tk.350 & Tk. 50
:::::
::::
Question-2:Mr Amin invests Tk 24000 in Dhaka Bank at 7.50%. How much additional
money must he invest at 10% so that the total income will be equal to 9.25% of his entire
investment?
[Dhaka BANK-MTO-2017]
Solution:
Let,
He investment additional amount =x
According to the question,
24000*7.50%+x * 10%= (24000+ x) 9.25%
Or,[(24000*7.50)/100] +10x/100=
[(24000+x)*9.25]/100
Or, 180000 +10x =222000+9.25x
Or, 10x -9.25x =222000-180000
Or, 0.75 x =42000
Or, x=56000
So, He invest additional amount is
56000 Tk
Answer:56000Tk
:::::::
:::::::
Question-3:Find the value of x^4+1/x^4 ; If x=√5-√4
[Dhaka Bank-MTO-2017]
Solution:
Given that,
x=√5-√4----------(1)
1/x= 1/(√5-√4) [Reverse both side]
Or, 1/x= (√5+√4)/[(√5-√4)(√5-√4)]
Or, 1/x=√5+√4---------(2)
From (1)&(2)
x+(1/x)=√5-√4+√5+√4=2√5

Now,
x^4+1/x^4
=(x^2)^2+(1/x^2)^2
= (x^2+1/x^2)^2-2*x^2*(1/x^2)
= (x^2+1/x^2)^2-2
= [(x+1/x)^2-2*x*1/x]^2-2
=[(2√5)^2-2]^2-2
=324-2
=322
Answer: 322
::::::
::::::
Question-4:{3/(x+1)}+{6/(2x+1)}={18/(3x+1)}
[Dhaka Bank-MTO-2017]

Answer: -(3/5)
{Try yourself}
=======================================
Section-F: Practice Math
¤Janata Bank Executive Officer-2017
=======================================
»««»«»«»«Shift:Afternoon«»«»«»«»

**The sum of two numbers is equal to thrice their difference. If the smaller of the number is
10.find the other number
Solution:
Let,
The larger number be= x
According to the question,
x+10 = 3(x-10)
or, x+10 = 3x-30
or, -2x=-40
Or,x = 20
Answer:20

*Question:The average age of 8 men is increased by 2 years when one of them whose age is
24 years is replaced by a woman. What is the age of the woman?
Solution:
8 person total age increases
=[8*2]=16 years
So,The woman age will be
=[16+24]
=40 years
Answer:40 years

*Question:50% of a% of b is 75% of b% of C. Which of the following is C?


Solutions:
According to the question,
(50/100*a/100 )of b=(75/100*b/100) of C
==================
==================
Or, C=0.667a
Answer:0.667a

*Question:A makes an article for TK.120 and sells it to B at a profit of 25% .B sells it to C
who sells it for TK.198 making a profit of 10%. What profit percent did B make?Solution:
25% profit
A's Selling price=125% of 120
=150Tk
=B'Cost price
10% Profit
C's Sp=110 TK
SP 110 TK Then CP=100 tk
«»198«»«»«»«»«»»=[100*198]/110
= 180 Tk
=B's Selling Price
So B profit
=[(180-150)/150]*100
=20%
Answer:20%

Question:A sum of TK.600 amounts to TK.720 in 4 years at simple interest. What will it
amount to if the rate of interest is increased by 2% ?[Difficult]
Solution:
Let
Rate of interest=r%
We know
P=100*a/100+(r*t)
Or, 600=(100*720)/[100+(4*r)]
======================
Or, r =5
Rate increased by 2%=(5+2)=7%
Again we know
I=Prn
Or, I=(600*7*4)/100
Or, I =168 Tk
So Amount=(600+168)=768 Tk
Answer:768 Tk

*Question:The sum of first five prime number is :


Solution:
The first five prime number
=2+3+5+7+11=28
Question:What is the following is equal to 3.14× 10^6?
Solution:
3.14*10^6
= ( 314/100)*10000
=3140000
Answer:3140000

Question:If x=1 - q and y = 2q+1 , then for what value of q. x is equal to y ?


Solution:
Since x = y
So,
1 – q = 2q+1
or, -3q = 0
So,q = 0
Answer:0

*Question:3 years ago , the average age of a family of 5 members was 17 years . A baby
having been born , the average age of the family is the same today. The present age of the
baby is
Solution:
3 years ago 5 member average age=17 years
Now their present average age
=17+3=20 years
And
Their total age now=20*5=100
5 person with baby's now total age=6*17=102 years
The present age of baby
=(102-100) years
=2 years
Answer:2 years

Question:There are two numbers such that the sum of twice the first and thrice the second is
39, while the sum of thrice the first and twice the second is 36. The largest of the two is
Solution:
Let,
The first number is x
and
Second number is y
First condition, 2x+3y = 39---(1)
2nd condition 3x+2y = 36----(2)
By solving the 1 & 2 equation we get x = 6 and y = 9
So, largest number is 9
Answer:9

*Question:A person was asked to state his age in years .His reply was , “take my age three
years hence , multiple is by 3 and then subtract three times my age three years ago and you
will know how old I am ” What was the age of person ?
Solution:
Person present age=x years
After 3 years=x+3
Before 3 years=x-3
According to the question,
3(x+3)-3(x-3)=x
Or,3x+9-3x+9=x
Or, x=18
Answer:18 years

*Question:If root 2^n = 64 , then the value of n is


Solution:
2^n = 64
or,2^n= 2^6
or, 2n = 2^12
So, n=12
Answer:12

*Question:In an election, 30% of the voters voted for candidate A whereas 60% of the
remaining voted for candidate B.The remaining voters did not vote. If the difference between
those who voted for candidate A and those who did not vote was 1200, how many individuals
were eligible for casting vote in the election ?
Solution:
Candidate A got = 30%
Remaining vote= 100-30 =70% Candidate B got = 70of60%
= 42%
The voters did not vote
=(70-42 )= 28%
Difference of A and did not vote is 30-28 = 2%
So========
2%«»«»»«»«»»«»«120
100%««»«»«»«»«»
= (120*100)/2=60,000
Answer:60000

*Question:A shopkeeper expects a gain of 22.5% on his cost price . If in a week his sale was
of TK.392 , what was his profit ?
Solution:
Cost price =100%
Gain = 22.5%

122.5%«»«»«»«»392
1% «»«»«»«»«»«»392/122.5
22.5% «»«»«»«»«»[392*22.5]/122.5
=72 Tk
Answer:72 Tk

*Question:10 men working 6 hours a day can complete a week in 18 days .how many hours a
day must 15 men work to complete the same work in 12 days ?
Solution:
MCQ Way======
M1*T1/W1=M2*T2/W2
Or,T2=6*10*18/15*12
T2=6 hrs
Answer:6 hrs

*Question:Two taps A and B can fill a tank in 5 hours and 20 hours respectively . If both the
taps are open then due to leakage , it took 30 minutes more to fill the tank . If the tank is full ,
how long will it take for the leakage alone to empty the tank ?
Solution:
MCQ Way===
Both A+B time taken
=A*B/A+B
=5*20/5+20
=4 hrs
Due to a leakage time taken to fill the tank=(4+.5)=4.5 hours
So leakage pipe time taken to empty
=(4.5*4)/4.5-4
=36 hrs
Answer:36 hrs

*Question:A train can travel 50% faster than a car. Both start from point A at the same time
and reach point B 75kms away from A at the same time . On the way , however , the train lost
about 12.5 minutes while stopping at the station . The speed of the car is[Difficult]
Solution:
Speed ratio of Train and Car
=150:100
= 3:2
Let,
Speed of train=3x
Speed of car=2x
According to the question,
75/2x -75/3x=12.5/60
=================
=================
Or, x = 60
The speed of the car=2*60
=120 km/hr
Answer:120 km/hr

*Question:A towel , when balanced was found to have lost 20% of its length and 10% of its
breadth . The percentage of decrease in area is Solution:
First loss=100-20=80
Then second loss
=80 - 80 of 10%
=8%
Total decrease=(20+8)=28%
Answer:28%

«»«»«»Morning Shift»«»«»«»«

*Question:
(87*87+67*67-2*87*61)=?
Solutions:
Let,
87=a
67=b
(a-b)^2 er formula
( 87-61)^2
= 26^2
= 676
Answer:676
Note:Question e answer nei

Question:Which of the following is not a prime number?


Answer:91

Question:138.009+341.981-146.305=123.6+?
Answer:210.085

*Question:The average of first 50 natural numbers is


Solutions:
MCQ Way====
Average=[Last+First]/2
=(50+1)/2
=25.5
Answer:25.5
*Question:The sum of two numbers is 40 and their difference is 4.The ratio of the numbers is
Solution:
Let
The numbers x & y
So,
x+y = 40--------(1)
And
x-y = 4-----------(2)
From two equations
x= 22
y = 18
Ratio = 11:9
Answer:11:9
*Question:One year ago,Punky was four times as old as her daughter Soma.Six years
hence,Punky's age will exceed her daughter's age by 9 years.The ratio of the present ages of
pinky and her daughter is
Solution:
Short cut way==
Back solve
13-4 = 9
(13-1)÷3 = 4
Ratio of present age Pinky and her daughter=13:4
Answer:13:4

*Question:In a certain store the profit is 320% of the cost .If the cost increases by 25%
but,the selling price remains constant,approximately what percentage of the selling price is
the profits?
Solutions:
Initial cost price = 100 Tk
320% profits sell price = 420 Tk
25% increase CP = 125 Tk
Profit =(420-120)=295 Tk
Percentage of profit
=(295*100)/420
=70%
Answer:70%
14.
A:C = ( 2:1)*3= 6:3
A:B = (3:2)*2 =6:4
B gets = 157300*4/13 = 48400
Question:A certain number of men can a piece of work in 100 days.If there were 10 less men
,it would take 10 days more for the work to finished.How many men were there originally?
Solutions:
Let,
Originally men=x
Short cut way====
x*100 = 110(x-10)
x= 110
Answer:110
Question:A sum of money lent out at simple interest amounts to Tk 720 after years and to Tk
1020 after a further period of 5 years.The sum is
Solution:
7yr sum+profit = 1020 Tk
2 yr sum+profit = 720 Tk
------------------------------------
5 yr «»«»«»«»«»«»«»«»300 Tk
2 yr profit «»«»»«»«»«» 300*2/5
=120 Tk
Sum = 720-120
= 600 Tk
Answer:600 Tk
Question:The ratio between the perimeter and the breadth of a rectangular is 5:1.If the area of
the rectangular is 216 sq.cm ,what is the length of the rectangle?
Solutions:
Let,
Length be =x cm
Breadth be=y cm
According to the question,
2(x+y):y=5:1
Or,2x+2y=5y
Or x=3y/2--------(1)
Now
Area=216
Or,xy=216
Or,3y^2=216*2
Or,y=12
So x=12*3/2=18 cm
Answer:18 cm

*Question:How many times in a day the hands of a clock are straight?


Solution:
»Straight line=22
»coincide line=22
»Right angle=44
»Acute/ obtuse=44

*Question:(0.04)^-1.5=?
Solution:
=1÷(4/100)^1.5
=1÷(1/25)^1.5
=1÷(1/5)^(2*1.5)
=1÷(1/5)^3
=125
Answer:125

Question :The sum of two numbers is 40 and their difference is 4. The ratio of numbers is
Solution:
Let
first number is x and second number is y
x+ y= 40 --------(1)
x–y= 4 ----------(2)
Solving 1 & 2 equation
x:y = 22 : 18
= 11 : 9
Answer :11:9
*Question :A is faster then B. A and B each walk 24km. The sum of their speeds is 7 Km/hr
and the sum of times taken by them is 14 hours. Then, A;s speed is equal to
Solution:
Let
The speed of A = x km/hr
and
Speed of B = 7-x km/hr
According to the question,
24/x+24/7-x =14
Or,(168-24x+24x)/7x+x^2=14
======================
======================
Or,(x-3)(x-4)=0
SO
X=3 or 4
A's speed=4 km/h
Because A is faster Than B
Answer:4 km/hrs

*Question:Two trains are running in opposite directions with the same speed. If the length of
each train is 120 meters and they cross each other in 12 second , then the speed of each
train(in km/hr) is
Solution:
Let
Speed of each train=x m/s
According to the question,
12=(120+120)/x+x
Or,x=10
So speed of each train
=10*(5/18)
=36 KM/hrs
Answer:36 km/hr
*Question A sum of money is borrowed and paid back in two annual installments of TK.882
each allowing 5% compound interest .The sum borrowed was
[Difficult Math]
Solution:
Let,
The sum is =x tk
In first years compound
882=x(1+5/100)^1
Or,882=21x/20
Or,x=840
Second year,
Amount=(840+882)=1722 tk
Second year compunted,
1722=x(1+5/100)^1
Or,1722=21x/20
Or,x=1640
The sum borrowed was=1640 tk
Answer:1640 tk

***Some Math Practice Book**


Preliminary Book
1.George Math Review
2.Khairul's Bank Math
3.Professor Bank Math Solution
4.Saifur's Math
Note:You Purchase any two book
One Master & One Back Up
**Written Book
1.Arifur Rahman Bank Written
[1986 To 2017]
2.Professor Bank Math Solution
3.Saifur's Bank Written

Solution,Edited & Complete

BY
Yousuf Ali
Special Thanks:Rubina Akhter
=======================================

Question :The costs of equities of symbol A and symbol B (in dollars) are two different
positive integers. If 4 equities of symbol A and 5 equities of symbol B together costs 27
dollars, what is the total cost of 2 equities of symbol A and 3 equities of symbol B in dollars?
[Nova's GRE MATH Bible]
[IBBL PO-2017]
Solution:
Math Fact:
[এখ A&B Equities এ But এখ এ A and B
equities
এখ খ equities এ 3 Tk ঑ type A & Bএ 12 & 15 Tk
খ ক ]

X and Y be cost of the equities type of A & B


The cost of 4 equities of A=4 X
The cost of 5 equities of B=5Y
According to the question,
4X + 5 Y =27
Let,
4X=P
5Y=Q
So P is multiple of 4 & Q is multiple of 5
Now
P+Q=27
Or, P=27-Q
Since,Q is multiple of 5
Q=5,10,15,20-------etc

Q=5; P=27-5=22 is not multiple of 4


Q=10;P=27-10=17 is not multiple of 4
Q=15;P=27-15=12 is multiple of 4
Q=20;P=27-20=7 is not multiple of 4
So
Q=15
P=12
Now
P=4X
Or,X=12/4=3
Q=5Y
Or,Y=15/5=3
Therefore,
2 equities of A & 3 equities of B
=2X+3Y
=2*3+3*3
=6+9
=15
Answer:15

Question:Two trains, train Alpha and Beta run in opposite directions on a circular track. Train
Alpha travels at a rate of 4π miles per hour and Train Beta runs at a rate of 6π miles per hour.
If the track has a radius of 6 miles and the trains both start from point Delta at the same time,
how long, in hours, after the trains depart will they again meet at
[GMATH]
Solution-1:
[এ Math এ এক Easy ]
Though,the direction is opposite but the track is circular
So,
The relative speed
=Bet's speed - Alpha's speed
=6π -4π
=2π m/h
Total distance of circular track
=2πr
=2π*6
=12π[Radius=6 miles]
We know
Velocity=Distance/Time
Or,Time=12π/2π
Or, Time=6
Answer:6 hours
Solution-2(Collected):=======
Circumference of the track = 2πr = (2)(π)(6) = 12π miles.

At a rate of 4π miles per hour, the time for Train Alpha to complete one revolution and arrive
back at point Delta = (circumference)/(rate) = (12π)/(4π) = 3 hours.
At a rate of 6π miles per hour, the time for Train Beta to complete one revolution and arrive
back at point Delta = (circumference)/(rate) = (12π)/(6π) = 2 hours.

Since Train Alpha arrives back at point Delta every 3 hours, and Train Beta arrives back at
point Delta every 2 hours, the time required for both trains to meet back at point Delta must
be a MULTIPLE of the two times:
3*2 = 6 hours.
In 6 hours, Train Alpha will complete a total of 2 revolutions (one every 3 hours), while
Train Beta will complete a total of 3 revolutions (one every 2 hours), with the result that both
trains will meet again at point Delta.
Answer:6 hrs

Question:A milk vendor has 2 cans of milk. The first contains 25% water and the rest milk.
The. second contains 50% water. How much milk should he mix from each of the containers
so as to get 12 litres of milk such that the ratio of water to milk is 3 : 5
[Indian Bix]
Solution:
Suppose,
Each can x and y litres of milk and water should be mixed to get water and milk=3:5
Can-1:
Water=x of 25%=x/4
Milk=x of (100-25)%=3x/4

Can-2:
Water=50% of y=y/2
Milk=50%of y=y/2
According to the question,
(X/4+y/2)/(3x/4+y/2)=3/5
=====================
======================
Or, x:y=1:1
Sum of ratio=1+1=2
Each can milk should be mixed
=12*1/2
=6 litres
Answer: 6 & 6 litres

Question:Tow varieties of sugar are mixed in a certain ratio.The cost of the mix per kg is tk
.50 less than that of the superior variety and tk .75 more than that of the inferior variety.What
was the ratio of superior variety to inferior variety in the mixture?
[MentorsQBank]
Solution:
Let,
Cost price superior variety=x
Cost price of inferior variety=y
Superior quantity unit=a
Inferior quantity unit=b
Total quantity mixture=a+b
Total cost price=ax +by
Mean price=ax + by/a+b
Suppose,
Mean price=z
According to the question,
X - Z=.50 or Z-X=-.50
Z - y=.75 or y- Z=-.75
Now,
Mean price=ax+by/a+b
Or,ax + by/a+b=z
==============
==============
==============
Or,a/b=(y-z)/(z-x)
Or,a/b=-.75/-.25
Or, a:b=3:2
Answer:3:2

Question :A speaks truth in 75% cases and B in 80% of cases. In what percentage are they
likely to contradict each other, narrating the same incident
Solution :
Let,
A = Event that A speaks the truth
B = Event that B speaks the truth

Then P(A) = 75/100 = 3/4


P(B) = 80/100 = 4/5

P(A-lie) = 1-3/4 = 1/4


P(B-lie) = 1-4/5 = 1/5

Now
A and B contradict each other =
[A lies and B true] or [B true and B lies]
= P(A).P(B-lie) + P(A-lie).P(B)
= (3/5*1/5) + (1/4*4/5) = 7/20
= (7/20 * 100) %
= 35%

Question:A man deposited tk 50000 at a certain interest for 1 yr.After 1 yr he received tk


55280 as both principle & interest after deduction of tk 120 as govt levy and 10% on interest
as govt tax.What was the interest rate in percentage?
[MentorsQBank]
Solution:
Here,
Deposited Amount = 50000 Tk
After 1 year
Receiving Amount = 55280 Tk
So,
Interest Received =(55280-50000)
= 5280 Tk
Govt Levy = 120 Tk
So,
Total(120+5280) = 5400 Tk which is (100-10)=90% amount internet

Now,
90% = =========5400
100%======
=[(5400*100)/90]=6000 Tk

Interest = 6000 Tk
Principal=50000 Tk
We know,
I = Pnr
Or, r=I/Pn
Or, r=[6000*100/50000]
= 12%
Answer:12%

Question:The ratio between the ages of Mary and her mother is 1: 2 and that of Mary and her
father is 1: 3 at the time of Mary's birth. Mary is 10 years old now. Find out the ratio between
the ages of Mary's mother and father at this age of Mary.
[GMATH]
Solution:
According to the question,
when Mary took birth,
The ratio between the ages of Mary and her mother is 1: 2
The ratio between the ages of Mary and her father is 1: 3
Let,
'x' is the age of Mary when she is born
Hence, we can say that her mother's age is 2x and her father's age is 3x---------i
Now,
Mary is 10 years old.
Therefore, from equation 1 we can say,
Her mother's age is 2x
i.e. 2 * 10 = 20
and her father's age is 3x i.e. 3 * 10 = 30 at this age of Mary.
Thus, the ages of Mary's mother and Mary's father are in the ratio 20:30 = 2:3

Solution-2:
According to the question,
Mary age:Mother age=1:2
Mary age:Father age=1:3
So,
Mary age:Mother age:Father age
=1:2:3
Let,
X is the present age of each
Mary age=x
Mother age=2x
Father age=3x
From question,
X=10
So,
Mother age=10*2=20 years
Father age=3*10=30 years
Ratio of Mother & Father
=20:30
=2:3
Answer:2:3

Question:A can contains a mixture of two liquids A and B is the ratio 7 : 5. When 9 litres of
mixture are drawn off and the can is filled with B, the ratio of A and B becomes 7 : 9. How
many litres of liquid A was contained by the can initially?
[Indian Bix]
Answer:21 litres
Details:Comments box

***IBBL PO-2017 Preliminary Question


Question:What is the average(arithmetic mean)of all multiples of 10 from 10 to 400
inclusive?
Solution:
Series=10+20+30.......+4000
Term=[(400-10)/10]+1
=40
Sum=N/2 * (Last term+first term)
=40/2 *(400+10)
=8200
Average of all multiples of 10 from 10 to 400
=8200/40
=205
Answer:205

Question:In a division sum,the remainder is zero.A student mistook the divisor by 12 instead
of 21 and obtained 35 as quotient.What is the correct quotient?
Let,
.
.
Number be=N
So,Number
N=35*12+0
=35*12
Correct quotient=(35*12)/21=20
Answer:20

Question:How many positive integers less than 100 have a remainder of 2 when divided by
13?
Solution:
Short Cut way.....
Let,
Number be =N
Number
N*13 + 2=100
Or, N=98/13
=7.53
=8
Answer:8

Question:How many odd,positive divisors does 540 have


Factors of 540=2*2*3*3*3*5
=2^2*3^3*5
So,
Odd positive integers
=(3+1)*(1+1)
=4*2=8
Answer:8

«»«»«»Answer of Quick MATH Test«»«»

***IBBL PO-2017 Preliminary Question


Question:What is the average(arithmetic mean)of all multiples of 10 from 10 to 400
inclusive?
Solution:
Series=10+20+30.......+4000
Term=[(400-10)/10]+1
=40
Sum=N/2 * (Last term+first term)
=40/2 *(400+10)
=8200
Average of all multiples of 10 from 10 to 400
=8200/40
=205
Answer:205

Question:In a division sum,the remainder is zero.A student mistook the divisor by 12 instead
of 21 and obtained 35 as quotient.What is the correct quotient?
Let,
.
.
Number be=N
So,Number
N=35*12+0
=35*12
Correct quotient=(35*12)/21=20
Answer:20
Question:How many positive integers less than 100 have a remainder of 2 when divided by
13?
Solution:
Short Cut way.....
Let,
Number be =N
Number
N*13 + 2=100
Or, N=98/13
=7.53
=8
Answer:8

Question:How many odd,positive divisors does 540 have


Factors of 540=2*2*3*3*3*5
=2^2*3^3*5
So,
Odd positive integers
=(3+1)*(1+1)
=4*2=8
Answer:8

¤ ক ঑ ¤

১. ক, খ ঑ গ এক ক ২০, ২৪ ঑ ৩০ ক এক ৬ ক
ক খ঑গ গ ক ক ক এক ক ক ? [গ ঑গ
গ ক ক : ০৫]
ক. ৩ খ. ৪ ●৫ ঘ. ৬

২. এক খ ১০ ঘ এ
খ ১৫ ঘ খ এক খ খ ক ঘ
?[ ঑ ক এ : ০৫/ : ০৪]
ক. ২০ ঘ খ. ২৪ ঘ গ. ২৮ ঘ ● ৩০ ঘ

৩. এক ড় এক ২৫ ড়
খ ৫০ খ ক
এক ক ক ক ?[ : ৯৯]
ক. ২০ . ● ২৫ . গ. ৩০ . ঘ. ৩৫ .

৪. এক ১০, ১৫ ঑ ১২ এক খ
ক ক গ ?[ ক ক : ৯৭]
ক. ১ ঘ ● ২ ঘ গ. ৩ ঘ ঘ. ৪ ঘ

৫. ক ২০ ঑ ৩০ ক খ ক
এক খ কখ ক ১৮ এ ?
[ ক ক ক: ০২]
●৮ খ. ১০ . গ. ১২ . ঘ. ১৪ .

৬. ক এক ক ১২ এ খ ২৪ ক এক ক ক ক ক ক
ক খ কক খ৩ ক ক ক ?
[ ,ক ঑ - ক: ০১]
● ১০ খ. ১২ গ. ১৫ ঘ. ১৬

¤PUBALI BANK SENIOR OFFICER-2017

¤PUBALI BANK OFFICER-2017

1.Two trains of equal length are running on parallel lines in the same direction at 46
kilometre/hour and 36 kilometre/hour. The faster train pass to the slowest train in 36 seconds.
The length of each train is-
Solution:
Let ,
The length of each train is x meter
Distance will be x+x = 2x
Relative Speed =46-36
=10 km/hr
=10*(5/18)
= 25/9 m/sec
Distance = Speed*Time
Here,
2x = (25/9) X 36
or,x= 50 (Ans)

2.A man buys tk 20 shares paying 9% dividend. The man wants to have an interest of 12% on
his money. The market value of each share is-
Solution:
Divided on Tk 20
=(9*20)/100
=9/5
Tk 12 income on Tk 100
Tk 9/5 ------------=(100/12*9/5)
=15 Tk
Answer:15 Tk
3.By selling an article for TK 100 a man gains tk 15 then his gain percentage is -
Solution:
Here,
Selling Price = tk 100
Gain=Tk 15,
Buying or cost price
=(100-15)=Tk 85,
Therefore Gain % =[(15*100)/85]=17.64%
Answer:17.64%
4.The age of a man is 3 times the sum of the age of his two sons. 5 years hence, his age will
be double of the sum of the ages of his sons the father's present age is-
Solution:
Let ,
The sum of present ages of the two sons be x
Father’s present age = 3x
Here,
(3x + 5) = 2 (x + 10)
Or,3x + 5 = 2x + 20
Or,x = 15.
Father’s present age is
(3*15)=45 years(Answer)

5. If tk 64 amount to 83.20 in 2 years what will tk 86 amount to in 4 years in the same rate
percent per annum?
Solution:
Let,
Rate of interest be=r
We know
P=(100*Amount)/(100+r*t)
Or,64=(100*83.20)/(100+2*r)
Or, 6400+128*r=8320
Or,r=15%
Again
P=86 Tk
r=15%
T=4 years
I=(86*15*4/100)
=51.6 Tk
After 4 years amount will be
=(51.6+86)=137.60 Tk
Answer:137.60 Tk

6.A train travels at an average 50 miles per hour for 2.5 hours and then Travels at a speed of
70 miles per hour for 1.5 hours How far did the train travel in the entire 4 hours ?
Solution:
Total distance covers in 4 hrs
=(50*2.5+70*1.5)
=230 miles
Answer:230 miles

8. How many times in a day are the hands of a clock in straight line but opposite in direction?
Solution:
24 hrs in a clock
¤Coincide and Straight line=22
¤Right angle & Acute or obtuse=44

9. The sum of a number and its reciprocal is 1/8 of 34. what is the product of the number and
its square root
Solution:
Let
The number be x.
Then,
x + 1/x = 34*1/8
Or,8x^2 - 34x + 8 = 0
Or,4x^2 - 17x + 4 = 0
Or,(4x - 1)(x - 4) = 0
x=4
Required number
=(4 * √4)
= 4 * 2 = 8.
Answer:8
10. A sum of money invested at compound interest amounts to Tk 4624 in 2 years and to Tk
4913 in 3 years .The sum of money is.
Solution:
Simple interest(3-2)=1 year
=(4913-4624)=289 Tk
SO,
Rate=[(100*289)/(4624*1)]
=25/4 or 6.25%
Let,
Sum of money be=x
Now,
X(1+25/4*100)^2=4624
==================
==================
Or, X=4096 Tk
Answer:4096 Tk

11.If 75% of a number is added to 75. then the result is the number itself .The number is
Solution:
Let,
The number be=x
75% of x + 75 = x
Or, 75/100*x +75=x
Or, 3x/4 + 75=x
==============
==============
Or, x=300
Answer:300
12.A rectangular field be fenced on three sides leaving a side of 20 feet uncovered.if the area
of the field is 680 square feet .how many feet of fencing will be required?
Solution:
We know
Length * Breadth = Area

=> 20 * Breadth = 680

=> Breadth = 34 feet


Area to be fenced
= 2B + L
= 2*34 + 20
= 88 feet
Answer:88 feet
13. Two pipes A and B can fill a cistern in 12 minutes and 15 minutes respectively while a
third pipe C can empty the full tank in 6 minutes .A and B are kept open for 5 minutes in the
beginning and then C is also open. In what time is the cistern emptied -
Solution:
Part filled in 5 min
= 5 * (1/12 + 1/15)
= 5 * 9/60
= 3/4
Part emptied in 1 min when all the pipes are opened
= 1/6 - (1/12 + 1/15)
= 1/60
Now,
1/60 part is emptied in 1min
3/4 part will be emptied in
( 60 * 3/4 )= 45 min
Answer:45 minutes

14.The least number which is a perfect square and is divisible by each of the number is 16,20
and 24 is-
Solution:
LCM of 16,20,24 is =240
Factors of 240 are
=2*2*2*2*3*5
Hence to make 240 a perfect square we need to multiply by 5×3 as they are not in pair.
=240×5×3
=240×5×3
=3600(Answer)

15.Gold is 19 times as heavy as water and copper is 9 times as heavy as water in what ratio
should this be mixed to get an alloy r 15 times as heavy as water :
Solution:
Let,
The mass of 1 volume unit of water be 1
Then 1 volume unit of gold has mass 19,
and
1volume unit of copper has mass 9.

Now let g be the volume of gold used, and let c be the volume of copper. The total mass
divided by the total volume is the density.

(19g + 9c)/(g + c) = 15

19g + 9c = 15g + 15c

4g = 6c
2g = 3c

g/c = 3/2

The volume ratio is 3:2.

16.The smallest prime number is-


Ans:2

17. X can do a piece of work in 40 days he works at it for 8 days and then Y finished it in 16
days. How long will they take together take to complete the work
Solution:
Work done by X in 8 days
=(1/40*8)
=1/5
Remaining work
=(1-1/5)
=4/5 portion
Now,
4/5 portion work is done by Y in 16
Whole work Y done
=(16*5/4)
=20 days
Both X & Y 1 days work
=(1/40 + 1/20)
=3/40
Hence, X and Y will together complete in=40/3 or 13.33 days
Answer:13.33 days

18. In a 300 m race A beats B by 22.5 metre or 6 seconds. B's time over the course is
Solution:
B runs 22.5 m in 6 seconds
B runs 300 m in (6×300)/22.5
= 80 seconds
B's time over the course = 80 seconds
Answer:80 second

19.if the mean of 5 observations X, X + 2 ,X + 4, x + 6 and X + 8 is 11 then the mean of the


last 3 observations is-
Solution:
First, you need to solve for x:

(x + x + 2 + x + 4 + x + 6 + x + 8) / 5 = 11

x + x + 2 + x + 4 + x + 6 + x + 8 = 55

5x + 20 = 55

5x = 35
x=7

Now, substitute for x in each of the last three terms:

x + 4 + x + 6 + x + 8 = (7 + 4) + (7 + 6) + (7 + 8) = (11) + (13) + (15) = 39 / 3 = 13

The mean is 13.

20. A,B,C started business with their investment in the ratio 1 :3 :5 .After 4 month A invested
the same amount as before and B as well as C withdrew half of their investment .The ratio of
their profit at the end of the year is-
Solution:
Let
Their initial investments be x, 3x and 5x respectively
Then
A:B:C
=(x*4+2x*8):(3x*4+3x/2*8):(5x*4+5x/2*8)
=20x:24x:40x
=5:6:10(Answer)

21. √(√176 + √2401) is equal to-

Ans:15

22.49 pumps can empty a reservoir n 6.5 days, working 8 hours a day . If 196 pumps are used
for 5 hours each day ,then the same work will be completed in-
Solution:
Easy solution
(M1*T1)/W1=(M2*T2)/W2
Or,(49*6.5*8)/1=196*5*T2/1
Or, T2=13/5 or 2.6 days
Answer:13/5 days

¤Agrani Bank SO Preliminary-2017


¤Date:09.06.2017
¤৩ ক ঑
=========================================
Solution,Edited and Complete
BY
Yousuf Ali
=======================================

Question-1:A gas tank is 1/5 full and requires 32 gallons to make it 3/7 th full.What is the
capacity of the tank ?
Solution:
Let,
Capacity of the tank is=x gallons
According to the question,
3x/7-x/5=32
Or, (15x-7x)/35=32
Or,8x=32*35
Or, x=140
Answer:140 gallons

Question-2:On selling 17 balls at Tk720,there is a loss equal to the cost price of 5 balls .The
cost price of a ball is
Solution:
Let,
The cost price of a ball is x Tk
The cost price of 17 balls=17 x Tk
The cost price of 5 balls =5x Tk
We know,
Loss =cost price-selling price
Or,5x=17x -720
============
============
Or, x=60 Tk
Answer:60 Tk

Question-3:A and B can do a piece of work in 9 days,B and c in 12 days,A and C in 18


days.If all of them work together,then how much time will they take to finish the same work?
Solution:
A and B can do in 1 day=1/9 portion
B & C can do in 1 day =1/12 portion
C and A can do in 1 day=1/18 portion
2(A+B+C) can do in 1 day
=(1/9+1/12+1/18)
=9/36
=1/4
So,
(A+B+C) can do in 1 day=1/(4*2)
A ,B and C can do the whole work
=1/(1/8)
=8 days
Answer:8 days

Question-4:Two fifth of one fourth of three seventh of a number is 15 .what is the half of the
number?
Solution:
Let,
The number be=x
According to the question,
X*(2/5*1/4*3/7)=15
Or, X=350
Half of that number
=350/2
=175
Answer:175
Question-5:In a class 75% passed in English,60% in Mathematics and 25% failed in both the
subjects.What percentage who passed in both subjects?
Solution:
MCQ Ways...
Total=All single-All both+None
100=75+60-Both passed+25
Or,Both subjects=60
Answer:60%
Note: ক ক ক ঑ ক ১০০ ।

Question-6:A person makes a profit of 10% on 25% of the quantity and a loss of 20% on the
rest of the quantity.What is the gain or loss in percentage on the whole?
Solution:
Let,
100 quantity cost price 100 Tk
25% quantity=100*25%=25
Profit 10%=25+25 of 10%=27.5
75 quantity loss 20%
=75-75 of 20%
=60
Total selling=(60+27.5)=87.5
Loss=(100-87.5)=12.5%
Answer:12.5% loss

Question-7:Increasing the original price of an item by 10% the decreasing by 20% and then
again increasing the price by 10% is equivalent to
Solution:
Let,
Original price=100 Tk
10% increase=110 Tk
20% decrease=(110-110*20%)
=88 Tk
Again,
10% increase =(88+88 of 10%)
=96.8 Tk
Decreasing=(100-96.8)=3.2%
Answer:3.2% decrease

Question-8:A family had provision of food for 15 days.After 5 days 8 guests came and the
provision lasted 6 days.How many are the members the family
Solution:
Suppose,
The number of members in the family=x
X person can consume in 15 days
=1 portion provision
1 person can consume in 1 day
=1/15x portion
According to the question,
5x/15x+6(x+8)/15x=1
==================
==================
Or, x =12
Answer:12

Question-9:In a row of trees,a tree is 7th from the left and 14th from the right end.How many
trees are there in the row ?
Solution:
(14+7)-1=20
Answer:20

Question-10:the average age of 12 children is 15 years.If another child comes the average age
comes to 13.What is the age of the new child?
Solution:
Age of new child
=(13*13-12*15)
=-11
Answer:Wrong

Question-11:A train 150 m long and running at a speed of 60 km per hour takes 30 seconds to
cross a bridge.What is a length of the bridge?
Solution:
Let,
Length of bridge=x m
Speed of train
=60*(5/18)
=50/3 mps
We know,
Distance=speed*time
Or, 150+x=60*(50*3)
================
================
Or, x=350
Answer:350 m

Question-12:A dishonest shopkeeper professes to sell ghee at his cost price.But he uses a
false weight of 950 g for a kg .His gain percentage is
Solution:
Gain percentage
=[(1000-950)/950*100]
=5.26%
Answer:5.26%

Question-13:A sum of money at simple internet amounts to Tk 2800 in 2 years and to Tk


3250 in 5 years at a rate of
Solution:
Principal+5 years interest=3250 Tk
Principal+2 years interest=2800 Tk
--------------------------------------------------------
3 years interest=450 Tk
1 ---------------------=150
2-----------------------=150*2=300 Tk
Principal=(2800-300)=2500 Tk
Rate of interest
=[(150/2500)*100]%
=6%
Answer:6%

Question-15:A dog takes 4 leaps for every 5 leaps of a hare but 3 leaps of the dog is equal to
4 leaps of the hare.Compare their speeds
Solution:
According to the question,
Dog 3 leaps is equal to 4 leaps hare
Dog 4 leaps is equal to
=(4*4)/3=16/3
Ratio of their speed
=16/3:5
=16:15
Answer:16:15

Question-16:In a mixture of liters milk and water are in the ratio of 3:2.How much water
would be added to the mixture to make the two equal?
Solution:
Milk in the mixture
=50*(3/3+2)
=30 liters
Water in the mixture
=(50-30)
=20 liters
Suppose,
X water liter would be added to the mixture to make half water & half milk
30:(20+x)=1/2:1/2
===============
Or, x=10
Answer:10 liters

Question-17:If x is 90% if y then what percent of x is y ?


Solution:
X=.9y
y=x/.9=1.111x =111.1% of x
Answer:111.1%
Question-18:A room of size 5m *3m and height 3m requires walls and celiing painting.what
is the area to be painted?
Solution:
Total area of painted
=The ceiling+ four walls
=length*breadth+2[L+b]h
=5*3+2[5+3]*3
=63 square m
Answer:63 sq.m
Question-19:The length and breadth of a square are increased by 40% and 30% respectively
.The area of the resulting rectangle exceeds the area of the square by
Solution:
Short Cut way
Area changed
=30+40+(30*40)/100
=82%
Answer:82%

Question-20:A husband and wife have six married sons.Each of them has four children.The
total number in the family is
Solution:
Total members
=(1+1+6+6+6*4)
=38
Answer:38

Question-21:A and B together have Tk 1210.if 4/15 of A's amount is equal to 2/5 of B's
amount.What is the amount B has?
Solution:
Let,
B's amount be=x
A's amount be=(1210-x)
Now,
4(1210-x)/15=2x/5
================
================
Or,x=484
Answer:484 Tk
Question-22:
5^-3+5^-3+5^-3+5^-3+5^-3
Answer:5^-2

Question-23:If two angle are said to be complementary angles and one angle is 52 then the
other is
Answer:38 degree

Question-24:How many degrees are between the hands of a clock b a 3:30


Solution:
Angle=[(11M -60 H)/2]
=[(11*30-60*3)/2]
=75 degree
Answer:75

Question-25:An angle which is less than 360 degree and greater than 80 degree is called
Ans:Reflex angle
Question-26: What is 200% of .010?
Answer:0.02
Question-27: if 2x + y=7 and x-y=2 then x+y=?
Answer:4

Question-28:For what value of x is


8^(2x-4)=16^x
Answer:6

Question-29:A father is 61 and his son is 16. In how many years will the father be twice as
old as his son
Solution:
Let,
After x years father age be twice as old as son
Now
61+x =2(16+x)
===========
============
Or, x =29
Answer:29 years

Question-30:Two motorists drove their cars at a speed of 45 km per hour and 50 km per hour
respectively .One car took 10 minutes longer than the other to travel a distance.Find the
distance travelled
Solution:
Suppose
The distance be=x miles
According to the question,
X/45-X/50=10/60
===============

(Pratise Math Without Solution 01 to 10)

1. If 12 candies are sold for tk. 10 then there is a loss of X%. If 12 candies are sold for Tk. 12
then there is a profit of x%. What is the value of x?

2. A man deposited tk. 5000 with a bank that pays interest at the rate of 5% per annum every
six month. The man will withdraw tk. 500 from his principal plus any interest accrued at each
six- month period. How much total interest can he expect to receive?

3. The price of a balcony seat in a theater is 1/3 of the price of a seat in the orchestra. When
the theater is completely sold out, the total receipts from the 600 orchestra seats and the 450
balcony seats are Tk. 4500. What is the price of the orchestra seat?

4. Twenty men can finish a piece of work in 30 days. After how many days should 5 men
leave the work so that it may be finished in 35 days?

5.A man interest and wages from his investment tk. 5000. If he invests double then the wages
increased 50% and total amount is tk. 8000. Determine the original income of the man both in
terms of interest and wages.
06. A man sells an article at a profit of 25%. If he had bought it at 20% less and sold it for Tk.
10.50 less, he would have gained 30%. Find the cost price of the article.

07. A and B can do a piece of work in 18 days, B and C can do it in 24 days, A and C can do
it in 36 days. In how many days will A, B and C finishes it, working together and separately?

08.Shakil started a business investing Tk. 25000 in 2009. In 2010, he invested an additional
amount of Tk. 10000 and Raihan joined him with an amount of Tk. 35000. In 2011, Shakil
invested another additional amount of Tk. 10000 and Jafor joined them with an amount of
Tk.35000. What will be Raihan's share in profit of Tk. 150000 earned at the end of 3 years
from the start of the business in 2009?

09.Dawood invested certain amount in three different schemes A, B and C with the rate of
10% p.a., 12% p.a. and 15% p.a. respectively. If the total interest accrued in one year was Tk.
3200 and the amount invested in Scheme C was 150% of the amount invested in Scheme A
and 240% of the amount invested in Scheme B, what was the amount invested in Scheme B ?

10. The average weight of three men A, B, and C is 84 kg. Another man D joins the group
and the average now becomes 80 kg. If another man E, whose weight is 3 kg more than that
of D, replaces A, then the average weight of B, C, D and E becomes 79 kg. What is the
weight of A?

Answer Sheet:========
1.9.09
2.687.5 Tk
3.6 Tk
4.15 days
5. Interest-1000 Tk
Wage-4000 Tk
6 .50 Tk
7.Together-16 days
A-48 days
B-28.8 days
C-144 days
8.50000 Tk
9.5000 Tk
10.75 kg
Math Solution-2018
#Dhaka_Bank_Cash_2018

#Exam_Taker:Business Studies,DU : : : :

Question-01: A depositor deposited 4000 at x% and 5000 at y% and earned 320 as interest. if he could deposit
5000 at x% and 4000 at y% then he would earn 310. what is value of x and y.

Solution:

According to the question,

4000× x/100+5000×y/100=320 40x+50y=320

Or, 4x+5y=32----------(1)

In the same way

(5000*x)/100+(4000*y)/100=310

Or, 5x+4y=31----------(2)

By doing (1)*5-(2)*4=»

20x+25y=160 20x+16y=124 -------------------------

Or, 9y=36

Or,y=4

Putting value of y in equation (1)

4x + 5*4=32

Or, x=3

So the value of (x,y)=(3,4) Ans:(3,4)

Question-2:Selling 12 candies at a price of tk 10 yields a loss of x% and selling 12 candies at a price of tk 12


yields a profit of x% ..What is the value of x ? [South East Bank PO-2017]

Solution:

Let,

12 candies cost price= Tk 100

x% loss selling price=(100-x)

Selling price(100-x) then cp 100 tk

sp 10. " {(100*10)/(100-x)}

x% profit selling price=(100+x) tk

Selling price tk(100+x) then cp tk100

Sp. 12. " {(100*12)/(100+x)}

According to the question,


{(100*10)/(100-x)}={(100*12)/(100+x)}

Or, x=9.09

So,the value of X is 9.09

9 Answer:9.09

] #BDBL_SENIOR_OFFICER_2018

#Exam_Taker:Arts Faculty,DU : : : : ===================================

Question-1:The profit of a company is given in Taka by P = 3x²-35x+50, where x is the amount in Taka spent on
advertising. For what values of x does the company make a profit?

Solution:

Here, P=3x² - 35x + 50

Now,if the company makes profit,then P>0

So, 3x² - 35x + 50 > 0 => 3x² - 30x – 5x + 50 > 0

=> 3x(x-10) – 5(x-10) > 0

=> (x-10)(3x-5) > 0--------------(1)

As this equation(1) is greater than 0,

So the value of the two roots must have different values in different intervals.

Now,the equation(1),we have x>10

Or, the value of x less than 5/3 and greater than or equal to 0 i.e . 0 ≤ x<5/3,Because advertising cost can not be
negative

So, the company makes a profit,the values of x={0 ≤ x<5/3 or ,x>10}

Answer:x={0 ≤ x<5/3 or ,x>10} ::: :::

Question-2:An amount of Tk. 7200 is spent to cover the floor of a room by carpet. An amount of Tk. 576 would
be saved if the breadth were 3 meters less. What is the breadth of the room? (এক ক ক ড়
৭২০০ ক খ ।ক ৩ ক খ ৫৭৬ ক।ক ক ?)

: ক , ঘ x . y গ খ z ক

, xyz=৭২০০--------(১)

xz(y-৩)=৭২০০-৫৭৬ =৬৬২৪-----(২)

ক (১) ক (২) গ ক => xyz/xz(y-৩)=৭২০০/৬৬২৪

, y=৩৭.৫ :৩৭.৫
:

৫৭৬ ক ক খ ৩ এ

১ ক ক খ ৩÷৫৭৬

৭২০০ ক ক খ (৩*৭২০০)÷৫৭৬

=৩৭.৫

:৩৭.৫ ::: :::

Question-3:Find the three digit prime number whose sum of the digits is 11 and each digit representing a prime
number. Justify your answer.

Solution:

Since the sum of the 3 digits is 11 and each digit represents a prime number,

So, the number less than 11 .

And the 3 digits may be 2,2,7 or 3,3,5

Because 2+2+7=11 and 3+3+5=11

Now using the digit 2,2,7 we have prime number 227 .

Because other two numbers i.e. 722 and 272 are divisible by 2

and

thus are not prime Similarly 353 is prime number other two numbers 533 & 335 is not prime number

In case of 227,

sum of the digits is 2+2+7 = 11.

And 2, 2, 7 all the digits are prime.

Similarly, In case of 353, sum of the digits is 3+5+3 = 11. And 3, 5, 3 all the digits are prime.

Answer: 227 & 353 ::: :::

Question-4:If a/(q-r) = b/(r-p)=c/(p-q) then show that, a+b+c = pa+qb+rc

Solution:

Let, a/(q-r) = b/(r-p) = c/(p-q) = k

So, a = k(q-r);

b = k(r-p);

and

c = k(p-q) Now,

L.H.S. => a+b+c = k(q-r)+ k(r-p)+ k(p-q) = k(q-r+r-p+p-q) = k x 0 = 0

And, R.H.S. => pa+qb+rc = p*k(q-r)+ q*k(r-p)+ r*k(p-q) = kpq-kpr+kqr-kpq+kpr-kqr = 0

So, L.H.S. = R.H.S. (Showed) ::: :::


Question-5:Prove that a cyclic parallelogram must be a rectangle.

Let ABCD be the cyclic parallelogram

Prove that,ABCD is a rectangle

Since ABCD is a parallelogram

<A = <C -------(1)

And, <A+<C = 180°

since A = C

So, <A+<A = 180°

Or, 2<A = 180°

Or, <A = 90° if any one angle of parallelogram is 90°, the parallelogram is a rectangle

. ::: :::

Question-6:After traveling 108 km, a cyclist observed that he would have required 3 hrs less if he could have
traveled at a speed 3 km/hr more. At what speed did he travel?

Solution:

Let,

The speed be x km/hr

According to the question,

(108/x)-{108/(x+3)}=3

Or,(x-9)(x+12)=0

So, x=9 x=-12[It is not acceptable]

Answer: 9 km/hr ::: :::

Question-7:Solve: x/2 + 6/y = 9;x/3 + 2/y=4

Solution: x/2 + 6/y=9-----------(1)

x/3 + 2/y=4-----------(2)

(ii) x 3 – (i)=» =>

x – x/2 = 3 => x/2 = 3

=> x = 6 From (i) =>

6/2 + 6/y = 9

=> 6/y = 6 Y = 1

Ans. (x, y) = (6, 1)

You might also like